You are on page 1of 127

Sept 7, 2022/ Jan 10, 2023 – Board Review – Dr.

Okon

Px has bleeding peptic ulcer. What would you do? -> ice chips. What would that do? Constrict the vessel, limit bleeding.
Not much you can do for upper GI – fresh plasma. Also cold packs

Cardiovascular Review
Review-Embryology, Gross Anatomy, Histology, Physiology, Pathology, Pathophysiology and Pharmacology

Embryology
- Develops from splanchnic mesoderm at week 3 from Neural Crest Cells
- Primitive heart tube fuses into one and forms 4 dilations and cranial outflow tract
- At 4 weeks beats spontaneously

Sinus venosus → smooth part of Rt Atrium, Coronary Sinus and oblique vein of L Atrium

Primitive Atrium → Trabeculated parts of Rt and L Atria

Primitive Ventricle → Trabeculated part of L ventricle

Bulbus Cordis → Trabeculated part of Rt ventricle (proximal), outflow part of both ventricles (distal)

Truncus Arteriosus (from neural crest cells) → Aorta and pulmonary trunk

- Heart tube → 23 day – rotation – and atrium rotates along with ventricle – upward
- When expect to see heart beating – as early as the 4th week

-
How does neural crest cells contribute → truncus arteriosus – aorta and pulmonary trunk and derived from neural crest
cells – why need to know? clinical – must examine other organs where there are neural crest cells → brain, thymus,
thyroid gland, GI tract (hind gut neural crest cells)
First assignment – check research the difference of neural crest cells – what are derived from this. Truncus arteriosus –
and provide other that are derived from them. Neural crest cells give rise to most of the peripheral nervous system
(PNS) and to several non-neural cell types, including smooth muscle cells of the cardiovascular system, pigment cells in
the skin, teeth, eye, head, face, adrenal glands, and craniofacial bones, cartilage, GI tract and connective tissue.

➔ Melanocytes, adrenal medulla (sympathetic neural system which make Epi and NE), nerves surround GI –
Meisener plexus (Enteric nervous system – submucosal plexus, myenteric plexus that regulates GIT), neurons
around spinal cord (dorsal root ganglion and sympathetic chain ganglia and collateral ganglia) , pia mater around
brain and arachnoid mater and together these 2 make up the Leptomeninges; nerves that supply pharyngeal
arches → CN5, CN7, and CN9 and CN10 and structures of head and neck, ossicle bones in ear

Good summary for further study on neural crest cells - links to 2 very short video links that may help understanding.
https://knowledgeburrow.com/what-is-derived-from-neural-crest-cells/

Fetal Circulation - Fetus cannot breathe on its own – 3 major venous systems - Only one umbilical vein and 2
umbilical arteries→

Umbilical vein - job to carry oxygen blood from mother/placenta to fetus to provide oxygen - drains to ductus venosis

Vitelline vein – drain deoxygenated blood from yolk stalk

Cardinal Vein – carries deoxygenated blood from the placenta

When baby born and umbilical cord is cut – inspect to check for 2 arteries and one vein. If only one artery – look for
congenital abnormalities

3 shunts – Ductus Venosus, Foramen Ovale, and Ductus Arteriosus (vessel which allows bypass of the oxygenated
blood from the pulmonary trunk to the arch of the aorta)

Lungs not functioning – all pressure is on right side – shunting in fetal life

Important to know about oxygen conc and oxygen


level is about 60% as returns
Fetal Structure → Adult remnant
Foramen ovale → Fossa ovalis of the heart

Ductus Arteriosus → Ligamentum arteriosum

Left umbilical Vein – Extra-hepatic portion → Ligamentum teres hepatis

Intra-hepatic portion (ductus venosus) → Ligamentum venosum

Left and right umbilical arteries – Proximal portions → umbilical branches of internal iliac arteries

Distal portions → Medial umbilical ligaments

Septation / Division of the Heart Tube


Begins in week 4, and complete in week 8. Atrial Septation−2 Septa & 2 Foramina

Septum primum (SP) – grows inferiorly towards endocardial cushions - Neural crest cells involved in the development of
the cushions

Foramen primum (FP) - located between the inferior edge of the SP and the endocardial cushion.

Foramen secundum (FS) - forms within the upper part of the SP

Septum secundum (SS) forms to the right of the SP from the roof of the atrium, descends, and partially covers the FS

Atrial – 2 septa and 2 Foramina, cushions are neural crest.

Foramen Ovale is opening formed between SP & SS

Foramen Ovale – allow blood to flow right to left – and joins – overlap part and end, overlap – and blood has higher
pressure on right and pushes the flap open. When pressure changes to higher on left – the neural crest cells –

Foramen ovale (FO) - opening between SP and SS.−Closure occurs after birth−Caused by Increased left atrial
pressure

Patent foramen ovale—caused by failure of septum primum and septum secundum to fuse after birth; most are
left untreated.

Can lead to paradoxical emboli (venous thromboemboli entering the systemic arterial circulation through right-to-left
shunt) as can occur in atrial septal defect (ASD)

Px with DVT – clot can go opening or shunt in heart – PFO is most common atrial septum defect , paradoxical emboli
(paradoxical since the emboli lands in artery rather than vein
Clinical – severed kidney and cyanosis → sample mcqs

A Caucasian neonate born to a 22year old primigravida has severe tachypnea and cyanosis. Physical examination shows
an irritable child with a continuous, machinery-like murmur appreciated between the scapulae. The remainder of the
physical exam shows no abnormalities. Echocardiography shows an aorta that lies anterior to and to the right of the
pulmonary artery. Failure involving which of the following embryologic processes is most likely responsible for this
patient's condition? C fusion – of the septum primum and septum secundum after birth → Patent ductus arteriosus

A 12-month-old child is diagnosed with an atrial septal defect. What is the most common cause of such a congenital
heart malformation? C Incomplete adhesion between septum primum and septum secundum

Atrial Septal Defects


Non-cyanotic congenital, more common in females, left to right shunting, lead to Eisenmenger complex – (long-
standing left-to-right cardiac shunt caused by a congenital heart defect causes pulmonary hypertension and eventual
reversal of the shunt into a cyanotic right-to-left shunt)

Secundum-type ASD:most common ASD - Causes: excessive resorption of the SP or underdevelopment and
reduced size of the SS or both. -Delayed symptoms if small

Primum type ASD:less common than secundum ASD - failure of the septum premium to fuse with the endocardial
cushions - lower aspect of the atrial wall, usually with a normal formed fossa ovalis

Ventricular Septal Defects


Week 7 to week 8, no foramen – 2 parts → 1) muscular part forms most of septum, 2) Membranous – cushion (neural
crest)

Valves – Aortic/pulmonary from endocardial cushions

Mitral/tricuspid from fused endocardial cushions of the AV canal

Ventricular septum defect – look at neural crest cells – this is VERY IMPORTANT- all the valves – derived from neural
crest cells – failure to migrate - more common in males, can be cyanotic and acyonotic, membranous form most
common – VSD most common

Left to right shunt – mixes with blood and goes to lung and pressure on right and reversal of shunt and becomes right to
left shunt

Leads to hypertension – cause and cyanosis

Patent Ductus Arteriosus – failure of ductus arteriosus to close after birth – seen in premature and maternal
Rubella, left to right shunt, acyonotic, continuous machinery-like murmur, kept open by PGE & low O2. What keeps
it closed? TX IV Indomethacin, acetylcholine, histamine Why we don’t prescribe during pregnancy

Eisenmenger complex – left to right shunt, this causes increased blood flow to lungs (pulmonary hypertension) →
right to left shunt and late cyanosis. As child grows, and no correction, the pressure on right builds up and hypertrophy
due to more blood.
Cyanotic Heart Diseases
1. Truncus Arteriosus – one great vessel instead of 2 leave the heart
2. Transposition of Great Arteries – pulmonary artery and aorta are reversed
3. Tricuspid Atresia – valve fails to form and blood can’t flow from rt atrium to rt ventricle
4. Tetralogy of Fallot – 4 defects – pulmonary stenosis, rt vent hypertrophy, overriding aorta, vsd
5. Total Anomalous Pulmonary Venous Return (TAPVR) – pulmonary veins connect to systemic venous system
rather than left atrium

Most common is #4 Tetralogy of Fallot →Stenosis (most important) – cyanotic, right to left shunt

#4 aka membranous interventricular septal defect → Boot shape heart

Assignment – medical conditions and their identifying features on x-ray – first 2 are:

Tetralogy of Fallot and boot shape heart


#3 Transposition of the Great Vessels
Failure of AP septum to develop spiral fashion, aorta → right ventricle , Pulmonary→ left ventricle →→ severe cyanosis

Mixing of oxygenated and deoxygenated blood – associated with VSD (ventricular septal defect), ASD (atrial septal
defect), PDA (patent ductus arteriosus) → Tx surgery soon after birth

“Egg on string” – X-ray - The heart appears globular due to an abnormal


convexity of the right atrial border and left atrial enlargement and
therefore appears like an egg.

#1 Persistent Truncus Arteriosus


Partial development of the AP septum. One large vessel leaves
heart from both ventricles, and has membranous VSD (ventricular
septal defect).

Symptoms – cyanosis, breathing issues, heart murmur, sweating,


rapid HR
Pharyngeal Arch
Develop in 4th week – 6 pairs

1st pair - 1st Part of maxillary artery (branch of external carotid)

2nd Pair - 2nd Stapedial artery and hyoid artery.

3rd pair - 3rd Common carotid artery and proximal part of internal carotid artery.

4th pair - 4th On left, aortic arch; on right, proximal part of right subclavian artery.

5th pair - ? disappears

6th pair - Proximal part of pulmonary arteries and (on left only) ductus arteriosus - The proximal part of the artery
persists as the proximal part of the right pulmonary artery.–The distal part of the artery degenerates. ●On the right,
because the distal part of the right sixth artery degenerates, the right recurrent laryngeal nerve moves superiorly and
hooks around the proximal part of the right subclavian artery, the derivative of the fourth pharyngeal arch artery

Gross Anatomy of the Heart – Borders & Surfaces


4 chambers – 2 atrial and 2 ventricles – 2 atrioventricular valves → Mitral and Tricuspid

2 Aortopulmonic valves – Aortic and Pulmonary valves

Borders – Right Atrium, Left ventricle, Apex – left ventricle L5ICS, Superior border – L and R auricles, Inf Border – Rt
ventricle

Surfaces – Anterior – Rt Vent, Posterior – Lt Atrium, Diaphragm – Rt ventricle, 3 main sulci.

Left Atrium is the most POSTERIOR part of the heart and enlargement (found in mitral stenosis) can lead to
compression of the esophagus (dysphagia) or left recurrent laryngeal nerve – a branch of the vagus causing
HOARSENESS (ORTNER) **Esophagus – px has left heart failure, and → dysphagia – to know clinical mcqs, example
Ortner - hoarseness

Right Ventricle is MOST ANTERIOR part of the heart and most commonly injured in trauma → Stab wound upward
to heart – right ventricle injured, ant, in 3rd of 4th intercostal space

Video – 12 years – stab wound – surgery – VID 20180821 – wa0012 Right ventricle injured
Heart has 3 layers – endocardium – single layer lines inside of heart, farthest from blood supply. Myocardium – thick,
striated muscular layer. Epicardium – visceral pericardium

Pericardium – between the outer fibrous and inner serous - - there is Pericardial Cavity – site of accumulation of fluid
→ Pericardial or Cardiac Tamponade – compression of heart from this fluid build up - Symptoms typically include
those of obstructive shock including shortness of breath, weakness, light headedness, and cough. Beck’s Triad

Clinically –Pericardium innervated by phrenic nerve. Patient with Pericarditis → referred pain to neck, arm or
shoulders

Coronary Circulation
Left anterior descending artery, preferably
known as the anterior interventricular artery (AIV), is
the continuing branch of the left coronary artery.
Supply anterior 2/3 of interventricular septum,
anterolateral papillary muscle and anterior surface of
the Left Ventricle – MOST COMMONLY OCCLUDED

Posterior Descending Artery (PDA) – Posteror


Invterventricular artery is a branch of the right
coronary artery. Supplies the posterior 1/3 walls of
ventricles and posteromedial papillary muscle

Right coronary Artery RCA supplies the AV node and


SA Node. Infarct may cause nodal dysfunction
(bradycardia or heart block)

Right (acute) marginal artery supplies the right


ventricle
Dominance of Circulation
80% - arise from RCA, 15% from LCx

Clinicians alike refer to ‘right’ or ‘left’ dominance with respect to the blood supply of the heart. If the right coronary
artery gives off the posterior interventricular branch (which perfuses the posterior region of the ventricular septum and
the posterolateral aspect of the left ventricle), then the heart would be described as being right dominant; the converse
is also true. In the majority of cases, the right coronary is the dominant artery

A 52year old man is brought to the emergency department for evaluation of fever, chills, and malaise. Cardiac
examination reveals a new holosystolic murmur radiating to the axilla. Blood cultures are obtained and he undergoes
transesophageal echocardiography. The ultrasound probe is placed in the mid-esophagus facing anteriorly and cardiac
chambers are interrogated. Anatomically, which of the following chambers is closest to the probe? A. Right atrium
***B. Left atrium C. Right Ventricle D. Left ventricle

During cardiac examination of a newborn infant, a murmur is detected, and the diagnosis of patent ductus arteriosus is
made. Which of the following best describes the direction of blood flow through the patent ductus arteriosus in this
infant? A. From aorta to left pulmonary artery B. From aorta to left pulmonary vein ***C. From aorta to right
pulmonary artery D. From left pulmonary artery to aorta E. From right pulmonary artery to aorta A and C both can
be correct.

In the newborn nursery, a neonate develops a bluish color when crying. Which of the following congenital defects may
be the cause of the cyanosis? A. Patent ductus arteriosus **B. Persistent truncus arteriosus C. Primum type atrial
septal defect D. Secundum type atrial septal defect E. Ventricular septal defect

Physical examination of a 6-year-old child reveals a heart murmur. An echocardiogram shows an ostium primum type of
atrial septal defect. This defect results from failure of the A. ostium primum to form within the septum primum B.
ostium secundum to form within the septum primum ***C. septum primum to fuse with the endocardial cushions
D. septum primum to fuse with the septum secundum E. septum secundum to fuse with the endocardial cushions

Thursday - Sept 8, 2022

Review – flow of blood

Have to know this very well thoroughly – vital when we treat heart failure and manage different types of heart failure.
Anatomy and histology - Pulmonary lung vs systemic whole body circulation

Arteries – 3 layers, thick walls and elastic → Tunica adventiatia, tunica media, and tunica intima

Arterioles – highest resistance to blood flow *** & regulated by adrenergic receptors

Capillaries – single layer for diffusion and exchange of gases, nutrient and fluid. Oxygen gets from air to nose to lungs,
and then into blood → Thru capillaries, fenestration for movement of fluid and gases

Venules – waste products, and deoxygenated blood then passed to veins

Veins – highest capacitance – max storage of blood in body

Ernest H. Starling – Starling forces – generally driving force of fluid in capillary – hydrostatic pressure and oncotic
pressure

Oncotic Pressure - What determines Oncotic Pressure? - or colloid osmotic-pressure, is a form of osmotic pressure
induced by the proteins, notably albumin, in a blood vessel's plasma (blood/liquid) that causes a pull on fluid back into
the capillary.

Pressure of blood is hydrostatic pressure – pressure that a contained fluid exerts

Protein – allow fluid to be retained in blood vessel – ALBUMIN – most abundant protein in body. Edema is extra fluid in
the interstitium and give Albumin which stays in capillaries (it is too big to pass thru) and it reduces edema by pulling
water back into capillary.

The 4 Starling Forces:


Pc – hydrostatic pressure in capillaries, Pi – hydrostatic pressure in interstitium (area around capillaries)
πc – Oncotic pressure in capillaries, πi – oncotic pressure in interstitium

Hemodynamics – Poiseuille’s Equation – relates flow, pressure and resistance

Resistance = directly related to η viscosity and L length of vessel & inversely to r radius

Resistance will increase when viscosity (thickness of blood) and length increase. Alcohol gel example it moves slow – gel
is thicker or more viscous →ie more resistance than water which would flow faster with less resistance). Example of
plastic straw – and radius – the smaller the size , the greater the resistance – so now apply clinically →

1) Hypotension - will see how we deal with this when we get to drugs
2) Hospital, assignment – and word – outline all the fluids and their different contents and concentrations,
example – saline – in Na and chloride and – 500ml – 154 of , and submit before Monday –
Albumin is very heavy - - when pass thru line – run slow. If you use normal saline – less density – viscous - and
length – in the IV – several gauge of IV lines – 14G, 16G, 18G, 20G and 22G - - largest is 14G – if using 22G – it is
shorter, but smaller radius – so flow would be very slow
Systole - heart contracts Diastole – heart relaxes and when receives blood itself
MAP Mean Arterial Pressure – average p in artery - = 2/3 DP + 1/3 S = DP + 1/3 PP

PP Pulse Pressure = S – D Stroke volume – amt blood ejected from heart – in mls – SV = LVEDV - LVESV

What increases Stroke volume? – these 3 things →make heart pump out more:

Contractility ↑ - anxiety, exercise


Preload ↑– volume oriented -how much venous blood that gets back to heart – if you increase preload – pregnancy
Afterload ↓– mean arterial pressure that the heart has to work against to eject blood and to open valve

Pulse pressure – PP directly proportional to SV and PP is inversely proportional to arterial compliance.


Increased PP in hyperthyroidism, aortic regurgitation (valve backflow LV increase), aortic stiffening (isolated systolic
hypertension in elderly), obstructive sleep apnea (Increased sympathetic tone), anemia, exercise (transient).
Decreased PP in aortic stenosis, cardiogenic shock, cardiac tamponade, advanced HF – heart failure

Compliance – how receptive to receive blood – ex atherosclerosis – decrease in compliance as will not allow arteries
to open – ex – young person – arteries are clear – increase compliance is high and PP is decreased or normal. Anything
that cause compliance will cause PP to move in opposite direction.

Ejection Fraction – what % is ejected stroke volume→ EF= SV/LVEDV x 100 as%. This describes what
percentage of blood was ejected from the heart. If ejection fraction ↑, there will be decrease in End Systolic Volume

TPR – Total Peripheral Resistance – arterioles MAP= CO × TPR. → TPR = MAP/CO − This is largely
determined by the arterioles

Want to understand concept – if increase resistance → MAP increase. If decrease CO by loose blood , the sympathetic
trigger vasoconstriction → TPR ↑. When bleeding, blood pressure regulation → barrow receptor and if blood lost, drop
in stretch and sensitive to stretch. Send message to brain. Brain sends back to medulla - sympathetic → heart, more
blood pump - thru alpha 1 (constrict vessels and resistance increase) and beta receptors (heart muscle pump).
Contractilty – ability of heart to pump out blood by contracton
Stimulated by beta 1 - → increase stroke volume - Catecholamine stimulation via β1 receptor: Activated protein kinase
A, phospholamban phosphorylation, Active Ca2+ ATPase leads to increased Ca2+ storage in sarcoplasmic reticulum,
Activated protein kinase A leads to Ca2+ channel phosphorylation and increased Ca2+ entry lead to increased Ca2+-
induced Ca2+ release Increased intracellular Ca2+, Decreased extracellular Na+ ( decreased activity of Na+/Ca2+
exchanger)

Remember that increase Contractility → increase stroke volume → increase pulse pressure

Electrophysiology – Electrical Activity of the Heart - ECG


Pathway – SA node → AV Node → Bundle of HIS → Purkinje Fibers
Martin Flack and Sir Arthur Keith ~1900 – SA node – group of cells - SA is pace maker – it is fastest in generating . When
look at ECG –> funny sodium channel

How to read an ECG → Rate, Regular rhythm, Axis, Conduction abnormality, Hypertrophic enlarged, Ischemia, Misc

Each small square 40 milisec – box .2 sec –


Height – 10 small square – 1 milivolt mV

Heart Rate – HR – 1500/# small boxes

HR = 300 / # large boxes – between R - R

Normal 60-90 bpm

Rhythm – pacemaker is SA node – P wave

P Wave – Atrial depolarization - contraction

PR wave is delay by AV node – 3-5 squares


allows ventricles time to fill

QRS – Ventricular depolarization - contracts

ST – phase after ejected blood

T Wave – ventricle repolarization -release


Go to site → litfl.com/ Life in the Fast Lane – look at ECK and study them https://litfl.com/ecg-interpretation-video-
lectures/

Rhythm -regular interval between R -pacemaker is SA node - must have P , one P for each QRS and T. Does P come
before. Do all P waves look alike (if not, atrial arrhythmia). P waves upright in Leads 1, 2, 3 and avF – if not then not from
sinus node. All this → sinus normal rhythm – if not →the SA is NOT in charge - when normal the SA is in charge – Look
for abnormal rhythm → example – atrial fibrillation

Axis – the mean direction of the electrical potential during ventricular depolarization -
Look at lead one – is it +ve and then look at Lead F00T - → +ve --_ NORMAL AXIS

Leads 1, 2 and 3 should be positive. If deviation – could be hypertrophy and time for current is taking longer. Or
infarction

Left Axis Deviation LAD- In this example – Lead I – we see the +ve and -ve deflection are
about same - → zero, in lead II & III, much bigger -ve . Whenever you have -ve in lead II – 3
large boxes → -3 minus 0 = so this is a negative deflection downward. Something is
shifting current to left side → normal deviation, WPW, left ant hemilock, RV pacing,
elevated diaphragm, lead misplacement & endocardial cushion effect.

Right Axis Deviation – in this example – we see a -ve deviation in Lead I, but a
positive in Leads II and III. → rt vent hypertrophy, lt post hemiblock, pulmonary
embolism, COPD, Lateral MI, WPW, dextrocardia or septal defects.

Prolongation of all QRS complexes – example of Left bundle branch block LBBB- aortic stenosis with prolonged QRS

QRS – longer than 3 squares, >120msec – if longer- → delayed conduction across ventricles – bundle branch, ventricular
hypertrophy, cardiomyopathy, WPW, hyperkalemia or drugs – eg TCA’s

Wolff-Parkinson-White Syndrome – congenital accessory pathway


– Delta Wave – slurring slow rise of QRS, QRS prolong >110ms,
discordant ST-segment

PR interval longer than the 3-5 squares normal → a block of AV node conducting further

Atrial flutter – is regular, but too fast, a form of supraventricular


tachycardia caused by a re-entry circuit within the right atrium. The
length of the re-entry circuit corresponds to the size of the right atrium,
resulting in a fairly predictable atrial rate of around 300 bpm
PR Interval Delay – in the AV conduction results in prolonged PR
interval → Heart Block

1st Degree Heart Block – prolonged PR – 6 squares in this ex

In this example, the P wave is with T, and the P-R interval is 6 small
squares. 0.28 secs → 1st degree

2nd Degree AV Heart Block – Mobitz Type I

Mobitz Type II – failure of conduction of His-Purkinje, non-


conducted P waves

3rd Degree Heart Block – constant P-P and R-R, but variable P-R
intervals. Short P-R interval , 120 msec

ST-segment elevation myocardial infarction (STEMI) –


transmural – all layers of heart effected
subendocardial infarction – area furthest away from blood
supply, in leads facing affected area

Most common occluded vessel – Left Anterior Descending LAD

Right Coronary Artery RCA – prominent R wave, V1 and V2

Left Circumflex – prominent R, V1 and V2

RR intervals are irregular and P wave is absent. Completely arrhythmic → Diagnosis: Atrial Fibrillation

Ventricular Fibrillation – No P-wave, QRS complex or T-wave seen. Cardiac arrest ensues when
ventricular contractions cease or are meaningless. This immediately leads to syncope and death

ST segment depression, T wave inversion (common V1-V6) – Ischemia, Infarction


Which of the following is the COMMONEST cause of long QT? A. low Na B. low Ca C. Low Mg **D. Low K

Friday September 9, 2022

SA Node - Action Potential of the SA Node – why is this important? → does not
have outside stimuli – known as pacemaker, and conduction is very fast.

3 phases – Phase 4 – open funny Na channel, Ca also enters until action pot is
reached → Phase 0 – voltage gated Ca channel open → Phase 3 – Ca channels close
and K channel open to repolarize – never any rest

Ventricular Muscle – 5 phases – 0, 1, 2, 3 & 4- and needs a stimuli. Ions are


responsible for each phase

Phase 0 – Na channels open once threshold pot has been reached and influx of
NA → Phase 1 – Na channels closed, K influx begins → Phase 2 – Ca channels
open – K efflux balanced with Ca influx which triggers release of stored Ca from
sarcoplasmic reticulum in cardiac myocyte leads to contraction → Phase 3 – Ca
channel inactived, K efflux leads to depolarization → Phase 4 – resting, K enters
(need stimulation from SA node action potential

Jan 12, 2023

Frank Starling Curve


More venous return = more preload → larger ventricular end-diastolic volume → more muscle fiber stretch → stronger
contraction.

Ensures that if more blood returns to the heart, the heart pumps that extra blood by increasing stroke volume.

Contractility is the force of contraction at a given preload, and therefore increasing preload does nothing to contractility
A – see muscles not fully stretched – by time get to B – muscle begin to stretch further which allows stroke volume to
increase. B to C – see Stroke volume increases but get to curve, no further increase in stroke volume. Limit to how much
the blood is ejected. Green and red – changes to heart – red – exercise, or digoxin and increase in stroke volume, but in
red – decrease – use of beta blockers or heart failure.

When you Exercise – force has increased – the greater the stretch of cardiac muscle the greater the force of contraction
– but there is a limit to that – that is what Starling said.

Now to red line – heart failure – not pumping blood sufficiently – preload has increased and stroke volume decreased –
receiving blood but not able. When given with positive inotrope → preload is reduced

Cardiac and vascular function curves – Cardiac Output vs Right Atrial Pressure

Intersection of curves= operating point of heart – venous return and CO are equal

Normal pt – and you can read MSP – mean systemic pressure

Increase in inotropy – to Pt 1 – increase in the contractility of the heart, but RAP decreases. Heart is pumping as much as
it can – example exercise.

Pt 2 decrease in inotropy – decrease in contractility – increase in RAP – example HF, narcotic overdose
Volume – #3 if we increase the volume we increase the RAP – example give
blood, when veins constrict → increase cardiac output

#4 – decrease in volume – decrease RAP – example loss of blood

TPR – if we go from normal to #5 or 6 – not much change in RAP, since the


total peripheral resistance does not have much effect on RAP

= MAP/CO

5 – vasopressin
Frank-Starling Curve- More venous return and #6 -→
= more preload exercise
larger ventricular end-diastolic volume →
more muscle fiber stretch → stronger contraction.

Ensures that if more blood returns to the heart, the heart can pump that extra blood by increasing stroke
volume.

Contractility is the force of contraction at a given preload, and therefore increasing preload does nothing to
contractility
What is normal rate of blood flow – blood comes from sup and inf vena cava, → right atrium – review of heart from last
day
Cardiac Cycle

See red line – the Ventricular pressure. AV valve closes and pressure rises to 80 – that is pressure of Aorta and so valves
open.

ECG – T wave, QRS – see correlation, and for S1 and S2 – know when they occur – Jugular Vein Pressure Curve

Waveforms a, c, and v waves as well as x and y descents.


a wave represents the atrial kick (atrial contraction) for
the right atrium – ABSENT → Atrial Fibrillation

- PROMINENT → Tricuspid Regurgitation


c wave represents contraction of the right ventricle, causing
the tricuspid to bow inward toward the right atrium
x descent – relaxation of atrium
V wave occurs during venous filling
y descent - During ventricular diastole, the tricuspid valve
opens and the blood in the atrium will empty into the right
ventricle. Prominent in constrictive pericarditis, absent in
cardiac tamponade

What happens in the right atrium – a-wave – atrial systole contraction – atrial waveform → c-wave contraction of rt
ventricle - systole– tricuspid valve closes at x descent – relaxation of atrium. V-wave is venous filling and y-descent
emptying

Clinical - Summary
A-wave is atrial kick will be absent in atrial fibrillation – so that there will be no P wave on the ECG

Prominent a-wave – Tricuspid regurgitation c-v wave – large upward deflection seen in AV regurgitation

c-v wave – Large Upward Defection → AV Regurgitation


Pressure – Volume Loop
A – end systolic volume at end of systole

B – mitral opens –

C- final preload – mitral close and pressure increase

D – pressure increase to 90 and aortic valve opens → eject to A

A to B – isovolumeric contraction

C to D – isovolumeric

Pressure Volume Loop – A – Aortic valve closes, B Mitral valve opens, C Mitral valve Closes, D Aortic valve opens

Black – - - is stroke volume

Isometric contraction…..
Monday September 12, 2022 – Jan 13, 2023

Heart Sounds and Murmurs

Heart sounds – beating of the heart -lub dub vs Murmurs – extra sound as blood flows
Normal blood flow is laminar and silent. Turbulent blood flow produces murmur in the heart

Heart sounds are generated by the closing of valves. Areas to auscultate for heart sounds−aortic, pulmonic,
tricuspid, mitral.

S1 - closure of the tricuspid and mitral valves.−never any splitting


S2 - closure of the pulmonic and aortic valves, referred to as P2 and A2, respectively
S3 – normal in children – in adults → sign of volume overload, early diastole and after the S2 - low pitched &
produced in the ventricle at termination of rapid filling. Often normal in children and px with hyper-dynamic
flow states

S4 – atrial kick – stiff and non-compliant ventricle – restrictive myopathy - low-intensity sound just before S1

2nd heart sound - Normal inspiration – splitting – caused by increase flow on right side and taking time to empty
out. Aortic before Pulmonary −Undergoes a physiological splitting during INSPIRATION−Why? −P2 is generated
after A2→ S2 is normally split because the aortic valve (A2) closes before the pulmonary valve (P2). The closing
pressure (the diastolic arterial pressure) on the left is 80 mmHg as compared to only 10 mmHg on the right. This higher
closing pressure leads to earlier closure of the aortic valve

Disorders of the 2nd Heart Sound


Paradoxical Splitting: aortic valve paradoxically closes after the pulmonic valve−Caused by a delay in left ventricular
emptying−Occurs during expiration−Examples: LBBB – left bod branch block and AS
Wide Splitting: when the P2–A2 split is longer than usual.−Caused by a delay in right ventricular emptying.−Examples:
RBBB, pulmonic stenosis

Fixed Splitting - Splitting occurs on both inspiration and expiration.−Atrial septal defect (ASD)

A P E To Man

Murmurs are extra sounds as blood flows – may be normal or indicate disorder.
Blood vessel – Bruit
Stenosis is a problem with opening a valve because it has been narrowed.
Regurgitation (or insufficiency) is a problem with keeping a valve closed

Hand grip – increases afterload – will increase aortic & mitral regurg & ventricular septal defect
Decrease the murmurs hypertrophic obstructive cardiomyopathy, mitral valve prolapse

Squatting – or raising legs - increases preload - → aortic and mitral stenosis, aortic & mitral regurg
Decreases the murmurs hypertrophic obstructive cardiomyopathy and mitral valve prolapse
Valsalva Maneuver – decreases preload – reduce pressure in thoracis cavity by compressing the cavity.

Systolic Murmurs Diastolic Murmurs


Aortic Stenosis Aortic Regurgitation
Pulmonary Stenosis Pulmonary Regurgitation
Mitral Regurgitation Mitral Stenosis
Tricuspid Regurgitation Tricuspid Stenosis
Mitral Valve Prolapse

Systolic Murmurs – Aortic Stenosis, Pulmonary Stenosis, Mitral Regurg, Tricuspid Regurg, Mitra Vave Prolapse
Aortic Stenosis – Increased EDP – symptoms angina, syncope, dyspnea, edema. Exam – narrow Pulse Pressure
Etiology−Congenital (bicuspid, unicuspid valve),−Calcification (wear and tear),−Rheumatic diseaseNormal aortic valve
area = 3-4 cm2−Mild AS >1.5 cm2−Moderate AS 1.0 to 1.5 cm2−Severe AS <1.0 cm2−Critical AS <0.5 cm2

Pathophysiology−Outflow obstruction → increased


EDP → concentric LVH → LV failure → CHF,
subendocardial ischemia

Symptoms−Exertional angina, syncope, dyspnea, PND,


orthopnea, peripheral edema

Physical Exam−Narrow pulse pressure, brachial-radial


delay, pulsus parvus et tardus, sustained
PMI−Auscultation: crescendo-decrescendo SEM
radiating to (R) clavicle and carotid, musical quality at
apex (Gallavardin phenomenon), S4, soft S2 with
paradoxical splitting, S3 (late)

Investigations−ECG: LVH and strain, LBBB, LAE, AFib−CXR: post-stenotic aortic root dilatation, calcified valve, LVH, LAE,
CHF−Echo: reduced valve area, pressure gradient, LVH, reduced LV function

Treatment−Asymptomatic: serial echos, avoid exertion −Symptomatic: avoid nitrates/arterial dilators and ACEI in severe
AS
Surgery if: symptomatic or asymptomatic severe AS with low-to-intermediate surgical riskand who meet an indication
(survival benefit, symptom improvement, and improvement inLV systolic function) - Surgical Options−Valve
replacement: aortic rheumatic valve disease and trileaflet valve−– prior to pregnancy (if AS significant)−– balloon
valvuloplasty (in very young or as a bridge to valve replacement for symptomatic patients with severe AS)

Interventional Options−Percutaneous valve replacement (transfemoral or transapical approach) is an option in selected


symptomatic patients with severe AS who are at an intermediate to prohibitive risk for surgery with a predicted post-
procedure survival ≥12 mo

Outflow obstruction – the pressure has to increase to overcome → hypertrophy


and CHF.

Tx – asymptomatic – avid exertion. Symptomatic, avoid nitrates & ACE Inhibitors,


surgery only severe

Pulmonary Stenosis – Etiology - rheumatic disease (rare), congenital, carcinoid syndrome, Pathophysiology -
RV pressure↑ → rt heart failure, Symptoms - chest pain, fatigue, edema, Physical Exam - murmur at 2nd left intercostal
space (worse when inspire), rt side S4 Investigations – ECG: RVH, CXR – prominent pulmonary artery, enlarged RV, Tx –
Balloon valvuloplasty if severe

Mitral Valve regurgitation ESV – decrease – when


contracting and mitral valve supposed to be closed, some blood
ejected in left atrium, some into left atrium and some to aorta. EDV
increase. SV increase

Etiology Marfan, acute MI, apical thrill, murmur at apex (radiates to


axilla), S3, Why is CO reduced → blood flows back into atrium, reducing
blood in ventricle. Tx decrease afterload (ACE inhibitors), decreased
preload with diuretics. Surgery – try to repair before surgery due to
endocarditis, rejection, coagulation risks.

Investigation – Swan-Ganz Catheter: prominent LA v wave.


Tx diuretics for preload, decrease ACE inhibitors for afterload

Surgical options – valve repair >7 px – advantage low rate endocarditis, less chance re-operation, no anticoagulation
Etiology−Mitral valve prolapse, congenital cleft leaflets, LV dilatation/aneurysm (CHF, DCM, myocarditis), IE
abscess, Marfan’s syndrome, HOCM, acute MI, myxoma, mitral valve annulus calcification, chordae/papillary
muscle trauma/ischemia/rupture (acute), rheumatic disease Pathophysiology−Reduced CO → increased LV
and LA pressure → LV and LA dilatation →CHF and pulmonary HTN Symptoms−Dyspnea, PND, orthopnea,
palpitations, peripheral edema
↑EDV of blood – only about 300 mls out of 500 is ejected into aorta, 100 mls regurgs back into atrium - → added to next
cycle – and added . ejection fraction is constant – 300/500 = 60%, 500+100= 600 and then contracts – so 60% of 600 will
be eected again =36020 ↑strke volume, -> 240 left → amount in heart increases over time. ↓Cardiac output , but there
is a limit from Frank Starling → CHF and HTN → dypsnesa, at night woke up by can’t breathe at night nocturnal - The
definition of orthopnea comes from the Greek words for upright ( ortho) and breath ( pnoea ). It's often a sign of heart
failure , but can be caused by other conditions, such as lung diseases, other heart conditions, and obesity. What Is
Orthopnea? People who have orthopnea will begin having trouble breathing when they lie down

Physical Exam−Displaced hyperdynamic apex, left parasternal lift, apical thrill−Auscultation: holosystolic
murmur at apex, radiating to axilla ± mid-diastolic rumble, loud S2 (if pulmonary HTN), S3 Investigations−ECG:
LAE, left atrial delay (bifid P waves), ± LVH; CXR: LVH, LAE, pulmonary venous HTN; Echo: etiology and
severity of MR, LV function, leaflets; Swan-Ganz Catheter: prominent LA “v” wave Treatment−Asymptomatic:
serial echos; Symptomatic: decrease preload volume blood (with diuretics), decrease afterload (ACEI) for
severe MR and poor surgical candidates; stabilize acute MR with vasodilators before surgery Surgery if: acute
MR with CHF, papillary muscle rupture, NYHA class III-IV CHF, AF, increasing LV size or worsening LV function,
earlier surgery if valve repairable (>90% likelihood) and patient is low-risk for surgery

Surgical Options−Valve repair: >75% of pts with MR and myxomatous mitral valve prolapse – annuloplasty
rings, leaflet repair, chordae transfers/shorten/replacement−Valve replacement: failure of repair, heavily
calcified annulus−Advantage of repair: low rate of endocarditis, no anticoagulation, less chance of re-
operation Try to valve repair first , no rejection, – while replacement if not repaired, or if heavily calcified.
NB– echo (show , chest x ray and ECG want to order these 3 tests, Shan Ganz catheter

Holosystolic murmurs→ Pansystolic


Important points to remember → Mitral regurgitation (radiates to the axilla) , tricuspid
regurgitation (at LLSB ↑ inspiration), ventricular septal defect

Tricuspid Regurgitation – RV dilation, rheumatic disease, IV drug abuse destroys valves on right side, congenital
Ebstein anomaly, carcinoid. Pathophysiology RV dilatation → Tricuspid Regurg (TR) and more RV dilatation→ Leads to
right side heart failure. →Symptoms- peripheral edema, fatigue, palpitations. Exam – “cv” wave in JVP, +ve
abdominojugular reflux, holosystolic murmur at LLSB accentuated by inspiration, left parasternal life. Investigation
ECG/Xray – Rt Atrial & RV Enlargement, AFib. CXR RAE and RV enlarged. Echo: diagnostic TX reduce preload with
diuretics. Surgery only if doing other mitral valve replacement. Repair – annuloplasty

Mitral Valve Prolapse - associated with Marfan, pectus excavatum. Symptoms – stabbing chest pain, anxiety. Dx –
mid systolic click. Mid to late systolic murmur at Apex – accentuated by Valsalva or squat-to-stand maneuvers. Tx β,
avoid caffeine. Mitral valve repair

Etiology−Myxomatous degeneration of chordae, thick, bulky leaflets that crowd orifice, associated with Marfan’s
syndrome, pectus excavatum, straight back syndrome, other MSK abnormalities; <3% of population
Pathophysiology−Mitral valve displaced into LA during systole; no causal mechanisms found for symptoms
Symptoms−Prolonged, stabbing chest pain, dyspnea, anxiety/panic, palpitations, fatigue, presyncope Physical
Exam−Auscultation: mid-systolic click (due to billowing of mitral leaflet into LA; tensing of redundant valve tissue); mid
to late systolic murmur at apex, accentuated by Valsalva or squat-to-stand maneuvers Investigations−ECG: non-specific
ST-T wave changes, paroxysmal SVT, ventricular ectopy−Echo: systolic displacement of thickened mitral valve leaflets
into LA Treatment−Asymptomatic: no treatment; reassurance−Symptomatic: β-blockers and avoidance of stimulants
(caffeine) for significant palpitations, anticoagulation if AFib Surgical Options−Mitral valve surgery (repair favoured over
replacement) if symptomatic and significant MR

Diastolic Murmurs - Aortic Regurg, Pulmonary regurg, Mitral Stenosis, Tricuspid Stenosis
Aortic Regurgitation – common in Marfan, atherosclerosis, CT disease Ehlers Danlos, Onset from trauma, failed
valve, IT. Volume overload. High systolic and low diastolic BP. Symptoms dyspnea, orthopnea, syncope, angina. Water
hammer pulse, Auscultation – murmur leaning forward on expiration, soft S1, absent S2, S3 late.

Etiology−Supravalvular: aortic root disease (Marfan’s, atherosclerosis and dissecting aneurysm, connective tissue
disease) −Valvular: congenital (bicuspid aortic valve, large VSD), IE−Acute Onset: IE, aortic dissection, trauma, failed
prosthetic valve

Aortic Regurgitation – valves are unable to close when supposed to. Frank Starling, the more the preload, the greater
the contraction. Initially heart will try to eject. EDV will increase – due to blood leaking back in. Large Pulse Pressure

Pathophysiology−Volume overload → LV dilatation → increased SV, high sBP and low dBP → increased wall tension →
pressure overload → LVH (low dBP → decreased coronary perfusion) Symptoms−Usually only becomes symptomatic
late in disease when LV failure develops−Dyspnea, orthopnea, PND, syncope, angina

Physical Exam−Waterhammer pulse, bisferiens pulse, femoral-brachial sBP >20 mmHg (Hill’s test wide pulse pressure),
hyperdynamic apex, displaced PMI, heaving apex−Auscultation: early decrescendo diastolic murmur at LLSB (cusp
pathology) or RLSB (aortic root pathology), best heard sitting, leaning forward, on full expiration, soft S1, absent S2, S3
(late) Investigations−ECG: LVH, LAE−CXR: LVH, LAE, aortic root dilatation−Echo/TTE: quantify AR, leaflet or aortic root
anomalies−Cath: if >40 yr and surgical candidate – to assess for ischemic heart disease−Exercise testing: hypotension
with exercise Treatment−Asymptomatic: serial echos, afterload reduction (e.g. ACEI, nifedipine,
hydralazine)−Symptomatic: avoid exertion, treat CHF−Surgery if: NYHA class III-IV CHF; LV dilatation and/or LVEF <50%
with/without symptoms−Surgical Options−Valve replacement: most patients−Valve repair: very limited role−Aortic root
replacement (Bentall procedure):−– when ascending aortic aneurysm present, valved conduit used

Exercise testing with hypotension. Tx reduce afterload with ACE Inhibitors, nifedipine, hydralazine. Avoid exertions, treat
CHF. Replace valve. Bentall procedure – aortic root replace.

Know your murmurs and left lower sternal border LLSB → sitting forward on full expiration and soft S1 and absent S2

>40 40 assess for ischemic heart and exercise test

Tx avoid exercise - New York HA Class 3-4 - New York Heart Association (NYHA) Functional Classification - Your physician
will likely “classify” your heart failure condition according to the severity of your symptoms.

Pulmonary Regurgitation -Etiology−Pulmonary HTN, IE, rheumatic disease, tetralogy of Fallot (post-repair)
Pathophysiology−Increased RV volume → increased wall tension → RV hypertrophy →right heart failure
Symptoms−Chest pain, syncope, fatigue, peripheral edema Physical Exam−Early diastolic murmur at LLSB Left Lower
sternal border, Graham Steell (diastolic) murmur 2nd and 3rd left intercostal space increasing with inspiration
Investigations−ECG: RVH−CXR: prominent pulmonary arteries if pulmonary HTN; enlarged RV−Echo: diagnostic
Treatment−Rarely requires treatment; valve replacement (rarely done)−Surgical Options−Pulmonary valve replacement

Distole and systole – how different – place finger at carotid artery and feel – when you listen to lub-dub, when max
pulse and sound coincides with lub – is systole , and then tell early mid and late -

Graham Steell murmur is a heart murmur typically associated with


pulmonary regurgitation. It is a high pitched early diastolic murmur heard
best at the left sternal edge in the second intercostal space with the
patient in full inspiration

Mitral Stenosis – Rheumatic disease, Left Atrial enlargement and pressure→ CHF, worse with tachycardia, AFib
and pregnancy (increase of preload). Symptoms – shortness of breath on exertion, orthopnea, malar flush, blue facies.
Best head with bell while px on left side (left lateral decubitus position). Tx avoid exertion, β-blocker digitalis to increase
diastolic filling time.
Etiology−(MCC)-Rheumatic disease, congenital (rare)Definition−Severe MS is mitral valve area (MVA) <1.5 cm2
Pathophysiology MS → fixed CO and LAE → increased LA pressure → pulmonary vascular resistance and CHF; worse
with AFib (no atrial kick), tachycardia (decreased atrial emptying time) and pregnancy (increased preload)
Symptoms−SOB on exertion, orthopnea, fatigue, palpitations, peripheral edema, malar flush,pinched and blue facies
(severe MS)
Physical Exam−AFib, no “a” wave on JVP, left parasternal lift, palpable diastolic thrill at apex−Auscultation: mid-diastolic
rumble at apex, best heard with bell in left lateral decubitus position following exertion, loud S1, OS following loud P2
(heard best during expiration), long diastolic murmur, and short A2-OS interval correlate with worse MS
Investigations−ECG: NSR/AFib, LAE (P mitrale), RVH, RAD−CXR: LAE, CHF, mitral valve calcification−Echo/TTE: shows
restricted opening of mitral valve−Cath: indicated in concurrent CAD if >40 yr (male) or >50 yr (female)
Treatment−Avoid exertion, fever (increased LA pressure), treat AFib and CHF, increase diastolic filling time (β-blockers,
digitalis)−Surgery if: NYHA class III-IV CHF and failure of medical therapy

Mitral Stenosis – problem with the valves opening. The left atrium pressure increases and can back up. Less blood
entering the ventricle, so less will be ejected – stroke volume decreases. EDV decreases –

Listen with BELL with px at left side – loud S1 and opening snap → shorter interval the more severe

Invasive Options−Percutaneous balloon valvuloplasty: young rheumatic pts and good leaflet morphology (can be
determined by echo), asymptomatic pts with moderate-severe MS, pulmonary HTN−Contraindication: left atrial
thrombus, moderate MR−Open Mitral Commissurotomy: if mild calcification + leaflet/chordal thickening−– restenosis in
50% pts in 8 yr−Valve replacement: indicated in moderate-severe calcification and severely scarred leaflets

Tricuspid Stenosis – Rheumatic disease, right heart failure → decrease CO. Symptoms peripheral edema,
fatigue. Exam prominent “a” wave in JVP, Kussmaul sign, diastolic rumble 4 th left intercostal space. Tx preolod reduction
with diuretics, slow HR.

Etiology−Rheumatic disease, congenital, carcinoid syndrome, fibroelastosis; usually accompanied by MS (in RHD)
Pathophysiology−Increased RA pressure → right heart failure → decreased CO and fixed on exertion
Symptoms−Peripheral edema, fatigue, palpitations Physical Exam−Prominent “a” waves in JVP, +ve abdominojugular
reflux, Kussmaul’s sign, diastolic rumble 4th left intercostal space. Investigations−ECG: RAE−CXR: dilatation of RA
without pulmonary artery enlargement−Echo: diagnostic Treatment−Preload reduction (diuretics), slow HR−Surgery if:
only if other surgery required (e.g. mitral valve replacement) −Surgical Options−Valve Replacement:−– if severely
diseased valve−– bioprosthesis preferred

Tuesday September 13, 2022

Cardiovascular Pressures
Cardiac Tamponade – equalization of pressures

Swan-Ganz Catheter – pulmonary artery→ capillary – Pulmonary Capillary Wedge Pressure )PCWP) - Can estimate the
Left Atrium by measuring it at this point.

Equalization of pressures – Cardiac Tamponade – Becks Triad

Jan 13/2023 continued

Regulation of Blood Pressure


Regulation of blood pressure occurs with two main mechanisms: NEED THIS FOR HEART FAILURE – Very important

(1) baroreceptor reflex - pressure sensors - Carotid Body – Glossopharyngeal nerve - Aortic Arch – Vagus nerve-
Modulate parasympathetic and sympathetic tone. Increase in blood pressure leads to increased stretch leads to
increased firing of the baroreceptors.−increase parasympathetic tone through the vagus nerve (CN X) on the SA node to
decrease heart rate −decrease sympathetic tone.

Decrease in blood pressure (loss of blood) causes less stretch on the pressure leading to decreased stretch which causes
decreased firing of the baroreceptors−Decreased vagal tone to the SA node, resulting in an increased heart
rate.−Increased sympathetic tone to increase heart rate, contractility and vasoconstriction.

(2) the renin-angiotensin-aldosterone axis (Kidney)- JG cells to secrete the enzyme renin into the bloodstream. −The
bloodstream always has angiotensinogen in it (from liver), and renin cleaves this angiotensinogen into angiotensin
I.−Angiotensin I gets cleaved → angiotensin II by angiotensin-converting enzyme (ACE).−Angiotensin II is a potent
vasoconstrictor, increasing systemic vascular resistance (SVR) and therefore ↑blood pressure, and also stimulate
aldosterone release from adrenal medulla. → excretion of K, and increases NAa+ and water follows it, by doing this will
raise BP

End Friday Jan 13th class –


In self study – present 5 questions on cardiac anatomy and physiology - Pathology – cont Sept 13 lecture

Monday Jan 16, 2023 – in St. Vincent - online

Arteriosclerosis - Group of diseases – arterial wall thickening and hardening


Monckeberg Medial Calcific Sclerosis – of medium size arteries and detected by x-ray

Arteriolosclerosis
- Haline – pink glassy arterial wall thickening with luminal narrowing seen in hypertension, diabetes and aging
- Hyperplastic – smooth muscle tunica media –“onion skin” concentric wall thickening – malignant hypertension

Atherosclerosis – vascular – intimal thickening of large and med size arteries → fatty streaks and plaques over decades

Risk Factors – hyperlipidemia, hypertension, smoking, diabetes, obesity, males, sedentary, stress, ↑ homocysteine, oral
contraceptives, ↑age, genetic, alcoholic abuse

Endothelium line artery. Damage and LDL laden


macrophage attach→ become Foam cells, and fatty streak (clinically reversible) and migrate into smooth muscle layer
forming dense fibrous plaque. FA 302 → Chronic ischemia, rupture, thrombosis or embolization

Hypertension – definition elevated bp leading to end-organ damage – reduction of blood supply to kidney – renal
vascular hypertension

D >90, S > 140 mmHg, 25% of population, Blacks more serious affected than whites, risk increases with age (in US it is 80
and 130 values)

Primary – environmental, genetic, diet – 95%

Secondary – 5-10% → causes –from Head to Toe → Cushing’s (tumor in pituitary) , Thyroidism (neck), Coarctation of
aorta, Adrenal Cortex- aldosterone, zona glomerulosa (abdomen) Conn Syndrome, medulla pheochromocytoma (rule of
10), kidneys – Renal vascular hypertension. Can result in end organ damage.

Malignant - 5%, Systolic >180 mmHg and/or Diastolic >120 mmHg, Funduscopic exam may reveal retinal hemorrhages,
exudates and papilledema → death, renal failure, intracerebral hemorrhage, CFR

Add Vascular Neoplasmas – Glomus, H


MCQ – A 44-year-old woman dies as a consequence of a “stroke”. At autopsy, she is found to have a large right basal
ganglia hemorrhage. She has an enlarged 550 gm heart with predominantly left ventricular hypertrophy. Her kidneys are
small, about 80 gm each, with cortical scarring, and microscopically they demonstrate small renal arterioles that have
luminal narrowing from concentric intimal thickening. Which of the following is the most likely condition associated with
her findings? a. hypercholesterolemia **b. hypertensive emergency c. diabetes mellitus d. polycystic kidney disease
e. Monckeberg’s sclerosis

b. – hypertension – highlight – note this – age 44 year dies, why? Stroke. Unlikely must have hypercholesterolemia but
bleed, no blood flow – some form of narrowing of arterioles, concentril thicken – malignant. Options Monckeberg has to
, no calcification, c – but why died, a – kidney → must be b. Relate question to concept we study.

Dig up questions and be familiar with them – will upload lecture notes now

ANEURYSMS AND ARTERIOVENOUS FISTULAS


Aneurysms are congenital or acquired weakness of the vessel wall media, resulting in a localized dilatation or
outpouching. Thrombus formation - Thromboembolism - Compression of nearby structures - Rupture -Dissection

Types:

Atherosclerotic aneurysms - weakening of the media


• secondary to atheroma formation
• typically occur in the abdominal aorta below the renal arteries.
• Half of aortic aneurysms >6cm in diameter will rupture within 10 years
• Those >5 cm are treated surgically
Syphilitic aneurysms
• ascending aorta in tertiary syphilis
• causes an obliterative endarteritis of the vasa vasorum, leading to ischemia and smooth-muscle
atrophy of the aortic media.
• dilate the aortic valve ring, causing aortic insufficiency.
Aortic dissecting aneurysm – will see mediastinal widening prominent aortic knob right pleural effusion
• Predisposing factors: Hypertension and Marfan syndrome
• blood from the vessel lumen enters an intimal tear and dissects through the layers of the media
• Degeneration (cystic medial degeneration) of the tunica media
• Severe tearing pain
• May compress and obstruct the aortic branches
DVT - Varicose veins - Venous insufficiency – in women – lower extremities,

Jeffrey Ndu - MCQs

The 72-year-old patient had prior elective aortic aneurysm resection but returned to a surgeon with the unusual
complaint that both lower legs fell asleep when he had been kneeling on a church bench. Physical examination reveals
tense pulsatile masses in the flexor space behind both knees. The MOST likely diagnosis is: a Baker cysts
** b Popliteal aneurysms c Bursitis d Osteoarthritis e Lesser saphenous thrombophlebitis
The answer is: B. Vascular atherosclerosis is a generalized disease and reconstructive surgery is palliative. Following
resection of an aneurysm in one area, the same propensity exists in others, and popliteal aneurysms are common in
patients with vascular artherosclerosis. If this patient’s cardiac status would allow him to safely tolerate more surgery,
elective repair of these new aneurysms should be considered, keeping in mind that his cardiac status may be quite poor

Wednesday September 14, 2022 - January 17, 223

Myocardial Infarction Myocardial hibernation—potentially reversible LV systolic dysfunction in the setting


of chronic ischemia. Contrast with myocardial stunning, a transient LV systolic dysfunction after a brief episode of acute
ischemia – systolic heart failure – end up with back up of bod into lungs – eft coronary artery -

Myocardial Infarction
Myocardial infarction (MI), colloquially known as "heart attack," is caused by decreased or complete cessation of blood
flow to a portion of the myocardium. Myocardial infarction may be "silent" and go undetected, or it could be a
catastrophic event leading to hemodynamic deterioration and sudden death - Localized area of cardiac muscle
undergoes coagulative necrosis due to ischemia. → Most common cause of death − Coronary atherosclerosis (90%) −
Decreased circulatory volume − Decreased oxygenation − Decreased oxygen-carrying capacity − Systemic hypertension

Arterial distribution − Left anterior descending artery (LAD) – 45%. − Right coronary artery (RCA) – 35%. − Left
circumflex coronary artery (LCA) – 15%. Classified as transmural, subendocardial, or microscopic − Transmural: most
common type - ischemic necrosis involves >50% of myocardial wall. regional vascular occlusion by thrombus. ST
elevated Mis (STEMIs) due to atherosclerosis and acute thrombosis. − Subendocardial infarction - Ischemic necrosis
involves
Location -Left Anterior Descending Artery LAD – 45% , Rt coronary artery RCA 35%, Left circumflex coronary artery LCA –
15%

Classification Types

1) Transmural – most common, ischemic necrosis >50% of myocardial wall, STelevatedMis – due to atherosclerosis
and acute thrombosis
2) Subendocardial Infarction - <50%, hypoperfusion due to shock, ECG changes Not noted
3) Microscopic Infarction – small vessel occlusion – due to → vasculitis, emboli, or spasm

Symptoms – chest pain – crushing, feeing of impending doom, elephant on chest, shortness of breath, JVD, nausea -
Severe “crushing” substernal chest pain − Radiates to the left arm, jaw, and neck. − Atypical presentation of MI - no
chest pain - most frequently in elderly patients, diabetics, women, and postsurgical patients. Typical → Chest heaviness,
tightness, and shortness of breath – Diaphoresis (sweating) - Nausea and vomiting - Jugular venous distension (JVD) -
Anxiety

TX – ECG → ST segment elevation, Q waves (see ~ page 13 for ECG results) – chest x-ray – →echo – for extra heart
sound

Microscopic and Gross changes Gross changes 0–12 h − No visible gross change 12–24 h − Vague pallor and
softening - 1–7 days − Yellow pallor 7–10 days − Central pallor with a red border - 6–8 wks − White, firm scar

1–4 h − None or wavy myocyte fibers at border or contraction band necrosis - 4 h–3 days − Coagulative necrosis
1–3 days − Neutrophilic infiltrate - 3–7 days − Macrophages - 7–10 days − Granulation tissue -
3–8 wks − Remodeled type III collagen becoming dense, collagenous scar

Investigations – ECG – ST segment elevation, Q wave representing myocardial coagulative necrosis in 24-48 hours
Serum Markers Used to Diagnose Myocardial Infarctions

Time Peak Return to normal by

CK-MB 4-8 hours 18 hours 2-3 days

Troponin I & T(CS) 3-6 hours 16 hours 7-10 days

LDH 24 hours 3-6 days 8-4 days

Vague pallor change 12-24 hour

Yellow pallor 1-7 days

Central pallor with red border 7-10 days

White firm scar 6-8 weeks

Microscope changes: 1-4 hours: none or wavy myocyte fibers/band necrosis; → 4 hours-3 days: coagulative necrosis; 1-
3 days: neutrophilic infiltrate; →3-7 days: Macrophages; → 7-10 days: granulation tissue; → 3-8 weeks: Remodeled
type III collagen → dense collagen scar

Complications − Cardiac arrhythmias − Sudden death − MC arrhythmia? − Congestive heart failure − Cardiogenic
shock (>40–50% myocardium is necrotic) − Mural thrombus and thromboembolism − Fibrinous pericarditis – Friction rub
− Dressler syndrome (autoimmune fibrinous pericarditis) − Ventricular aneurysm ( between 1 and 2 months) − Cardiac
rupture. - Ventricular free wall rupture - between days 3 and 7 - Interventricular septum rupture - Papillary
muscle rupture→ Mitral Regurgitation

Ventricular Aneurysm
ECG – ST elevation seen > 2 weeks following an acute myocardial infarction – usually precordial leads, concave or
convex, usually well formed Q / QS waves, T waves small compared to QRS, - unlike hyperacute T-waves of acute STEMI

Complications – Sudden cardiac death with 1 hour of onset of symptoms. Chronic Ischemic heart disease – insidious
onset of progressive CHF, left ventricular dilation

Ischemic Heat Disease→ include: Angina, myocardial infarction, chronic ischemic heart disease
#1 cause of death – Angina - Imbalance of myocardial oxygen supply and demand

Angina: Squeezing or pressure-like sensation in the chest that patients have during myocardial ischemia.
Stable angina (most common type) Caused by coronary artery atherosclerosis with luminal narrowing >75%. -
Chest pain - increased cardiac demand (exertional or emotional), - Relieved by rest or nitroglycerin (vasodilation). -
Electrocardiogram shows ST segment depression (subendocardial ischemia).

Prinzmetal variant angina caused by coronary artery vasospasm - produces episodic chest pain often at rest;
relieved by nitroglycerin - Electrocardiogram shows transient ST segment elevation (transmural ischemia).

Unstable or crescendo angina − caused by formation of a nonocclusive thrombus − characterized by increasing


frequency, intensity, and duration of episodes − episodes typically occur at rest. This form of angina has a high risk for
myocardial infarction.

Chronic ischemic heart disease: − chronic oxygen deprivation, progressive congestive heart failure (CHF) − Due
to replacement of ischemic cardiac myocytes with noncontractile fibrous tissue over time.

Congestive Heart Failure – presence of insufficient cardiac output to meet metabolic demand of the
body’s tissues - Classified into − Left-sided vs Right-sided − Systolic HF vs Diastolic HF

Left Heart Failure - Failure of the left ventricle to eject sufficient blood into the aorta, causing the blood to back up
into the lungs. Causes – ischemic heart disease, systemic hypertension, myocardia diseases and aortic or mitral valve
disease. Left ventricular hypertrophy and dilatation − Clinical Symptoms: − Dyspnea, orthopnea, paroxysmal nocturnal
dyspnea, rales, and S3 gallop. Pulmonary hypertension

Right sided heart failure - Most commonly caused by left-sided heart failure − Pulmonary or tricuspid
valve disease − Cor pulmonale: Lung disease → pulmonary HTN →↑right ventricular pressure → right ventricular
hypertrophy →right-sided heart failure. Signs: JVD, hepatosplenomegaly, dependent edema, ascites, weight gain,
pleural and pericardial effusions. - Right ventricular hypertrophy and dilatation - Chronic passive congestion of the liver
 Cardiac sclerosis/cirrhosis

VALVULAR HEART DISEASE


Degenerative calcific aortic valve stenosis - Common valvular abnormality characterized by age-related
dystrophic calcification, degeneration, and stenosis of the aortic valve. − Common in congenital bicuspid aortic valves. −
Complications: Left ventricular hypertrophy (LVH) and congestive heart failure − Treatment - aortic valve replacement

Mitra Valve Prolapse - Enlarged, floppy mitral valve leaflets − Microscopically show myxomatous degeneration. −
Affects individuals with Marfan syndrome. − Asymptomatic and a mid-systolic click can be heard on auscultation. −
Complications - infectious endocarditis and septic emboli, rupture of chordae tendineae with resulting mitral
insufficiency → sudden death

Rheumatic valvular heart disease/Acute Rheumatic fever − pharyngeal infection with Group A β-
hemolytic streptococci. − production of antibodies that cross-react with cardiac antigens (type II hypersensitivity
reaction) − Affects children (ages 5–15 years) − Symptoms occur 2–3 weeks after a pharyngeal infection − Lab -
elevated antistreptolysin O (ASO) titers.

Diagnosis - requires − 2 major OR 1 major and 2 minor criteria, plus a preceding group A strep infection.

Major Criteria Minor Criteria


Carditis, Polyarthritis, Chorea Fever, Polyarthralgia, Labs: elevated ESR or leukocyte count

Erythema marginatum, Subcutaneous nodules ECG: prolonged P-R interval

Rheumatic Heart Disease – divide into Acute and Chronic


1) Acute rheumatic heart disease − Myocardium, endocardium, and pericardium. − Aschoff body - fibrinoid
necrosis surrounded by macrophages (Anitschkow cells), lymphocytes, and plasma cells. − Endocarditis; left atrial
endocardial thickening (MacCallum plaques). − Fibrinous pericarditis may be present

2) Chronic rheumatic heart disease − Mitral and aortic valvular fibrosis, characterized by valve thickening and
calcification; fusion of the valve commissures; and damaged chordae tendineae. − Complications - mitral stenosis and/or
regurgitation, aortic stenosis and/or regurgitation, congestive heart failure, and infective endocarditis.

Chronic Rhematic Heart Disease - - has had → mitral high pressure valves –

Infectious bacteria endocarditis – 2 types Acute and Subacute - Bacterial infection of the cardiac
valves - Vegetations on the valve leaflets - Risk factors − Rheumatic Heart disease, Mitral valve prolapse , Bicuspid
aortic valve, Degenerative calcific aortic stenosis, Congenital heart disease, Artificial valves, Indwelling catheters, Dental
procedures, Immunosuppression, Intravenous drug use

1 Acute endocarditis − High virulence organism - Staphylococcus aureus, Prognosis is poor, with mortality of 10–
40%. IV drug use is the most common etiology, Tricuspid valve – First valve to be affected

2) Subacute endocarditis − Low virulence organism - Streptococcus group viridans (Alpha-hemolytic, Bile-
resistant, Optochin-resistant) − <10% mortality

Much less common - S. bovis (gallolytica) endocarditis & Septicemia – Rule out Colorectal CA, Staph epidermidis –
prosthetic valves

Clinical Features - Fever, chills, weight loss, and cardiac murmur. Embolic phenomena: Retina - (Roth spots); and
distal extremities (Osler nodes [painful, red subcutaneous nodules on the fingers and toes], Janeway lesions [painless,
red lesions on the palms and soles], and splinter fingernail hemorrhages). Diagnosis − Serial blood cultures.
Complications − septic emboli, − valve damage − congestive heart failure − myocardial abscess, − Dehiscence of an
artificial heart valve - Tricuspid valve – is first affected – also sub acute

Marantic endocarditis (nonbacterial thrombotic endocarditis [NBTE]) − characterized by small,


sterile vegetations along the valve leaflet line of closure in patients with a debilitating disease. − Major complications are
embolism and secondary infection of the vegetations

MYOCARDITIS − inflammation of the heart muscle (myocardium). The inflammation can reduce the heart's
ability to pump blood. Myocarditis can cause chest pain, shortness of breath, and rapid or irregular heart rhythms
(arrhythmias). Infection with a virus is one cause of myocarditis. - Caused by infectious (Coxsackie A and B viruses,
Chagas disease) − Acute heart failure or even dilated cardiomyopathy

CONGENITAL HEART DISEASE


Coarctation of Aorta – Presented by Lorato – Most common cause of childhood heart disease
1) Preductal coarctation (infantile-type) associated with Turner syndrome - severe narrowing of aorta
proximal to the ductus arteriosus. - associated with a patent ductus arteriosus (PDA), - right ventricular hypertrophy
(secondary to the need for the right ventricle to supply the aorta through the patent ductus arteriosus). Infancy with
congestive heart failure – Symptoms -Accompanied by weak pulses and cyanosis in the lower extremities Prognosis
poor without surgical correction

2) Postductal coarctation (adult-type) − stricture or narrowing of the aorta distal to the ductus arteriosus. −
present in a child or an adult − hypertension in the upper extremities, and hypotension and weak pulses in the lower
extremities. − collateral circulation may be supplied via the internal mammary and intercostal arteries; the effects of this
collateral circulation may be visible on chest x-ray with notching of the ribs due to bone remodeling as a consequence of
increased blood flow through the intercostal arteries. − Complications congestive heart failure (the heart is trying too
hard), intracerebral hemorrhage (the blood pressure in the carotid arteries is too high), dissecting aortic aneurysm (the
blood pressure in the aortic route is too high)

Complications – aortic rupture, HF, systemic HTN, Berry aneurysm Presentation – SX SOB with exertion, fatigue, poor
feeding in infants, headache, systolic murmur, HTN in upper extremities, underdeveloped legs, leg cyanosis, delayed
pedal pulse. Investigations- CXR – rib notching, cardiomegaly, Echo – narrow descending artery TX – Prostaglandin E
(neonates), Diuretics (HF), beta-bockers (HTN), Surgica repair (child HTN, adults), balloon angioplasty, stent replacement

CARDIOMYOPATHIES
Cardiomyopathy is a disease of the heart muscle that makes it
harder for the heart to pump blood to the rest of the body.
Dilated, hypertrophic, and restrictive (in order of decreasing
prevalence)

Dilated cardiomyopathy Most common form -


Cardiac enlargement with dilatation of all 4 chambers - Congestive heart failure (typical mode of presentation). Causes
− Genetic in 20–50% of cases, − Alcohol − Medications (Adriamycin [doxorubicin]), − Cocaine, − Viral myocarditis
(Coxsackievirus B and enteroviruses), − Parasitic infections (Chagas disease), − Iron overload − Pregnancy. Physiology
Destruction of myocardial contractility, which affects systolic function. Investigations Echocardiogram typically shows
decreased ejection fraction. Complications include mural thrombi and cardiac arrhythmias Prognosis poor with 5-
year survival of 25%. Treatment heart transplantation. Gross examination Myocyte hypertrophy with interstitial
fibrosis on microscopy, and eccentric hypertrophy

Hypertrophic cardiomyopathy (also called asymmetrical septal hypertrophy, and idiopathic hypertrophic
subaortic stenosis [IHSS]) − common cause of sudden cardiac death in young athletes. Asymmetrical
hypertrophy of cardiac muscle that causes decreased compliance affecting diastolic function. Cause Autosomal
dominant disorder (>50% of cases) Idiopathic. Muscle hypertrophy is due to the increased synthesis of actin and
myosin, and  Microscopic examination: Hypertrophied cardiac muscle fibers and in disarray. Hypertrophic
cardiomyopathy is most prominent in the ventricular septum, where it can obstruct the ventricular outflow tract. This
can potentially lead to death during severe exercise when the cardiac outflow tract collapses, preventing blood from
exiting the heart. Lab Systolic S4 murmur. Etiology – 60-70% are familial, aut dominant. TX stop high intense sports,
beta blockers, avoid vasodilators & diuretics that increase preload

Restrictive cardiomyopathy walls of the


heart are rigid. Heart is restricted from stretching and
filling with blood properly. Least common of the three
Don’t confuse with constrictive pericarditis, a disease
which presents similarly but is very different in treatment
and prognosis. Restriction of cardiac filling during diastole
Etiologies include amyloidosis, sarcoidosis,
endomyocardial fibroelastosis, and Loeffler
endomyocarditis. − Increased deposition of material leads
to decreased compliance Gross examination ventricles
are not enlarged. Microscopy will reflect the underlying
cause. Arrhythmogenic right ventricular cardiomyopathy
− thinning of the right ventricle due to autosomal
dominant mutations that encode desmosomal junctional
proteins. − On microscopy, there is fatty infiltration of the
myocardium.
Carcinoid Heart Disease − Right-sided endocardial and
valvular fibrosis secondary to serotonin exposure carcinoid −
Tumors that have metastasized to the liver. − Plaque-like
thickening (endocardial fibrosis) of the endocardium and valves
of the right side of the heart. − Other Sx: Skin flushing,
diarrhea, cramping, bronchospasm, wheezing, and
telangiectasias. − Elevated urinary 5-hydroxyindoleacetic acid
(5- HIAA)

Carcinoid heart disease - >serotonin, tumor→ liver, plaque like,


Sx – skin flush, cramping, bronchospasm, wheezing

Cardiac Tumors Primary cardiac tumors - rare. − Benign; the malignant tumors are sarcomas. − Treatment is
excision. Cardiac myxoma − benign tumor usually arising within the left atrium near the fossa ovalis − In decades 3-6
of life; can present like mitral valve disease. − In 10% of cases there is an autosomal dominant condition known as
Carney complex (myxomas with endocrine abnormalities and lentigines or pigmented nevi). − Characterized
microscopically by stellate-shaped cells within a myxoid background. − Complications include tumor emboli and “ball-
valve” obstruction of the valves. Cardiac rhabdomyoma is a benign tumor usually arising within the myocardium that
is associated with tuberous sclerosis.
Primary cardiac myxoma Tumor

Tuberous Sclerosis Complex - also called tuberous sclerosis complex, or


TSC) is a rare, multi-system genetic disease that causes non-cancerous (benign)
tumors to grow in the brain and on other vital organs such as the kidneys, heart,
eyes, lungs, and skin.

Autosomal dominant − Mutation in genes TSC1 and TSC2,which encode tumor


suppressor proteins hamartin and tuberin, respectively − Multiple hamartomas −
Cortical tubers − Renal angiomyolipomas − Cardiac rhabdomyomas − Pulmonary
hamartomas

Pericardial Disease
Pericarditis - most common disease of the pericardium − Characterized by inflammation that can by a wide variety
of agents. -Infectious: Coxsackie B virus (most common), tuberculosis, other bacteria. Noninfectious: Uremia
(secondary to renal failure), autoimmune (SLE, Dressler syndrome), nearby. inflammation (post-MI, myocarditis),
metastatic cancer. Diagnosis: History and Physical examination, ECG. Findings: Classically, patients will have chest
pain relieved by sitting forward; on cardiac auscultation, a pericardial friction rub. ECG: Diffuse ST-segment
elevation and PR-segment depression. The ST-segment elevation. Complications Pericardial effusion − Fluid
accumulation in the pericardial sac. − Can lead to cardiac tamponade. − Chronic fluid accumulation in the
pericardium allows for stretching of the pericardium; therefore, larger volumes can build up without causing
tamponade. Hemorrhagic effusion most likely is due to tuberculous or malignant pericarditis Large effusion, electrical
alternans can occur; the heart is swinging in a large effusion and causes the recordings to alternate.
Constrictive pericarditis: − Pericardial layers
adhere to each other and fill with fibrous tissue −
Prevents filling of the heart. Cardiac tamponade -
life-threatening emergency. − Heart compressed by fluid
in the pericardium, preventing filling of the ventricles
and subsequently decreased cardiac output. −
Secondary to an effusion or bleeding caused by trauma
(such as a knife through the heart), ventricular wall
rupture, or extension of an aortic dissection into the
heart − Beck’s triad: hypotension from decreased
cardiac output, jugular venous distention from blood
backing up into the veins because the heart cannot fill,
and muffled heart sounds because the heart is being
strangled with fluid

Pulsula paradoxus - exaggerated fall in a patient's blood pressure during inspiration by greater than 10 mm Hg.

Pulsus paradoxus - present in tamponade; an exaggeration of a normal physiologic response. − During


inspiration, the negative intrathoracic pressure increases venous return to the right side of the heart, filling the right
ventricle with more volume. This larger filling of the right ventricle causes the septum between the right and left
ventricles to bow toward the left ventricle, reducing left ventricular filling and causing a small decrease in cardiac output
and thus blood pressure. In cardiac tamponade, because there is so little room in the heart, this effect is exaggerated
and the blood pressure drops by more than 10 mm Hg during inspiration, called pulsus paradoxus. Treatment of
pericarditis involves treating the underlying cause when possible (such as dialysis in uremic pericarditis) as well as
using antiinflammatory medications to prevent the inflamed pericardium from developing an effusion or scarring. With
tamponade, immediate pericardiocentesis (needle drainage of the fluid) is indicated.

January 18, 2023 – CVS Pathology 3(1) Lecture notes – after class presentation

Vasculitides – FA 314-315 Vasculitis refers to a varied group of disorders which all share a common underlying
problem of inflammation of a blood vessel or blood vessels. The inflammation may affect any size blood vessel,
anywhere in the body. It may affect either arteries and/or veins.
Large Vessel Vasculitides
Takayasu arteritis − Younger patients (age <50) – pulseless, affects Aortic arch, Asian female <40 yr, fever, wt loss,
fatigue, muscle ache. Initial symptoms may be nonspecific (fatigue) with a variable course to more severe symptoms
(blindness) and involvement of the aortic arch. − Microscopically, there is vessel wall thickening and variable
inflammation (from a mononuclear adventitial infiltrate to medial necrosis with granulomas).

Giant cell arteritis − formerly called temporal arteritis, but the temporal arteries are not always involved. −– female
>50 yr, branches of carotid inflamed, headache, blindness, jaw claudication ↑ESR, Tx corticosteroids immediately to
prevent blindness - Vertebral and ophthalmic arteries and aorta − Headache to more severe symptoms (blindness). −
Microscopically, there are inner media granulomas in classic cases. − Treatment is steroids and anti-TNF therapy

Medium Vessel Vasculitides


Kawasaki disease − Mucocutaneous symptoms and cervical lymph node enlargement in children. − Involvement of
the coronary arteries leads to cardiovascular sequelae, which can be circumvented with immunoglobulin therapy. −
Microscopically, there is transmural vascular inflammation.

Thromboangiitis obliterans (Buerger disease) − Affects smokers, causing inflammation and thrombosis of
the arteries supplying the hands and feet, − Leads to claudication, cold sensitivity, ischemic pain, gangrene with
autoamputation of digits. − Treatment is smoking cessation.
Polyarteritis nodosa − systemic necrotizing vasculitis occurring most often in young adults (M > F). − An
association with hepatitis B virus. − The clinical course is one of episodic nonspecific symptoms (low-grade fever). −
Pulmonary involvement is rare; renal artery involvement can be fatal. − Immunosuppressive therapy can achieve
remission in most cases.

Small Vessel Vasculitides


ANCA (antineutrophil cytoplasmic antibody)- associated. − Granulomatosis with polyangiitis, formerly
known as Wegener’s granulomatosis − eosinophilic granulomatosis with polyangiitis, formerly known as
Churg-Strauss syndrome Immune complexes − Anti-glomerular basement membrane disease − IgA vasculitis,
also known as Henoch-Schönlein purpura)

Granulomatosis with polyangiitis − Occurs in middle-aged men; − Characterized by granulomas of the lung and
upper respiratory tract, glomerulonephritis, and a necrotizing granulomatous vasculitis. − PR3-ANCAs are present in
most cases.

Eosinophilic granulomatosis with polyangiitis − Associated with asthma, extravascular granulomas


(respiratory tract), and a systemic vasculitis that features eosinophils; − Eosinophil counts may be extremely high in
peripheral blood. T lymphocytes and antibodies to MPO (P-ANCA) play a role in the etiology. − Increased IgG4 levels.
ANCA is present in cases with glomerulonephritis. − Sequential phases are allergic, followed by eosinophilic and
vasculitic. − Cardiac involvement may be fatal. Steroids are therapeutic. − Microscopic findings depend upon the organ
biopsied: Purpuric leg lesions show a leukocytoclastic vasculitis; the glomerulonephritis tends not to show eosinophilic
infiltrates; the extravascular pulmonary granulomas contain eosinophils.

Stop here – Dr. Okon – Jan 18 – review -

RAYNAUD DISEASE VERSUS PHENOMENON - blood vessels narrow, or vasospasm, when stress is
experienced or the fingers or toes are exposed to cold temperatures. This narrowing of the vessels prevents blood from
reaching the affected area, making the skin appear white and blue.

Primary Raynaud phenomenon (Raynaud disease) − occurs without another illness triggering the
condition. Primary Raynaud’s disease is much more common and tends to be milder than secondary Raynaud’s -
Typically occurs in young women 15-30 years old − Episodic small artery vasospasm in the extremities, nose, or ears; it
results in blanching and cyanosis of the fingers or toes upon stress or (more commonly) exposure to cold. − The
pathogenesis may involve CNS and intravascular factors.

Secondary Raynaud phenomenon - or Raynaud’s syndrome occurs as a result of an illness or another


contributing factor. And is caused by arterial insufficiency secondary to an underlying disease such as scleroderma
(CREST).

CREST Syndrome – Calcinosis, Raynaud phenomenon, esophageal dysmotility, sclerodactyly, Telangiectasia →


Sclerodermas –
VASCULAR NEOPLASMS
Hemangiomas − extremely common, benign vascular tumors. − most common tumor in infants appearing on the
skin, mucous membranes, or internal organs. − The major types are capillary and cavernous hemangiomas. −
Hemangiomas may spontaneously regress.

Hemangioblastomas − Associated with von Hippel-Lindau disease, which may cause multiple hemangioblastomas
involving the cerebellum, brain stem, spinal cord, and retina, as well as renal cell carcinoma. Glomus tumors
(glomangioma) are benign, small, painful tumors of the glomus body that usually occur under fingernails.

Kaposi sarcoma is a malignant tumor of endothelial cells − associated with Kaposisarcoma–associated virus (HHV8).
− multiple red-purple patches, plaques, or nodules that may remain confined to the skin or may disseminate. −
Microscopically, there is a proliferation of spindle-shaped endothelial cells with slit-like vascular spaces and extravasated
erythrocytes. − classic European form, transplant-associated form, African form, AIDS-associated form

Angiosarcoma (hemangiosarcoma) − malignant vascular tumor with a high mortality that most commonly
occurs in skin, breast, liver, and soft tissues. Liver angiosarcomas are associated with vinyl chloride, arsenic, and
thorotrast.
Now move to Pharmacology - September 15, 2022 -….Jan 18, 2023 (2 nd half of class)

Adrenergic receptors – sympathetic – all have beta receptors - CVS Pharmacology


α-1 agonist - vascular smooth muscle, visceral smooth muscle, radial smooth muscle of iris, CNS neurons - increase
mean BP via vasoconstriction. − Increased BP may elicit a reflex bradycardia − Cardiac output may be ↓ but also offset
by ↑ venous return - Drugs and uses: − Phenylephrine: nasal decongestant and ophthalmologic use (mydriasis without
cycloplegia), hypotensive states. → total peripheral resistance → ↑BP

α-2 Agonist - some presynaptic terminals, pancreatic islets, platelets, smooth muscles, CNS neurons - stimulate
prejunctional receptors in the CNS to decrease sympathetic outflow. Their primary use is for mild to moderate HTN.
Drugs and uses: − clonidine and methyldopa (mild to moderate hypertension)

β Agonists − Systemically, beta-agonists ↓ mean BP via vasodilation (β2) and ↑ HR (β1)

Drugs and uses: − Isoproterenol (β1 = β2) − Dobutamine (β1 > β2): congestive heart failure − Selective β2 agonists:
salmeterol, albuterol, terbutaline (asthma); terbutaline (premature labor)

β-1 Agonist - myocardium, JG cells→ release of renin → angiotensin → ↑BP, some presynaptic terminals, CNS neurons
- ↑ HR, ↑ SV, ↑CO, and ↑ pulse pressure

β-2 Agonist - inhibitory → cause relaxation – which means in GI tract, it will reduce the need for px to perform certain
activities. Ex flight response→ cannot urinate, no bowel activity, digestion is stopped, all muscle relaxed, b-2 inhibit,
When exercising and sympathetic activated, β-2 will try to receive enough blood → skin flushed symp of vaso recp. On
liver β-2 - glucose breakdown, heart – in some way reduce the energic consumption, β-2 is energy consumption → acts
on visceral smooth muscle, vascular smooth muscle, liver, myocardium, skeletal muscle, some presynap, CNS neurons -
↓ TPR, ↓ BP

all smooth muscle presynaptic cAMP


α1 α2
sympathetic contraction

vasoconstriction ↑BP

Heart Rate ↑ all smooth muscle

Renin → kidney relax airway – open up


β1 β2
Angiotensin →

BP↑ B Vol ↑ Stimulation Inhibition


Know this will help us to understand side effects. Example – flight → sympathetic system activated → HR goes up, and
alpha 1, vaso constrict GI liver, visceral smooth muscles, b-2 inhibitors and able to get more oxygen – fight/flight
response→decrease the output of a certain organ

Agonist – drug that acts similar to substrate and stimulate receptor

Antagonist – drug that inhibits or slows activity to receptor


α-1 – increases mean blood pressure by vaso constriction. When give a-1 agonist → ↑ BP → ↑ stretch on arterial wall,
sensed by barrow receptors and send a message to nuclei to medulla → sends messages back that sympathetic system
inhibited and para sympathetic is activated.

➔ Act on SA node– reflex bradycardia. α-1 but reflex bradycardia. CO2 may be decreased , but may not decrease
as much, the α-1 agonist, cause vaso constriction the vein as well, Example drugs – phenylephrine – nasal
decongestant – dilate pupil (α-1 ) and no loss of accommodation. Also, hypotensive states can be treated

When you constrict the veins –increase venous return to heart so not CO cardiac output -not reduced

Mixed-Acting Agonists: Norepinephrine vs. Epinephrine

Norepinephrine (α1, α2, β1) − α1: ↑ TPR, ↑ BP − β1: ↑ HR, ↑ SV, ↑ CO, ↑ pulse pressure − Potential reflex
bradycardia - No effect on β2

Epinephrine Dose-dependent effects: − Low-dose: β1, β2 stimulation − High-dose: α1, β1 (β2)

• β2-specific effects: − Smooth muscle relaxation: bronchioles, uterus, blood vessels

Metabolic effects: ↑ glycogenolysis (muscle and liver) ↑ gluconeogenesis ↑ mobilization and use of fat

Differentiation of high-dose epinephrine versus norepinephrine: – Epinephrine reversal: Use of α1 blocker to


reverse hypertension to hypotension in a patient receiving too much epinephrine – Hypertension was due to
predominant α1 tone on the vasculature – Hypotension results from unmasking β2 receptors

Uses of Norepinephrine and Epinephrine − Cardiac arrest − Adjunct to local anesthetic − Hypotension −
Anaphylaxis (epinephrine only) − Asthma (epinephrine only)

Homework –Lorato – on alpha receptor antagonist, Chelsea – beta receptor antagonist, and Miriam – loop diuretics
ADRENERGIC ANTAGONISTS
α Receptor Antagonists Alpha-receptor antagonists decrease TPR and decrease mean BP. • May cause reflex
tachycardia and salt and water retention • Major uses: – Hypertension – Pheochromocytoma (nonselective α blocker)
– Benign prostatic hyperplasia (BPH; selective α1 blocker) • Drugs:  – Nonselective blocker: phentolamine (competitive
inhibitor), phenoxybenzamine (noncompetitive inhibitor) – Selective α1 blocker: prazosin, doxazosin, terazosin,
tamsulosin – Selective α2 blocker: mirtazapine (used as antidepressant)

β Receptor Antagonists β1 blockade: − ↓ HR, ↓ SV, ↓ CO − ↓ renin release  β2 blockade: − May precipitate
bronchospasm (in asthmatics) and vasospasm (in patients with vasospastic disorders) − ↓ aqueous humor production
Metabolic effects − Blocks glycogenolysis, gluconeogenesis − ↑ LDLs, Tgs

Cardioselectivity (β1): − Less effect on vasculature, bronchioles, uterus, and metabolism − Safer in asthma,
diabetes, peripheral vascular diseases

Intrinsic sympathomimetic activity (ISA): − Act as partial agonists − Less bradycardia (β1) − Slight vasodilation or
bronchodilation (β2) − Minimal change in plasma lipids (β2) Pharmacokinetic properties: no CNS entry of atenolol
General uses of beta-blockers: − Angina, hypertension, post-MI (all drugs) − Antiarrhythmics (class II: propranolol,
acebutolol, esmolol) − Glaucoma (timolol) − Migraine, thyrotoxicosis, performance anxiety, essential tremor
(propranolol)

• Combined alpha-1 and beta blocking activity: − Labetalol and carvedilol − Use in CHF (carvedilol) and in hypertensive
emergencies (labetalol)

• K+-channel blockade and β-blocking activity: sotalol

Jan 18, 2023 - Diuretics

Diuretics – remove excess fluid out of the body!


Osmotic Diuretics − Mannitol (IV) inhibits water reabsorption throughout the tubule. – complex sugar and
attracts water − It increases urine volume. Uses: − ↓ IOP in glaucoma − ↓ intracerebral pressure − Oliguric states (e.g.,
rhabdomyolysis) Side effects: acute hypovolemia - NB Contraindicated in CHF and pulmonary edema FA 607
Increase excretion of bicarbonate→ acidosis – and hypokalemia

Carbonic Anhydrase Inhibitors: acetazolamide and dorzolamide • Mechanism: carbonic anhydrase inhibition
→ − ↓ H+ formation inside PCT cell – ↓ Na+/H+ antiport – ↑ Na+ and HCO3 − in lumen – ↑ diuresis Uses:
Glaucoma, Acute mountain sickness, Metabolic alkalosis Side effects: − Bicarbonaturia and acidosis, Hypokalemia,
Hyperchloremia, Paresthesias, Renal stones, Sulfonamide hypersensitivity

Loop Diuretics: furosemide, torsemide, and ethacrynic acid Mechanism: Na+/K+/2Cl− transporter inhibition, →
↓ intracellular K+ in TAL − ↓ back diffusion of K+ − ↓ positive potential − ↓ reabsorption of Ca2+ and Mg2+ − ↑
diuresis Uses: − Acute pulmonary edema − Heart failure − Hypertension − Refractory edemas − Anion overdose −
Hypercalcemic states

Loop diuretics → manage acute heart failure, hypertension


Side effects: − Sulfonamide hypersensitivity (furosemide) − Hypokalemia and alkalosis − Hypocalcemia −
Hypomagnesemia − Hyperuricemia (actively secreted by the OAT) − Ototoxicity (ethacrynic acid > furosemide)  Drug
interactions − Aminoglycosides (enhanced ototoxicity) − Lithium (chronic loop administration, ↓ clearance)

Digoxin (↑ toxicity due to electrolyte disturbances) must use cautiously

Ototoxicity – sulphur compounds can result in allergic rx in some

Aminoglycosides – should not be combined with loop due to enhanced ototoxicity

Lithium not combined with loop – over long time, reduce lithium clearance and → toxic for lithium as Li competes
with K binding sites and the more a px excretes K out, then more Lit binding sites→ decrease clearance of Lithium, same
with Digoxin and or Digitalis. Digoxin – competes with potassium K+ toxicity – and decrease in K levels and also
fights for binding sites

Thiazide Diuretics - Uses of Thiazide – fluid overload – in CHF – FA 609


Thiazide diuretics are a type of diuretic (a drug that increases urine flow). They act directly on the kidneys and promote
diuresis (urine flow) by inhibiting the sodium/chloride cotransporter located in the distal convoluted tubule of a nephron
(the functional unit of a kidney)

Hydrochlorothiazide, chlorthalidone, and indapamide Mechanism: Na+/Cl− transporter inhibition, results in: − ↑
luminal Na+ and Cl− in DCT − ↑ diuresis Uses: − Hypertension, CHF − Nephrolithiasis (calcium stones) − Nephrogenic
diabetes insipidus Side effects: − Sulfonamide hypersensitivity − Hypokalemia and alkalosis − – Hypercalcemia − –
Hyperuricemia (actively secreted by the OAT) − – Hyperglycemia − – Hyperlipidemia (except indapamide)

Drug interactions and cautions: − – Digoxin (↑ toxicity due to electrolyte disturbances) − – Avoid in patients
with diabetes mellitus

Potassium Sparing Diuretics – FA 609

Drugs: Spironolactone: aldosterone-receptor antagonist Uses: − Hyperaldosteronic state − Adjunct to K+-wasting


diuretics − Antiandrogenic uses (female hirsutism) − Congestive heart failure Side effects: hyperkalemia and acidosis;
antiandrogen

Drugs Amiloride and triamterene: Na+-channel blockers − Uses: adjunct to K+-wasting diuretics, lithium-induced
nephrogenic diabetes insipidus (amiloride) − Side effects: hyperkalemia and acidosis

➔ CHF and for females with hirsutism – hair growth

Salient points, like drug – Amiloride – Na channel also used to tx px with Lithium induced nephrogenic diabetes
insipidus – bipolar px on Lithium are at risk – Amiloride is DOC, AE – hyperkalemia and acidosis

Anti-Hypertensives
Anti- hypertension – put together all your knowledge - we know that Pousiell, R – viscocity and length are inverse to
radius and if we increase radius, lower resistance and lower the pressure. Also know the site of great in arterioles and
they control BP and TPR – is bound to effect BP – so to control BP , reduce TPR – How? Vaso dilation – increase radius,
second is to what generates BP →systole – control P that heart generates – reduce load, reduce the CO, and body fluid
volume, or preload.

Strategies to control blood pressure


− ↓ TPR (total peripheral resistance) − ↓ CO − ↓ body fluid volume ↓ BP may result in homeostatic regulation: −
Reflex tachycardia (↑ sympathetic activity) − Edema (↑ renin activity)

IF ↓ BP – reflex tachycardia → increase sympathetic activity which triggers rennin release and result in edema.

Hypertension control – FA 316 – ex 160/80 know that just diastole increase is from heart not blood vessels, so not to
give drug to decrease TPR as only reduce P more. Want drug that act on heart only.

FIRST-LINE ANTIHYPERTENSIVES
Thiazide diuretics - Angiotensin-Converting Enzyme Inhibitors (ACEIs) and
AngiotensinReceptor Blockers (ARBs) - ACEIs: captopril, lisinopril (and other “−prils”) − Block formation of
angiotensin II − Resulting in prevention of AT1-receptor stimulation − ↓ aldosterone, vasodilation − ACEIs prevent
bradykinin degradationARBs: losartan (and other “−sartans”) − Block AT1 receptors − Same results as ACEIs on BP
mechanisms − ARBs do not interfere with bradykinin degradation - Renin inhibitor: Aliskiren − Blocks formation of
angiotensin I − Same results as ACEIs on BP mechanisms − Aliskiren does not interfere with bradykinin degradation

Must consider race of px →African absorb more Na+, and so thiazide seems more beneficial, for whites, start with ACE
or angiotensin blocker. But not both, ACE I – dry cough , and aldosterone

How to tackle pharma mcqs, easier than physiology.

AB - --statins, no AE cough, better and renin inhibitor –

Uses: − Mild-to-moderate hypertension (all) − Protective of diabetic nephropathy (ACEI/ARBs) − CHF (ACEI/ARBs)
Side effects: − Dry cough (ACEIs) − Hyperkalemia − Acute renal failure in renal artery stenosis − Angioedema
Contraindication: pregnancy

Note for diabetes begin on ACE , then ARB

Calcium-channel blockers (CCBs) block L-type Ca2+ channels in heart and blood vessels. − Results in ↓
intracellular Ca2+ − Causes ↓ CO (verapamil and diltiazem), ↓ TPR (all CCBs) Drugs: verapamil, diltiazem,
dihydropyridines (−“dipines,” prototype: nifedipine) • Uses: − Hypertension (all drugs) − Angina (all drugs) −
Antiarrhythmics (verapamil, diltiazem) • Side effects: − Reflex tachycardia (“−dipines”) − Gingival hyperplasia
(“−dipines”) − Constipation (verapamil) - FA – 318

L side on the heart – when give Ca promotes contraction of the heart, which increase Stroke volume and CO. if block this
→ decrease CO on heart. In case of heart vessels, so cause inhibit constriction, and decrease resistance

Vasodilators may have specificity.

− Arteriolar: Ca2+-channel blockers, hydralazine, K+- channel openers − Venular: nitrates


Both arteriolar and venular: “the rest”

Orthostatic (postural) hypotension results from venular dilation (not arteriolar) and mainly results from α1 blockade or
decreased sympathetic tone. Act on vessels mainly. On veins it will relax and result in pooling of blood in veins

Beta blockers – can cause ED – px in St. Vincent

Have to be cautious with beta blockers, and diabetic – hypoglycemic and no tremor as the beta blocker inhibit

DRUGS ALTERING SYMPATHETIC ACTIVITY


Beta Blockers Side effects: − Cardiovascular depression − Fatigue − Sexual dysfunction − ↑ LDLs and TGs

Cautions in use: − Asthma − Vasospastic disorders − Diabetics (alteration of glycemia and masking of tachycardia due
to hypoglycemic events)

Alpha-1 Blockers − Decrease arteriolar and venous resistance − Reflex tachycardia Drugs: prazosin, doxazosin,
terazosin Uses: − Hypertension − BPH: ↓ urinary frequency and nocturia by ↓ the tone of urinary sphincters Side
effects: “first-dose” syncope; orthostatic hypotension; urinary incontinence − Advantage: good effect on lipid profile (↑
HDL, ↓ LDL)

Alpha-2 Agonists Drugs: clonidine and methyldopa (prodrug) − α2 stimulation: ↓ in sympathetic outflow; ↓
TPR but also ↓ HR Uses: − Mild-to-moderate hypertension (both) − Opiate withdrawal (clonidine) − Hypertensive
management in pregnancy (methyldopa) Side effects: positive Coombs test (methyldopa); CNS depression (both);
edema (both) Drug interactions: tricyclic antidepressants ↓ antihypertensive effects of α2 agonists

Alpha 2 - Avoid – to px with on tri anti-depressants decrease the α-2 agonist

DIRECT-ACTING VASODILATORS Drugs Acting Through Nitric Oxide

Hydralazine − ↓ TPR via arteriolar dilation Use: moderate-to-severe hypertension Side effects: SLE-like syndrome
and slow acetylators; edema; reflex tachycardia

Nitroprusside − ↓ TPR via dilation of both arterioles and venules Use: hypertensive emergencies (used IV) Side
effects: cyanide toxicity (co-administered with nitrites and thiosulfate)
Next – still on Jan 19, 2023 – next slides - September 16, 2022

Anti-Hypertensives Cont'd
Sodium nitrite or amyl nitrite can be used in cyanide poisoning. • Promotes formation of methemoglobin (MetHb),
which binds CN− ions, forming cyanomethemoglobin • Prevents the inhibitory action of CN− on complex IV of electron
transport chain • Cyanomethemoglobin is then reconverted to methemoglobin by treatment with sodium thiosulfate,
forming the less toxic thiocyanate ion (SCN−) • MetHb is converted to oxyhemoglobin with methylene blue

See medication in black tape IV – doesn’t like light – breaks it down** - one→ cyanide -
present as – nitrates Hb → methemoglobin
What med for px that has diabetic→ ACE inhibitors – Hydralazine, labetalol, methyldopa, nifedipine FA 316

Dilators – veno – include nitrates and act both on arteries and veins –

- See – IV has black tape – to prevent light – breakdown → cyanide can effect electron transport chain.

Na nitrate – promotes hb and prevents inhibition of cyanide. Cyanmethemoglobin is reconverted to by giving sodium
thiosulfate to less toxic and easily excrete SCN – Sodium thiosulfate – give methylene blue

Chronic hypertension in pregnancy FA 316 → Methyl DOPA – During, because, before If during → tx
hydralzaine, but before → methal DOPA – differentiate these. Preeclampsia (new-onset hypertensionin
pregnancy) – labetalol or hydralazine.

Drugs Acting to Open Potassium Channels Drugs: minoxidil and diazoxide − Open K+ channel, causing
hyperpolarization of smooth muscle − Results in arteriolar vasodilation Uses: − Insulinoma (diazoxide) − Severe
hypertension (minoxidil) − Baldness (topical minoxidil) Side effects: − Hypertrichosis (minoxidil) − Hyperglycemia (↓
insulin release [diazoxide]) − Edema − Reflex tachycardia

Drugs that open Ca channel, K helps to help maintain membrane potential. F of K – Action potential can’t occur. Open p
Ca and – diascide –

Indication Suitable Drug(s) Angina →Beta blockers, CCBs - Diabetes →ACEIs, ARBs - Heart failure →ACEIs,
ARBs, beta blockers Post-MI → Beta blockers - BPH → Alpha blockers - Dyslipidemias → Alpha blockers, CCBs,
ACEIs/ARBs - Chronic kidney disease → ACEI, ARBs

Hypertensive emergency → nitroprusside, labetalol, or D1 agonist fenoldopam is given intravenously as therapy.FA318

Pulmonary hypertension – affects arteries in the lungs and right side of the heart – shortness of breath,
dizzy, cyanosis lips and skin, palpitations, DRUG - Bosentan - vaso constrictor, PG12, Epoprostenol (Flolan),high does
Ca channel blockers, relax muscle in walls of blood vessels.

Bosentan − Endothelin (ET)-1 is a powerful vasoconstrictor through ET-A and -B receptors. Bosentan is an ET-A
receptor antagonist. − Administered orally − Side effects: those associated with vasodilation (headache, flushing,
hypotension) − Contraindication: pregnancy

Prostacyclin (PGI2): Epoprostenol − Administered via infusion pumps


Sildenafil − Inhibits type V PDE − Increases cGMP − Pulmonary artery relaxation − Decreases pulmonary hypertension

How does Heart Failure Occur? Same as saying decrease in Cardiac Output – what happens – BP drops, and results
in px having response from barrow receptors→ sympathetic increase activity → vaso constriction → TPR and →
afterload increase, and increase in HR → Failing heart
The Failing Heart - Regulation of Blood Pressure - Renin-angiotensin-aldosterone: − JG cells to secrete the enzyme
renin into the bloodstream. − The bloodstream always has angiotensinogen in it, and renin cleaves this angiotensinogen
into angiotensin I. − Angiotensin I gets cleaved into angiotensin II by angiotensin-converting enzyme (ACE). −
Angiotensin II is a potent vasoconstrictor, increasing systemic vascular resistance (SVR) and therefore blood pressure,
and also mediates aldosterone release. give meds to reduce stress on heart – decrease sympathetic, afterload, ace
inhibitors, and also drugs that push out volume, Decrease preload – diuretic, ace inhibitors, nitrates, and arb’s,

↓ preload 1) Diuretics – Loop or thiazide diuretics - Increase the urine flow and consequently increase sodium and
water excretion 2) ACEIs or ARBs – Vasodilators 3) Venodilators – Hydrazaline + isosorbide dinitrate ( preferred for
chronic therapy in patients who cannot tolerate ACEI or ARB) - ARB – Angiotensin Receptor Blockers, ARBs are indirect
vasodilators, not direct vasodilators. ACEI – Angiotensin-converting enzyme inhibitors

↓ afterload 1)ACEIs (ARBs as alternative) - bock the renin angiotensin system, decrease the constriction of
blood vessels in the periphery, drop the blood pressure, increase Na and water excretion. They decrease the work of the
left ventricle and get rid of some of the excess salt and water. 2) Arteriodilators

↑ contractility 1) Digoxin 2) Beta agonists 3) PDE III Inhibitors - inhibits the action of the phosphodiesterase
enzyme PDE3. They are used for the therapy of acute heart failure and cardiogenic shock. PDE-3 inhibitors: cilostazol,
dipyridamole, milrinone, and amrinone

↓ remodeling of the cardiac muscle


1) ACEIs (ARBs alternative)

2) Spironolactone - is used in advanced CHF

3) Beta Blockers - (metoprolol, bisoprolol, carvedilol) – also Provide antiarrhythmic effect – Beta bockers Block
effect of epinephrine/adrenaline and decrease the heart rate. Sometimes the ventricle recovers when you get
the heart rate down from the 90’s into the 60 and 70’s
PRIMARY TREATMENTS FOR CHF • ACEI (ARB as an alternative) • Beta blockers (metoprolol,
bisoprolol, carvedilol) − Provide antiarrhythmic effect and also ↓ remodeling • Diuretics − Loop or thiazide diuretics
to decrease preload − Spironolactone or eplerenone to block aldosterone receptors and ↓remodeling (used in
advanced CHF) • Hydralazine + isosorbide dinitrate − Preferred for chronic therapy in patients who cannot tolerate
an ACEI or ARB - Hydralizine – for black px

Used cautious with renal falure since ecreted tru kidney, also saw that not be combined with loop diruetiec, it competes
with K for same binding sites. -> arrythmia, delay in AV node, avoided with anything that affects K levels, competes with
binding to

For CHF – Ca – contactlilty, tachycardia, EXCEPT WPS – condition where accesory pathway – see below
Milrinone – used in CHF – to decrease BP, Diastolic heart failure – use beta blockers and diuretics

Digoxin – Does not improve survival - Inhibition of cardiac Na+-K+ ATPase. ↑ intracellular Na+ ↓ Na+/Ca2+
exchange ↑ intracellular Ca2+ ↑ Ca2+ release from sarcoplasmic reticulum ↑ actin-myosin interaction ↑
contractile force

Indirect effect: inhibition of neuronal Na+-K+ ATPase (results in ↑vagal activity)

Long t1/2 – requires loading dose

Side effects – A/E – early signs - anorexia, nausea, ECG changes - later signs – disorientation, yeow-green color
disturbances, blurry vision - toxic dose can cause cardiac arrhythmias - interacts with diuretics → ↓ K+ ↓ Mg2+
↑ Ca2+

Caution must be used if there is any renal impairment and cannot be used in Wolff-Parkinson-White Syndrome

Wolff-Parkinson-White Syndrome - Also known as Ventricular preexcitation syndrome, (WPW) - caused by an


accessory pathway (known as the bundle of Kent) between the atria and ventricles. This extra pathway causes the
ventricles to begin depolarizing before they normally would, leading to a premature upsloping in the QRS complex
known as a delta wave.

Re-entry circuit in which an electrical impulse can travel in a circuitous path in one of two directions: − (1) orthodromic,
meaning that the signal travels antegrade down the AV node (as normal), but then retrograde through the bundle of
Kent − (2) antidromic, meaning the signal travels antegrade through the bundle of Kent and then retrograde through the
AV node. Orthodromic tachycardias; normal-appearing QRS complexes (because electrical signal travels normally down
bundles of His), antidromic tachycardias are characterized by widened QRS complexes.

Unusual delay going thru bundle of Kent → wide QRS, Atrial fibrillation – also short PR interval – wide QRS – and delta
wave
Antiarrhythmic Drugs – Drugs Affecting the Cardiac Action Potential

Control the rhythm – NOT the rate -conducted a retrospective population‐based cohort study including 22 635 patients
with AF newly treated with rhythm control (antiarrhythmic drugs or ablation) Compared with rate control, rhythm
control initiated within 1 year of AF diagnosis decreased the risk of stroke -
https://www.ahajournals.org/doi/10.1161/JAHA.121.023055

Class 1 – Na+ channel blocker Na channel blocker s- in 3 sub classes - 1a moderate – Quinidine, Procainamide, 1b weak –
Lidocaine, Phenytoin 1c strong – Flecainide, Propafenone→ phase 1,

Class 2 – beta blockers – inhibit Metoprolol – phase 4

Class 3 – K+ channel blocker – Amiodarone, Sotalol - phase 3

Class 4 – Ca channel blockers – phase 2

→Exam give diagram of action potential and ask what type of what type of antiarythmic has been given
Phase 0 - ? − Rate of depolarization depends on number of Na+ channels open, which in turn depends on resting
membrane potential of the cell. − Class I antiarrhythmic drugs can slow or block phase 0 in fast-response fibers.

Phase 1 - ? − Antiarrhythmic drugs have no significant effects on these transient currents.

Phase 2 − Antiarrhythmic drugs have no significant effects on these currents during this phase of the action potential
(AP).

Phase 3 − Repolarization phase in which the delayed rectifier K+ current rapidly increases as the Ca2+ current dies out
because of time-dependent channel inactivation. − Class III antiarrhythmic drugs slow this repolarization phase. − Note
that during phases 0 through 3 a slow Na+ current (“window current”) occurs, which can help prolong the duration of
the action potential.

Phase 4 − Return of membrane to resting potential—maintained by activity of the Na+/K+-ATPase.


The major distinctive feature of slow fibers is their spontaneous depolarization, shown by the rising slope of phase 4 of
the AP, referred to as the pacemaker potential or “pacemaker current.” Although not completely understood,
pacemaker potential is a composite of inward Na+ (If) and Ca2+ (ICa-T) currents and outward K+ currents (IK). • Class II
and IV antiarrhythmic drugs can slow phase 4 in pacemaker fibers

Now to go over these blockers and how they affect the Cardiac Action Potential

CLASS I: Na+ CHANNEL BLOCKERS Class 1A − Antiarrhythmic: block fast Na+ channels (↓ INa) −
Preferentially in the open or activated state—“state-dependent” blockade − Also blocks K+channel (prolongs
repolarization), ↑ action potential duration and effective refractory period

Quinidine − Muscarinic receptor blockade, - ↑ HR and AV conduction. − May also cause vasodilation via alpha block
with possible reflex tachycardia. − Orally effective, wide clinical use in many arrhythmias; in atrial fibrillation, need initial
digitalization to slow AV conduction. Adverse effects: cinchonism (GI, tinnitus, ocular dysfunction, CNS excitation),
hypotension, prolongation of QRS and ↑ QT interval associated with syncope (torsade). Drug interactions:
hyperkalemia enhances effects and vice versa; displaces digoxin from tissue binding sites, enhancing toxicity.

Usually give digitalis (not dixogin) before giving Quinidine – it blocks effect on SA node – Cinchonism – GI etc –

Procainamide − Less muscarinic receptor block − Metabolized via N-acetyltransferase (genotypic variation) to N-
acetyl procainamide (NAPA), an active metabolite Adverse effects: systemic lupus erythematosus (SLE)–like syndrome
(30% incidence) more likely with slow acetylators; hematotoxicity (thrombocytopenia, agranulocytosis); CV effects
(torsade)- Torsade de Pointes Torsade is defined as the combination of polymorphic ventricular tachycardia plus a
prolonged QT-interval.

NB - jot down SLE like symptoms *** this is one – 30% chance of this – more common in slow aceytlators
CLASS I: Na+ CHANNEL BLOCKERS Class 1B - Antiarrhythmic: block fast Na+ channels (↓ INa) - Block
inactivated channels—preference for tissues partly depolarized (slow conduction in hypoxic and ischemic tissues).
Increased threshold for excitation and less excitability of hypoxic heart muscle. → ↓ APD—due to block of the slow Na+
“window” currents, but this increases diastole and extends the time for recovery.

Lidocaine − Post-MI, open-heart surgery, digoxin toxicity– ventricular arrhythmias only  Side effects: CNS toxicity
(seizures); least cardiotoxic of conventional anti-arrhythmics − IV use because of first-pass metabolism

Mexiletine − Same uses as lidocaine − Oral formulations NB this - has oral form – lidocaine only IV

CLASS I: Na+ CHANNEL BLOCKERS Class 1C− Block fast Na+ channels (↓ INa), especially His-Purkinje tissue −
No effect on APD − No ANS effects Drug: − Flecainide − Limited use because of proarrhythmogenic effects, leading to
↑ in sudden death post-MI and when used prophylactically in VT - rarely used

Class II – beta blockers - Prevent β-receptor activation, which would normally ↑ cAMP  ↓ SA and AV nodal
activity  ↓ Slope of phase 4 (diastolic currents) of AP in pacemakers  Drugs: − Propranolol (nonselective) and the
cardioselective drugs: acebutolol and esmolol  Uses:  Prophylaxis post-MI and in supraventricular
tachyarrhythmias(SVTs)  Esmolol (IV) is used in acute SVTs

CLASS III: K+ CHANNEL BLOCKERS − Decrease IK (delayed rectifier current) slowing phase 3
(repolarization) of AP − Increase APD and ERP, especially in Purkinje and ventricular fibers

Amiodarone − Mimics classes I, II, III, and IV − Increase APD and ERP in all cardiac tissues Uses: any arrhythmia -
t1/2 >80 days − Binds extensively to tissues (large Vd and multiple effects) Side effects: pulmonary fibrosis, blue
pigmentation of the skin (“smurf skin”), phototoxicity, corneal deposits, hepatic necrosis, thyroid dysfunction

Sotalol − Decreases IK, slowing phase III − Non-selective beta blocker: β1 blockade, leading to ↓ HR, ↓ AV conduction
Use: life-threatening ventricular arrhythmia Side effects: Torsade - Torsade de Pointes Torsade is defined as the
combination of polymorphic ventricular tachycardia plus a prolonged QT-interval.

Class IV – Ca 2+channel blockers - Block slow cardiac Ca2+ channels − ↓ phase0, ↓ phase 4 , ↓ SA, ↓ AV
nodal activity

Verapamil and Diltiazem − Prototype Ca2+-channel blockers − Uses: supraventricular tachycardias Side effects:
constipation (verapamil), dizziness, flushing, hypotension, AV block Drug interaction: − Additive AV block with β-
blockers, digoxin − Verapamil displaces digoxin from tissue-binding sites. Don’t combine with beta blockers

Adenosine • Activates adenosine receptors: causes Gi -coupled decrease in cAMP • Decreases SA and AV nodal
activity • Uses: DOC for paroxysmal supraventricular tachycardias and AV nodal arrhythmias • Administered IV: t1/2
Side effects: flushing, sedation, dyspnea • Adenosine is antagonized by methylxanthines (theophylline and caffeine)

Magnesium • Use: torsades - magnesium is 1st line therapy Patients with torsade should receive magnesium, even
if they have a normal magnesium level
Angina
Antianginal Drugs Drug strategies in stable and vasospastic angina involve:
−↓ oxygen requirement by ↓ TPR, CO, or both (nitrates, CCBs, and beta blockers).
−↑ oxygen delivery by ↓ vasospasm (nitrates and CCBs).
what cause O2 to heart while reduce O2 requirement of heart → reduce workload of heart, when comes to spasms.

Why chest pain → angina

Nitrates - Nitrates are Prodrugs of nitric oxide. − Venodilation →↓ preload →↓ cardiac work →↓ oxygen
requirement − Nitrates ↓ infarct size and post-MI mortality

• Drugs: − Nitroglycerin: sublingual, transdermal, and IV formulations − Isosorbide: oral, extended release for
chronic use Side effects: − Flushing, headache, orthostatic hypotension − Reflex tachycardia and fluid retention
Cautions and contraindications: − Tachyphylaxis with repeated use − Cardiovascular toxicity with sildenafil
(VIAGA) NB - need to give break, never combine with sildenafil (Viagra) as both cause
dilaton→ hypotension
Beta Blockers and Carvedilol − Used in angina of effort − β-blockers are contraindicated in vasospastic
angina − Carvedilol is clinically equivalent to isosorbide in angina of effort

Calcium Channel Blockers (CCBs) − All CCBs can be used. − Nifedipine is important for vasospastic angina.
Why contradiction beta blockers – block the beta receptors and leave alpha and → alpha vaso constriction and stable
angina – ca channel blockers

Ranolazine - Ischemia causes increased sodium which prevents calcium exit through Na+/Ca++
exchanger pump. Ranolazine blocks late inward Na+ current in cardiac myocytes, thereby decreasing calcium
accumulation
Results in decreased end diastolic pressure and improvement of diastolic coronary flow
Side effects include constipation and nausea; increased QT makes the drug contraindicated in patients with
long QT syndrome or taking drugs which increase QT

Drugs that decrease mortality in patients with stable angina include aspirin, nitroglycerin,
and beta blockers.
Nitroglycerin is the preferred drug for acute management of both stable
and vasospastic angina.

Hyperipidemia – High Cholesterol levels


HMG-CoA Reductase Inhibitors: Atorvastatin, rosuvastatin, and other “–statins”
– At their highest therapeutic doses, atorvastatin and rosuvastatin are considered “high-intensity” statins and
can lower LDL-C by ≥ 50%
– Lower doses of statins are classified as “low” or “moderate” intensity
Mechanisms: – HMG-CoA reductase inhibition, results in:
Decreased liver cholesterol, Increased LDL-receptor expression– reabsorb cholesterol in circulation,
Decreased plasma LDL
Decreased VLDL synthesis results in: ↓ triglyceridemia
Side effects:
– Myalgia, myopathy (check creatine kinase) – Rhabdomyolysis– Hepatotoxicity (check liver function tests)
Drug interaction:
• – Gemfibrozil (↑ rhabdomyolysis) – increase muscle break down
• – Cytochrome P450 inhibitors enhance toxicity of statins in liver

Bile Acid Sequestrants: cholestyramine and colestipol


Mechanism: complexation of bile salts in the gut, results in:
– Decrease enterohepatic recirculation of bile salts– Increase synthesis of new bile salts by the liver–
Decrease liver cholesterol– Increase LDL-receptor expression – Decrease blood LDL
Side effects: – Increase VLDL and triglycerides - – Gastrointestinal disturbances– Malabsorption of lipid-
soluble vitamins - – Hyperglycemia
Drug interactions with orally administered drugs (warfarin, thiazides, digoxin, etc.)
Contraindication: hypertriglyceridemia

Nicotinic Acid (Niacin, Vitamin B3): inhibition of VLDL synthesis, results in:
– Decreased plasma VLDL – Decreased plasma LDL – Increased plasma HDL
Side effects: – Flushing, pruritus, burning pain (use aspirin) – Hepatotoxicity – Hyperglycemia
Gemfibrozil, Fenofibrate (Fibrates): Mechanism: bind to the PPARα and increase
expression of lipoprotein lipases, results in: Decreased VLDL and IDL – Modest ↓ LDL (however, in some
patients with combined hyperlipidemias, ↑ LDL) – Increased HDL (most patients)
Used in hypertriglyceridemia, Side effects: gallstones, myositis

Ezetimibe Mechanism: prevents intestinal absorption of cholesterol, results in ↓ LDL


Side effect: gastrointestinal distress

Orlistat Therapeutic use: weight loss Mechanism: inhibits pancreatic lipase → ↓ triglyceride
breakdown in the intestineSide effects: oily stools (steatorrhea), diarrhea; ↓ absorption of lipid soluble
vitamins

Monday – Respiratory – and we will complete GI next week – Jan 20 – end of class

Monday September 19, 2022 – January 23, 2023

Respiratory Physiology
Endodermal layer→ by week 4 lower resp system of trachea, bronchi and lungs begin – diverticulum from
ventral wall of the foregut, The muscles, cartilage & CT come from mesoderm.
Respiratory diverticulum enlarge to form lung bud → bifurcate into 2 bronchial buds by month 6
(4 weeks the heart start beating)

A – outpouches, C complete separation .

25th and 28th week – critical (surfactant production begins)– this stage, separation of tracheoesophageal
septum – a lot of aveoli form – Type I and Type II – and before 25 weeks (age of viability- in St Vincent 26
weeks)– survival low. Age of viability (less in USA, around 21 weeks)
Example small balloon with small amt of water – water will take up space – there is some fluid in alveoli – the
surfactant reduces the amount of fluid to allow alveoli t expand. Without surfactant – to much fluid. 35 weeks –
amount of amniotic fluid is less, and surfactant is at mature levels. As soon as takes 1 st breath and cough, and
surfactant participates in expansion of airway.
Esophagus and tracheas share common source.
Task Homework – to find write a project on fistula – define and type that and how each one is managed – and
treatment for each of them
Polyhydramnios - In utero – can tell with ultra sound →increase in amt of amniotic fluid compared to age of fetus –
multiple birth or incorrect age of fetus may also be a cause of this. When infant crying – abdominal distension – infant
ingesting air and cause distension (esp when crying) – blind pouch and when swallow, fluid does not go to stomach
– they will be regurgitation of milk, and lead to gagging
Reflux of gastric – cause pneumonia → a case aspiration when breast feeding
Distal and upper end in blind pouch

Tracheoesophageal fistula (an abnormal communication between the trachea and esophagus) caused by a
malformation of the tracheoesophageal septum. Esophageal atresia and polyhydramnios (increased volume of amniotic
fluid). Symptoms Regurgitation of milk, Gagging and cyanosis after feeding, Abdominal distention after crying, Reflux
of gastric contents into lungs causing pneumonitis

The fistula is most commonly (90% of cases) located between the esophagus and distal third of the trachea.

Congenital Pulmonary Disorders


Pulmonary hypoplasia occurs when lung development is stunted. This condition has 2 congenital causes:

(1) congenital diaphragmatic hernia (a herniation of abdominal contents into the thorax, which affects the development
of the left lung), especially – seen on x-ray – intestine content in the chest wall – that points to this – usually left side and
abdominal contents go into lung area. or

(2) bilateral renal agenesis kidneys fail to form → this causes oligohydramnios, which increases the pressure on the
fetal thorax and Potter’s sequence). One of the features of Potter’s sequence is bilateral pulmonary hypoplasia. -–
amniotic fluid, but not urine – failure results to no fluid in sac, oligohydramnios → Potters sequence – FA 587 – bilateral
pulmonary hypoplasia
Pulmonary Anatomy
Thoracic cavity – 2 lateral lung compartments –
mediastinum is central compartment, 11 intercostal
spaces.

Passage of instruments thru intercostal space is done in


the lower part of the space to avoid neurovascular
structures

3 false ribs 8-9 10 – 1-7 are connected to sternum, 8-9


attach to cartilage of 7 and 11 and 12 floating

Each rib has blood supply – intercostal bundles – over


lower border of the rib -

Lungs – right 3 lobes, left 2 lobes, visceral layer closest, Parietal – somatic nerve, the visceral, no pain, the pain if any
stretch. In lungs

2 zones – conducting – dead space – anatomic (nose, throat) and respiratory zone
Trachea → angle of Louis at T2 - - in restaurant px starts to cough, and difficult to breath – Heimlich maneuver to
dislodge. As clinician – where lodge in rt bronchus – posterior basal segment of rt inf lobe

→ trachea bifurcates into the left and right main bronchi at the carina.

Aspiration of a foreign body will more often enter right primary bronchus -it is shorter, wider and more vertical

Histology – lungs receive entire CO, modify blood, metabolic transformation of lipoproteins and prostaglandins. Also
the enzyme – ACE, which converts angiotensin I to II, is produced in lungs endothelial cells

Mucosa in Conducting zone is ciliated pseudostratified columnar cells and cilia sweep mucus/debris out
Bronchioles level – transition to simple cuboidal epithelium
Aveoli – simple squamous epithelium to facilitate gas exchange
Smoker - metaplasia – it is from epithelia – the cancer in cigarettes

Immotile cilia -→ 2 disorders Cystic fibrosis and Kartagener - The USMLE is obsessed with the contrast
between Kartagener syndrome and cystic fibrosis in regard to infertility A disorder characterized by immotile,
dysfunctional cilia, resulting in sperm immotility and defective fallopian tube-mediated ovum transport. This leads to
male and female infertility, as well as increased risk of ectopic pregnancy.

Cystic Fibrosis – CFTR deletion of phenylalanine in 508 – Chloride deficiency and mucous is thick and difficult to
move → frequent infections of respiratory. Secreted mucous is thick/viscous. Frequent respiratory infections

Kartagener Syndrome – lacking dynein – immotile cilia and cilia can’t move- males’ immotile sperm - infertility
Goblet Cells – abundant in conducting zone epithelium → produce mucus to trap debris and help moisten air.
Decrease in number distally

Kulchitsky cells – Enterochromaffin Cells – neuroendocrine cells. Stain +ve for chromogranin A, cells of origin
for Small Cell Carcinoma of the lungs

Clara cells – non-ciliated cells – secrete →


- Type I pneumocytes – simple squamous epithelial cells – gas exchange

- Type II pneumocytes – regenerative of lamellar bodies → produce surfactant

Surfactant is a protein-lipid material that reduces surface tension. Main lipid (lipid and water to not mix) component of
surfactant is dipalmitoylphosphatidylcholine→ reduction liquid to air ratio in aveioli

Corticosteroids increase surfactant production

Insulin inhibits surfactant production

34 to 35 weeks of gestation – Mature levels of surfactant

Muscles in respiration is passive mostly– inspiraton is active


− Diaphragm is the primary respiratory muscle quiet inspiration at rest

− innervated by the phrenic nerve originates from the C3, C4, and C5 spinal nerves.

Diaphragm also forms the boundary between the thoracic and abdominal cavities – different oint examples:

Structures passing through Diaphragm

− IVC - T8 – also have thoracic duct

− Esophagus - T10 – penetrates the diaphram

− Aorta - T12 – from thoracic cavity to abdomen – esophagel vein

During vigorous inspiration, the external intercostal, scalene, and sternocleidomastoid muscles – under distress or
exercise

Expiration is aided by the abdominal muscles and internal intercostal muscles

Physiology – how it works – how we breathe -


In asthma – use of to breath out – use as much force to breath out – abdominal muscles, one thing – -use abdominal
muscle to breath out – abdominal thoracic expiration

Spirometry measures only changes in lung volume. - Cannot measure residual volume (RV), TLC and FRC

What can we measure:


⚫ Tidal volume (Vt): amount of air that enters or leaves the lung in a single respiratory cycle (500 mL)

⚫ Functional residual capacity (FRC): amount of gas in the lungs at the end of a passive expiration; the neutral or
equilibrium point for the respiratory system (2,700 mL); a buffer so that lungs do not collapse - it is a marker for
lung compliance

Key – which volume – breath – Tidal volume – normal – when finish what is left – FRC,

⚫ Without the Functional residual capacity FRC acting as a buffer for continued gas exchange during apneic
periods of inspiration and expiration, these conditions would in effect create an intrapulmonary shunt, inducing
deoxygenated blood from the pulmonary capillaries to empty into the pulmonary veins.

Inspiratory capacity (IC): maximal volume of gas that can be inspired from FRC (4,000 mL) -breath in as much as can

⚫ Inspiratory reserve volume (IRV): additional amount of air that can be inhaled after a normal inspiration (3,500
mL)

⚫ Expiratory reserve volume (ERV): additional volume that can be expired after a passive expiration (1,500 mL)

⚫ Residual volume (RV): amount of air in the lung after a maximal expiration (1,200 mL)

⚫ Vital capacity (VC): maximal volume that can be expired after a maximal inspiration (5,500 mL)

⚫ Total lung capacity (TLC): amount of air in the lung after a maximal inspiration (6,700 mL)

Ventilation – come in handy – key thing – is to remember names

Total ventilation - minute volume or minute ventilation.

− Total volume of air moved in or out (usually the volume expired) of the lungs per minute.

⚫ V•E = VT × f

Dead Space - Regions of the respiratory system that contain air but are not exchanging O2 and CO2. -

Anatomic dead space

− Anatomic dead space (anatVD) includes the conducting zone

− The size of the anatVD in mL is approximately equal to a person’s weight in pounds. Ex in 145 lbs →
145mls is the amount of air in my anatomic dead space

Composition of the anatomic dead space and the respiratory zone


Alveolar dead space - (alvVD) refers to alveoli containing air but without blood flow in the surrounding capillaries.
Physiologic dead space (physioIVD) refers to the total dead space in the lung system (anatVD + alvVD).

Blood heavier than air – apex of lung – low perfusion, base → higher perfusion than ventilaton -

Can have no blood flow to lung – so increase in dead space

⚫ Minute ventilation (V•) is the total volume of air entering the lungs per minute

Alveolar Ventilation – to know how much air gets to alveoli – out of 500 mls some in conducting zone – so we
subtract the dead space from tidal volume and multiply by frequency of breathing – when conditions with shallow
breathing → or rapid → px have total alveolar ventilation. Ex rapid breathing → In rapid, shallow breathing, total
ventilation may be above normal, but alveolar ventilation may be below normal. If px breathing fast – so needs
mechanical ventilation

Alveolar ventilation V•A represents the room air delivered to the respiratory zone per breath.

− V•A = (VT - VD) f

9:45 Jan 23 – end of class - Respiratory lecture notes

Lung Pressures
Lung recoil - the inward force created by the elastic recoil properties of alveoli. Acts to collapse the lung
Chest wall recoil - the outward force of the chest wall
FRC - Functional residual capacity as counterbalance
Intrapleural pressure (IPP) – pressure generated by the Pleural cavity - outward recoil of the chest and inward
recoil of the lung create a negative (subatmospheric) IPP. Contributes no lung recoil and no chest wall recoil

Transmural pressure gradient - pressure gradient across any tube or sphere - Inside pressure minus outside
pressure - At FRC, IPP is negative, and thus PTM is positive. This positive outward force prevents alveolar collapse -
Atelectasis
Note then - IPP has to be negative – pleural cavity – when px has fluid build up in pleural cavity → the pressure in IPP
shifts from -ve to +ve

Positive outward force prevents lung collapse – the difference p in and out → transmural - +ve to prevent aveoli collapse
ex tumor that compress lungs and reduce the transmural pressure and becomes-ve and → atelectasis

Each phase – IPP is -ve – and allow air to get into lung

Picture a balloon already slightly inflated vs complete deflated – which expand faster – the partly inflated – think of it as
+ve IPP greater than outside, compared to completely deflated as think of -ve – so apply to lung. The alveoli, must never
be completed deflated or collapsed → some air must be in alveoli

Stop here on Jan 23, 2023 – will continue tomorrow – January 24, 2023 – start with self study
Before Inspiration - The lung is at FRC Functional Residula Capacity is the buffer so lungs do not collapse – IPP
Intrapleural pressure is negative at FRC because the inward elastic recoil of the lungs is opposed by the outward-
directed recoil of the chest wall. Alveolar pressure must be zero (picture a balloon – if it had +ve pressure, and could
not take in more air easily. So balloon is not overinflated). If IPP was +ve – ex px with pneumothorax – bleeding in chest
– blood in pleural cavity – so +ve IPP, compress on lungs and impede expansion of the lungs.

During Inspiraton - Intrapleural pressure becomes more negative (the amount of air coming to the lungs) The more
negative IPP causes PTM (TPP – trans pulmonary pressure gradient – pressure inside minus pressure
outside pulmonary structure and increases during inspiration) to increase, which in turn causes expansion of the lungs.
The greater the contraction, the greater the change in intrapleural pressure and the larger the PTM (TPP) expanding
the lung. The expansion of the lung increases alveolar volume. the rise in volume causes pressure to decrease, resulting
in a negative (subatmospheric) alveolar pressure. Air rushes into the lungs. The lung expands until alveolar pressure
equilibrates with atmospheric pressure.

Transmural pressure gradient - pressure gradient across any tube or sphere. Inside pressure minus outside pressure

At FRC, IPP is negative, and thus PTM is positive. This positive outward force prevents alveolar collapse (atelectasis).

The transmural pressure (PTM) is the difference between intrapleural pressure ( PA) and alveolar pressure (Pa), [PTM=
PA - Pa]. If the intrapleural pressure is more negative, the lungs increase in volume to expand. However, if the
intrapleural pressure is positive, the lungs will collapse, which decreases lung volume.

TPP – transpleural pressure , as lungs and rising of alveoli volume → pressure in alveoli – P will decrease and -ve

When you inspire – IPP – is always -ve before inspire – and becomes more -ve as you inspire. As air moves into lungs –
pressure becomes more +ve in lungs c→ expanding the lungs. Ex Surgery, px who had stab – air in thorac in pleural
cavity. Tx drain out air – px refused → pneumothorax – how about 100% O2 – over the weekend – his lungs gradually
expanded. The pressure giving 100% O2 would maintain increase airflow into the lungs and TPP increase and gradually
expand the lung. The air in the pleural cavity. If same px placed on no O2 and breathe on on → the O2 is only 21% of
atmosphere, not sufficient for adequate expansion of the lungs. → further lung collapse. IPP TPP

Expiration – passive activity – relaxation of the muscles of inspiration causes intrapleural pressure increase. Decrease
of PTM transmural pressure leads to the drop of alveolar volume, which increases alveolar pressure.

Intraalveolar pressure – it is slightly negative during inspiration and slightly positive during expiration. No matter
how large a breath is taken – intraalveolar pressure always returns to 0 at the end of inspiration and expiration. Total
atmospheric pressure =0

Heart and lung work together – when we inspire – IPP is always -ve , and TMP in lungs and will increase and become +ve
and cause the lungs to expand, also have blood vessel in thoracic cavity, and muscle - when expire IPP – will go from -8
to -5 – but never +ve. Opposite of inspiration. TMP would decrease and cause drop in aveolar volume and means the
walls are coming together.

No matter how big a breath the TAP always returns to 0


Cardiovascular Changes with Ventilation
Inspiration – Intrapleural pressure becomes more negative (decreases). Increases the PTM across the vasculature,
causing the great veins and right atrium to expand

This expansion decreases intravascular pressure, thereby increasing the pressure gradient driving VR to the right heart.

Systemic venous return and right ventricular output are increased. An increase in the output of the right ventricle delays
closing of the pulmonic valves and typically results in a splitting of the second heart sound. Pulmonary vessels expand
and the volume of the blood in the pulmonary circuit increases. Pulmonary vascular resistance (PVR) is lowest at FRC.

Venous return to the left heart, and the output of the left ventricle is decreased.

Decreased systemic arterial pressure (drop in systolic most prominent). Inspiration reduces vagal outflow to the heart
resulting in a slight rise in heart rate (respiratory sinus arrhythmia).

Expiration - Reverse process


→ Valsalva maneuver is a forced expiration against a closed glottis. This forced expiration creates a positive
IPP which compresses the great veins in the chest. This in turn reduces VR. Example - when force down without
inspiration during bowel movement. Valsalva – increase IPP in summary

Clinically →many uses – limits blood flow to rt side of heart – and reduce blood pressure

Ventilators – need to know each of these ventilators


Positive Pressure Ventilation
Assisted Control Mode Ventilation (ACMV) - The inspiratory cycle is initiated by patient or automatically.
Expiration is accomplished in the normal manner

Positive End-Expiratory Pressure (PEEP) - Positive pressure is applied at the end of the expiratory cycle to
decrease alveolar collapse. It is useful in treating the hypoxemia of acute respiratory distress syndrome. Small alveoli
have a strong tendency to collapse, creating regions of atelectasis. The larger alveoli are also better ventilated, and
supplementary oxygen is more effective at maintaining a normal arterial PO2. Positive pressure ventilation and
accentuated by PEEP causes a decrease in venous return and cardiac output.

Normally – when have conductive zone – px that have snoring, or obstructive sleep apnea – the problem – obstructs the
airway – want to maintain continuous flow of air – so give device +ve pressure to keep it open. In alveoli which are small
have strong tendency to collapse – so want to give +ve pressur . Why →Small balloon is harder to inflate – small alveoli
greater tendency to collapse – so if low surfactant – we give +ve pressure

Continuous Positive Airway Pressure (CPAP) - CPAP, continuous positive pressure is applied to the
airways. Useful to treat obstructive sleep apnea (OSA) since the lung and upper airways (nasopharynx) remain at a
larger volume throughout the respiratory cycle.CPAP is administered by mask. The patient breathes spontaneously.

Pneumothorax – Traumatic (perforation of chest wall) or spontaneous (rupture of an alveolus). Intrapleural


pressure increases from a mean at -5 cm H2O to equal atmospheric pressure.
Lung recoil decreases to zero as the lung collapses. Chest wall expands. At FRC, the chest wall is under a slight tension
directed outward. It is this tendency for the chest wall to spring out and the opposed force of recoil that creates the
intrapleural pressure of -5 cm H2O.

Transpulmonary pressure is negative. IPP – go from -5 up to 0 and even +ve –

Tension pneumothorax most commonly develops in patients on a positive-pressure ventilator. TPM – as try to
inhale in this – lungs fail to expand – pressure out is > inside. Happens on +ve pressure ventilator

Lung compliance – ability to receive air - Lung compliance, or pulmonary compliance, is a measure of the lung's
ability to stretch and expand

Lung compliance = Change in lung volume (tidal volume)

change in surrounding pressure.

Increased compliance means more air will flow for a given change in pressure. Reduced compliance means less air
will flow for a given change in pressure

Surface tension forces – Liquid air interface - - tend to reduce the surface area that allow gases –

Components of Lung Recoil - Collagen and elastin fibers of the lung


The larger the lung, the greater the stretch of the tissue and the greater the recoil force.

− The surface tension forces in the fluid lining the alveoli. Surface tension forces are created whenever
there is a liquid–air interface.

Surface tension forces tend to reduce the area of the surface and generate a pressure. They act to collapse the alveoli;
therefore, these forces contribute to lung recoil. Surface tension forces are the greatest component of lung recoil.

Note - Law of LaPlace. Pressure ∝tension/radius

→Surface tension greater in small balloon – tension across a sphere – smaller aveloil have greater tendency to collapse

Surfactant reduces the surface tension and prevents collapse of the small alveoli. It lowers surface tension forces more
in small alveoli than in large alveoli.

Respiratory distress syndrome – tomorrow –Jan 25, 2018


Infant RDS (hyaline membrane disease) is a deficiency of surfactant.

Adult respiratory distress syndrome (ARDS) is an acute lung injury


Dolapo – mcqs

1 45-year-old man – spironolactone and digoxin for CHF → low K+ and high Na+, high-normal urine volume

2 drug side effects compensatory tachycardia and exacerbate flid retention, lupus like syndrome → mechanism
increased cGMP
AIRWAY RESISTANCE
Radius of an Airway - The first and second bronchi – site of most of the airway resistance.

− Parasympathetic nerve stimulation produces bronchoconstriction.

− M3 receptors; increases airway secretions.

Circulating catecholamines produce bronchodilation. Beta-2 receptors.Resistance 1/radius 4

If want to increase resistance, have to decrease radius of airway

Mechanical Effect of Lung Volume


As lung volume increases, airway resistance decreases. IPP becomes more and more negative. This increases the PTM
across small airways, causing them to expand. The result is decreased resistance.

Walls of alveoli are physically connected to small airways. As alveoli expand, they pull open small airways. The result is
decreased resistance.

IPP- mor negative – and Pulmonary trans Pressure across small airways – decreased resistance.

Pulmonary Function Testing


FVC: this is total volume exhaled - Age, gender, body size influence the absolute amount of FVC, it is expressed as a
percent of predicted (100% of predicted being the “ideal”).

FEV1 (forced expiratory volume in 1 second):the volume of air exhaled in the first second; it is commonly compared to
the FVC such that one determines the FEV1/FVC ratio. - This ratio creates a flow parameter; 0.8 (80%) or greater is
considered normal.

Restrictive pulmonary disease is characterized by reduced volume (low FVC, but normal flow), while
obstructive disease is characterized by reduced flow (low FEV1/FVC).

FEV1 – in 1 sec and compared to FVC → FEV1/FVC should be >80%,

Low ratio → obstructive. If normal or increased → restrictive pulmonary disease


Obstructive – asthma – expire is volume decreased

Obstructive vs Restrictive
Obstructive – increase in airway resistance that is measured as a decrease in expiratory flow. Examples: chronic
bronchitis, asthma and emphysema. Pattern: Total lung capacity ↑TLC will be normal or larger than normal. But during
maximal forced expiration from TLC, a smaller than normal volume is slowly exprired. FVC may or may not be less.
Restrictive pulmonary disease - ↑elastic recoil— ↓ lung compliance—↓ all lung volumes.
Examples - ARDS, interstitial lung diseases → sarcoidosis and idiopathic pulmonary fibrosis (IPF). ↓ - FEV1↓, the
FEV1/FVC↑. Low FVC with low FRC and RV

Acute Respiratory Distress Syndrome ARDS, sarcoidosis and IPF


TLC is smaller than normal, lungs have recoiled, TLC is smaller than normal, and FEV1/FVC ratio increased, but low FVC

Note – have to differentiate these 2 - #1 you have to know – physiology and pathology →

FVC is always decreased when pulmonary


function is significantly compromised.
A decrease in FEV1/FVC ratio is evidence of
an obstructive pattern.
A normal or increased FEV1/FVC ratio is
evidence of a restrictive pattern.
Low TLC is diagnostic of restrictive lung
disease.
C Restrictive – compliance of lungs reduced, and not just also elastic recoil has increased ->

D Variable extra thoracic – px has scoliosis effects inspiration, but expiration remains normal. E – fixed upper → when
px has sleep apnea – flattened line F myasthenia gravis – complete reduction of both

Alveolar Gas Exchange - Partial Pressure of a Gas in Ambient Air Pgas = Fgas × Patm

Partial Pressure of a Gas in Inspired Air PIgas = Fgas (Patm – PH2O)

Inspired air is defined as air that has been inhaled, warmed to 37°C, and completely humidified, but has not yet engaged
in gas exchange. Partial pressure of H2O is 47 mm Hg

Oxygen is only one diffuses across capillary PP of gas in ambient air, PP of gs and inspired air.

PO2 and PCO2 in the alveolar compartment and pulmonary end capillary blood are the same (perfusion-limited).

There is a slight change (PO2↓) between the end capillary compartment and systemic arterial blood because of a small
but normal shunt through the lungs.
Alveolar–systemic arterial PO2 differences = A - a O2 gradient.

This difference (5-10 mm Hg) often provides information about the cause of a hypoxemia.

AA gradient – difference between the alveolar O2 PP and the artery – but the time it crosses – some is lost, shunt that
results in some of the O2 moving back to venous circulation – AA gradient exceeds limit – what is range AA – 5-10mmHg.
If greater, than something unusual happening. Check for Hypoxia in px

Factors Affecting Alveolar PCO2 →metabolic rate and alveolar ventilation

Alveolar ventilation: main factor affecting alveolar PCO2 → ventilation increases,PACO2 decreases; if ventilation
decreases, PACO2 increases

Alveolar PCO2 and body metabolism have a direct metabolism → Exercise, Body temperature

CO2 preset in air, but also produced in body from breakdown – by product of metabolism. What effect PC02 rate change
→ metabolism rate and ventilation rate (breathing rate) determine this

Ventilation – example effected by – what is breathe rate. If increase than PACO2 decreases and inverse

Metabolic rate – exercise, body temp effect this.

PAO2 = (Patm - 47) FiO2 – PACO2/RQ

Patm = atmospheric pressure, at sea level 760 mm Hg


Increase in atmospheric pressure (hyperbaric chamber) increases alveolar PO2, and a decrease (high altitude) decreases
alveolar PO2.

FiO2 = fractional concentration of oxygen , room air 0.21


− An increase in inspired oxygen concentration increases alveolar PO2.

PaCO2 = alveolar pressure of carbon dioxide, normally 40 mm Hg


- Increase in alveolar PCO2 decreases alveolar PO2, and a decrease in alveolar PCO2 increases alveolar PO2

RQ - Respiratory quotient (RQ) is the ratio between CO2 production and O2 consumption at the cellular level.
− Normally 0.8

ALVEOLAR–BLOOD GAS TRANSFER: FICK LAW OF DIFFUSION - V•gas= A/T×D × (P1-P2)

A = surface area for exchange, ↓ in emphysema, ↑ in exercise

T = thickness of the membranes between alveolar gas and capillary blood, ↑ in fibrosis and many other restrictive
diseases

A loss of surface area and/or an increase in the thickness of the membrane system between the alveolar air and the
pulmonary capillary blood. The rate of oxygen and carbon dioxide diffusion decreases. The greater the structural
problem, the greater the effect on diffusion rate
Sea level – 760 mmHg

What can affect surface area? – typically – emphysema – destroys surface. Exercise tends to increase it.
Thickness – as thicker – ex fibrosis. Area and thickness – gas plays a role – does it dissolve easily ex O2 and CO and CO2
have different solubility coef cf, CO 2faster than O2

Diffusion limiting – DLCO index as measure – CO is unique – diffusion limited gas

Diffusing Capacity of the Lung - If the substance equilibrates between the capillary and interstitium, it is
said to be in a perfusion-limited situation. If the substance does not equilibrate between the capillary and interstitium, it
is said to be in a diffusion-limited situation.

DLCO is an index of the lung’s structural features.


Carbon monoxide is a unique gas - Doesn’t equilibrate between the alveolar air and the capillary blood. A diffusion-
limited gas.

Transport of O2 and CO2 and the Regulation of Ventilation


⚫ Oxygen content = 0.2 mL of oxygen per mL of blood

Oxygen is transported in the blood in two forms, dissolved oxygen and oxygen bound to hemoglobin

⚫ O2 is poorly soluble in blood, so at a normal Pao2 of 100mm Hg, the amount of oxygen dissolved in blood is
negligible and insufficient to meet the body’s metabolic requirements

⚫ Hemoglobin can bind four O2 molecules when the iron is in the ferrous form (Fe2+).

Oxygen Content of Blood (Cao2) - CaO2 = (amount of oxygen bound to hemoglobin, major) + (amount
of oxygen dissolved in plasma, minor)

⚫ CaO2 = (1.34 × Hb × SaO2) + (0.003 × PaO2)

Oxygen Delivery to Tissues DO2 = Q × CaO2 - Q is the cardiac output.

Hemoglobin-O2 Dissociation Curve - Percentage saturation of Hb as a function of partial pressure of


O2 (Po2). Sigmoidal in shape Positive cooperativity Bohr effect - Decreased pH and increased Pco2 on the Hb-O2
dissociation curve
Shift to right – what factors → Bohr effect pH and pCO2, acidosis ,↑ temp DPG, sickle cell HB. – Hb is willing to upload
O2 to body

Left sift – opposite – Hb is holding on to O2 tightly, when to right. Loading and offloading of O2 to body and this is what
the graph tells us

Carboxyhemoglobin - A form of Hb created when carbon monoxide is bound to Hb instead of O2.


⚫ CO binds to Hb with a 250-fold greater affinity than O2 and reduces the O2-carrying capacity of Hb by binding
where O2 normally would.

⚫ Causes a leftward shift of the Hb-O2 dissociation curve

− often described as having cherry red skin, Pulse oximetry reading may be falsely normal

− Best test for oxygen saturation in carbon monoxide poisoning would be an arterial blood gas.

− Treated with 100% O2 at very high flow rates

Methemoglobin - When the Fe2+ in Hb is oxidized to ferric iron (Fe3+), it is called methemoglobin and is unable to
bind O2. Drug exposures (e.g., nitrates, dapsone), hemoglobinopathies, and enzyme deficiencies.

⚫ May have “chocolate brown” blood and cyanosis.

⚫ It is treated by administering methylene blue intravenously

Prevent cyanide – give Na nitrate and Ferrous – give methyline blue IV - binds to O2
CO2 transport –

Ventilation-to-Perfusion Ratio - Ratio of alveolar ventilation (V) to pulmonary perfusion (Q)


⚫ V/Q ratio across the entire lung in a person who is standing happens to be about 0.8.

⚫ V/Q is uneven across the entire lung due to uneven perfusion


The V/Q ratio is greatest in zone 1 (where V/Q = 3) and lowest in zone 3 (where V/Q = 0.6)

V/Q Mismatch - Two extremes of V/Q mismatch

Shunt describes a situation in which blood flow is normal but zero ventilation takes place (i.e., V/Q = 0).
Lobar pneumonia with alveolar infiltrates, bronchial cancer, aspiration

Shunted blood mixes with blood from well ventilated region and causes a drop in PaO2 - widening of the A-a gradient

Dead space: areas of lung that are well ventilated are not perfused (V/Q = ∞) - Pulmonary Embolus

Both extremes of V/Q mismatch result in elevations of the A-a gradient but can be differentiated by the administration
of 100% O2

In which of the following ways does myasthenia gravis cause hypoxemia? A. Neuromuscular junction pathology causes
hypoventilation, leading to chronic hypoxemia B. Increases the A-a oxygen gradient C. Fibrosis and sclerosis of the
alveoli cause diffusion impairment D. Ventilation-perfusion mismatch caused by a fibrotic scar form in the apex of the
lung E. Complete occlusion of an airway caused by a sclerotic foreign body

Answer A - FA 675 Note (D – TB)

Break – Self study

Wednesday Sept 21, 2022

Difference between Obstructive and Restrictive lung disease


Obstructive diseases make it hard to exhale all the air from your lungs. Restrictive diseases make it hard to fill the lungs
with air and inability to expand or loss of elastic recoil of lungs.
Difference between malignant and benign neoplasms? (MD3 prof asked Dr. Okon this)

What cardio vascular changes during respiration? (my question) The heart rate increases during inspiration and
decreases during the post-inspiration/expiration period. This respiratory-related change in heart rate, respiratory sinus
arrhythmia (RSA), helps to match pulmonary blood flow to lung inflation and to maintain an appropriate diffusion
gradient for oxygen in the lungs and to compensate for the decreased left ventricular output while breathing in.

During inspiration, venous return increases as the thoracic cavity’s pressure becomes more negative. This reduced
intrathoracic pressure draws more blood into the right atrium. This results in greater venous return.

By increasing the heart rate with a reflex mechanism, body maintains the cardiac output. Cardiac output is the product
of heart rate and the volume of blood pumped out by the heart in a single beat.

In contrast, left ventricular output is higher in expiration and heart rate lower. Corresponding to this the volume of pulse
will be higher in expiration and lower in inspiration.

Project – map out atrial tracing arteries and then – arteries – sphenopalatine artery – common cause →
The sphenopalatine artery is a terminal branch
of the internal maxillary artery originating from
the external carotid artery system. The SPA is
the major blood vessel to the nasal cavity
mucosa: supplying the superior, middle, and
inferior turbinate; lateral nasal wall; and nasal
septum.

Rhino-Sinusitis

Rhino Sinusitis – obstruction of drainage into nasal cavity,


maxillary sinuses typically affected

Clinically important to know that most commonly caused by virus


→ (eg, rhinovirus); may lead to superimposed bacterial infection,
most commonly H influenzae, S pneumoniae, M catarrhalis.

Paranasal sinus infections may extend to the orbits, cavernous


sinus, and brain, causing complications (eg, orbital cellulitis,
cavernous sinus syndrome, meningitis)

January 25, 2023 – Dr Okon – 8 am

Epistaxis – Nose Bleed - Question- lets say you are in van – find a px nose bleed, now what are differential → kids
different from adults – Most commonly occurs in anterior segment of nostril (Kiesselbach plexus). (Posterior – more
likely to require attention)

●Life threatening hemorrhages occur in posterior segment (sphenopalatine artery, a branch of maxillary artery).
●Common causes include foreign body, trauma, allergic rhinitis, and nasal angiofibromas (common in adolescent males)

Causes – foreign body, trauma, allergic rhinitis, and nasal angiofibromas (common in adolescent males), cold, humidity,
hypertension

Differential - rhinosinitus, Hemophilia, von Willebrand, Warfarin toxicity, NSAIDS, ASA

How to treat→ px to 1) keep head elevated (vein from plexus) 2) pinch the nose at superior portion of nose 3) no relief
apply gauze with epinephrine (vasoconstriction). Sphenopalatine artery ligation is a commonly employed surgical
intervention for control of posterior epistaxis unresponsive to nasal packing.

Head and Neck Cancer - Mostly squamous cell carcinoma. Risk factors include tobacco, alcohol, know the virus
type HPV-16 →(oropharyngeal), EBV →(nasopharyngeal). Common.

Field cancerization: carcinogen damages wide mucosal area → multiple tumors that develop independently after
exposure

Nasopharyngeal carcinoma may present with unilateral nasal obstruction, discharge, epistaxis.

Eustachian tube obstruction may lead to otitis media, with or without effusion, hearing loss
Congenital Cystic Lung Lesions
Congenital cystic adenomatoid malformation (CCAM)−a hamartomatous lesion, abnormal lung
tissue that does not function properly, but continues to grow

Bronchopulmonary sequestration (BPS)−non-functioning bronchopulmonary segment separate from the


tracheobronchial tree. DX → Serial ultrasonography - Some resolve spontaneously, though minimally invasive surgery
may be required.

Congenital cystic adenomatoid malformation (CCAM)

Right side is typical , and left BPS – ultra sound – see section of lung and alveoli is enlarged and is typical

Atelectasis
An area of collapsed or nonexpanded lung −Reversible−Predispose to
infection due to decreased mucociliary clearance – can’t get rid of
debris.

Types−Obstruction/resorption atelectasis: collapse due to resorption


of air distal to an obstruction; examples include aspiration of a foreign
body- ex peanut in child, chronic obstructive pulmonary disease
(COPD), and postoperative atelectasis – after surgery.

Compression atelectasis- fluid, air, blood, or tumor in the pleural


space.

Contraction (scar) atelectasis - fibrosis and scarring of the lung.


Patchy atelectasis is due to a lack of surfactant, as occurs in
hyaline membrane disease of newborn or acute (adult) respiratory
distress syndrome (ARDS)

PULMONARY INFECTIONS
Bacterial pneumonia - Signs and symptoms include fever, chills; productive cough with yellow-green (pus) or
rusty (bloody) sputum; tachypnea – fast breathing; pleuritic chest pain; and decreased breath sounds (px takes very
shallow which do not get to deep lung), rales, and dullness to percussion.

Elevated white blood cell count with a left shift – because of infection. Bone marrow immature WBC→ band form

Lobar or segmental consolidation (opacification) bronchopneumonia- patchy opacification; → Pleural effusion


Lobular Pneumonia - Consolidation, Streptococcus pneumoniae (95%) or Klebsiella
4 classic phases→1 Congestion, 2 red hepatization,3 gray hepatization; 4 resolution (healing)

Broncho pneumonia - Scattered patchy consolidation centered on bronchioles. Microscopic examination of tissue
shows acute inflammation of bronchioles and surrounding alveoli. Diagnosis: Sputum gram stain, sputum culture, blood
cultures. Complications of pneumonia: Fibrous scarring and pleural adhesions, lung abscess, empyema (pus in a body
cavity), and sepsis. Treatment: Antibiotics

Lung Abscess - Localized collection of neutrophils (pus) and necrotic pulmonary parenchyma. Cause → Aspiration –
aspirate – common in alcoholic their own vomit. May also occur following a pneumonia, especially one due to S. aureus
or Klebsiella: include empyema, pulmonary hemorrhage and secondary amyloidosis

Atypical pneumonia−Interstitial pneumonitis without consolidation. Cause → virus - Mycoplasma pneumoniae,


influenza virus, parainfluenza virus, respiratory syncytial virus (RSV) (young children), adenovirus, cytomegalovirus
(CMV) (immunocompromised), varicella virus. Diagnosis −Chest x-ray - diffuse interstitial infiltrates. Diagnosis →
−Elevated cold agglutinin titer – Mycoplasma. −Lung biopsy - lymphoplasmacytic inflammation

Tuberculosis (TB) Acquired by inhalation of aerosolized bacilli. Symptoms - Fevers, night sweats, weight loss,
cough, and hemoptysis.

Primary pulmonary TB:−Initial exposure to the disease−Ghon focus of primary TB subpleural caseous granuloma
formation, either above or below the interlobar fissure.−Ghon complex refers to the combination of the Ghon focus and
secondarily-involved hilar lymph nodes with granulomas. Most primary pulmonary tuberculosis lesions (95%) will
undergo fibrosis and calcification.

Tuberculosis – FA 140 – Ghon complex – fibrosis and calcification

Progressive pulmonary TB −Cavitary tuberculosis, miliary pulmonary tuberculosis, and tuberculous


bronchopneumonia.

Secondary pulmonary TB (also known as post primary or reactivation TB) - −reactivation of an old, previously
quiescent infection or with reinfection secondary to a second exposure to the mycobacteria.- −friable nodule at the lung
apex (Simon focus) - −Biopsy of affected tissues will typically show AFB-positive caseating granulomas.
Dissemination: can spread to →−meninges; cervical lymph nodes (scrofula) and larynx; liver/spleen, kidneys, adrenals,
and ileum; lumbar vertebrae bone marrow (Pott disease); and fallopian tubes and epididymis.
Myobacteria M. Avium Complex (MAC) – occurs in AIDS px with CD4 counts <50 cell/mm3 → cause
disseminated disease

HIV – opportunistic infection in patient

Sarcoidosis – FA 676

Systemic granulomatous disease - Sarcoidosis is a disease characterized by the growth of tiny collections of
inflammatory cells (granulomas) in any part of your body — most commonly the lungs and lymph nodes. But it can also
affect the eyes, skin, heart and other organs.

Females > males with typical age 20–60. - Most common in African American women.

Clinical presentation varies. Asymptomatic, or presenting symptoms may include cough and shortness of breath; fatigue
and malaise; skin lesions; eye irritation or pain; and fever or night sweats.

Detected on chest x-ray as bilateral hilar lymphadenopathy or parenchymal infiltrates.

Noncaseating granulomas may occur in any organ of the body.


Lung - diffuse scattered granulomas - Lymph node involvement - hilar and mediastinal adenopathy.

Skin, liver and/or spleen, heart, central nervous system, bone marrow, and gastrointestinal tract are also frequent
targets of the disease.

Eye involvement can be seen in Mikulicz syndrome (involvement of the uvea and parotid).

Diagnosis−Serum angiotensin converting enzyme (ACE) - may be elevated. Vit D −X-ray studies - bilateral hilar
lymphadenopathy. −No pathognomonic microscopic features
Prognosis is favorable

***Notebook- non-caseating granulomas


Obstructive Pulmonary Disease

- must know difference between obstructive and restrictive

Restrictive– shipyard – and smoke etc – mixed disorder– must know this
Chronic obstructive pulmonary disease (COPD) - −Chronic bronchitis or emphysema. −both associated with
smoking.

Chronic bronchitis – COPD - Definition−Persistent cough and copious sputum production for at least 3 months
in 2 consecutive years. −cough, sputum production, dyspnea, frequent infections, hypoxia, cyanosis, and weight gain

Pathology – irritant – excess mucus producing goblet cells, Hypertrophy and hyperplasia of bronchial mucous glands,
inflammation leading to Fibrosis

Microscopy - Reid index equals the submucosal gland thickness divided by the bronchial wall thickness between the
pseudostratified columnar epithelium and the perichondrium; normal ratio is ≤0.4).

Complications: Increased risk for recurrent infections; secondary pulmonary hypertension leading to right heart failure
(cor pulmonale) and can lead to lung cancer.

Long-standing severe chronic bronchitis leads to Cor-pulmonale with cardiac failure


Cor pulmonale – rt heart failure due to lung failure- vs rt heart failure – difference e- lung – chronic bronchitis – can lead
to cancer as well

Emphysema – COPD - Definition: Abnormal enlargement of air spaces distal to the terminal bronchioles
characterized by destruction of the alveolar septa with little or no fibrosis Destruction of alveolar septa results in
enlarged air spaces and a loss of elastic recoil.

4 Types
Centrilobular emphysema - proximal portion of the acinus and the cause is cigarette smoking.
Panacinar emphysema - the entire acinus and the common cause is alpha-1 antitrypsin deficiency.

Distal acinar emphysema (unknown cause) - extension to the pleura causes pneumothorax.
Irregular emphysema, post-inflammatory scarring involves the acinus in an irregular distribution.

Etiology - Protease/antiprotease imbalance. Lungs are overinflated and enlarged−Progressive dyspnea, pursing of lips
and use of accessory respiratory muscles to breathe, barrel chest (increased anterior-posterior diameter), and weight
loss. Lungs are overinflated.

Both Chronic Bronchitis and Emphysema are OBSTRUCTIVE


Asthma – Chronic Hyperreactive airways, which undergo episodic bronchospasm – FA 674- 4 types:
1 Atopic (type I IgE-mediated hypersensitivity reaction) asthma−most common form−positive family history

2 Nonatopic asthma−Triggers - respiratory infections (usually viral), stress, exercise, or cold temperatures.

3 Drug-induced asthma−10% of adults−Example: Aspirin

4 Occupational asthma−workplace triggers

Clinical → Wheezing, severe dyspnea, and coughing, Lung overinflation. Status asthmaticus is a potentially fatal
unrelenting attack of asthma. Microscopic examination - Curschmann spirals (twisted mucus plugs admixed with
sloughed epithelium), eosinophils, or Charcot-Leyden crystals (protein crystalloids from broken down eosinophils).

3 features – 1 chronic – 2 bronchial hyper reactive airway, and 3) reversible

- take in breath and trap the air. Chest x-ray lungs overinflated, & darker than usual due to trapped air.

Autopsy findings include mucus plugs, increased mucous glands with goblet cell hyperplasia, inflammation (especially
with eosinophils), edema; hypertrophy and hyperplasia of bronchial wall smooth muscle, and thickened basement
membranes

Bronchiectasis - Abnormal permanent airway dilatation due to chronic necrotizing inflammation. Clinical findings
include cough, fever, malodorous purulent sputum, and dyspnea. Causes - bronchial obstruction by foreign body,
mucus, or tumor, necrotizing pneumonias, cystic fibrosis, and Kartagener syndrome. Dilated bronchi and bronchioles
extending out to the pleura. Complications - abscess, septic emboli, cor-pulmonale, and secondary amyloidosis

FA 675

Kartagener Syndrome - Kartagener syndrome - autosomal recessive condition caused by immotile cilia due
to a defect of dynein arms (primary ciliary dyskinesia). HEART on right side → dextrocardia due absent ciliary
movement in embryogenesis. Characterized clinically by bronchiectasis, chronic sinusitis, and situs inversus – FA
49 – ear infection, less fertility. Cilia is defective. In these px, the heart on left, may be on right side. The ceum also
reversed.

Restrictive Lung Diseases


Acute respiratory distress syndrome (ARDS): Diffuse damage of alveolar (DAD)– FA 678
Definition: Non-cardiac pulmonary edema with refractory hypoxemia (despite hi flow O2)- impaired gas exchange

Rapid onset of noncardiac pulmonary edema & progressive refractory hypoxemia. The decrease of arterial oxygen
despite administration of oxygen at high flow rates → Refractory hypoxemia,

Diagnosis – X-ray bilateral lung opacities, respiratory failure within 1 week of alveolar insult, Decreased PaO2/FiO2 <300,
hypoxemia and not due to HF/fluid overload, affected lungs are heavy, stiff and noncompliant

Microscopically - Intra-alveolar edema, and hyaline membranes line the alveolar spaces
Pathophysiology - Acute lung trauma from unregulated systemic inflammatory response to acute injury or inflammation
of epithelium and capillaries, resulting in progressive respiratory failure that is unresponsive to oxygen treatment.

Clinicians use the term ARDS, while pathologists use the term diffuse alveolar damage (DAD).

Etiology: shock, sepsis, trauma, gastric aspiration, radiation, oxygen toxicity, drugs, or pulmonary infection. -Activated
neutrophils mediate cell damage.

Clinically, patients show dyspnea, tachypnea, hypoxemia, cyanosis, and use of accessory respiratory muscles.

Shock from bleeding, heart failure , sepsis, trauma, gastric aspiration, Ex – px vital signs – bp increase, pulse increase,
130 bp temp high, kidney – creatine elevated, Na low, K elevated, liver function → sepsis – uncontrol bacterial infection.

TX – treating underlying cause, and ventilation - O2, prone ventilation with patients lying face down to improve oxygen
flow – resolution – need a regeneration of new cells. – Prognosis – 40-50%

Respiratory Distress Syndrome


RDS in newborn – Type II – lung don’t’ mature before 24 weeks – Hyaline membrane disease of newborn – due to
→ surfactant deficiency.

Associated with: multiple births, males, cesarean, maternal diabetes - Diabetes – insulin – surfactant production – when
mother has high glucose – goes into fetus, and stimulate pancreas to release large amt of insulin in infant ,k it also block
surfactant → RDS in infant - **** Infants are normal at birth but within a few hours develop increasing respiratory
effort, tachypnea, nasal flaring, use of accessory muscles of respiration, an expiratory grunt, and cyanosis even if not
born premature – but born to diabetic ,

Chest radiograph - bilateral “ground-glass” reticulogranular densities. Autopsy findings - atelectasis and hyaline
membranes. Treatment is surfactant replacement, mechanical ventilation, and continuous positive airway pressure
(CPAP). Respiratory distress syndrome of the newborn - prevented if labor can be delayed; corticosteroids are used to
mature the lung. With improved therapies, now >90% of babies survive.

Multiple birth tends to have RDS – as not born to term, earlier born the lungs not fully mature, male gender, TX –
surfactant, mechanical ventilation
Still looking at Restrictive lung Diseases:

Chronic interstitial lung disease


Idiopathic pulmonary fibrosis (IPF) - fatal disease. −Patchy interstitial fibrosis and inflammation. “Honeycomb
fibrosis” refers to dilated cystic spaces lined with type II pneumocytes; this histology is consistent with end-stage lung.

Nonspecific interstitial pneumonia−Better prognosis than IPF. −Cellular pattern and a fibrosing pattern.

Cryptogenic organizing pneumonia−responds to steroids. −Plugs of connective tissue inside the alveolar spaces.
Collagen vascular disease pneumonitis−Patterns of parenchymal and pleural involvement.−Prognosis is poor.

Smoking-related pneumonitis−Desquamative interstitial pneumonia features alveolar macrophages.

Respiratory bronchiolitis features bronchiolocentric macrophages.−Smoking-related interstitial fibrosis shows septal


collagen deposition without significant associated inflammation.

Hypersensitivity pneumonitis −exposure to moldy hay, patients present with a febrile acute reaction or a chronic
disease with weight loss. −Biopsy shows peribronchiolar acute and chronic interstitial inflammation +/- noncaseating
granulomas. −The disease is immunologically mediated.

Eosinophilic pneumonia −Septal inflammatory infiltrate and eosinophils within alveolar spaces.
Loeffler’s syndrome is a self-limiting type of eosinophilic pneumonia with peripheral blood eosinophilia.
Break Jan 25, 2023 – Wednesday January 26, 2023 -

Occupation-associated pneumoconiosis - Pneumoconiosis are fibrosing pulmonary diseases caused by inhalation of


an aerosol (mineral dusts, particles, vapors, or fumes)−Dose and duration of exposure; and the size of the particle.

Coal worker’s pneumoconiosis −Due to anthracosis, - carbon pigment (anthracotic pigment)accumulates in


macrophages along the pleural lymphatics and interstitium. −Clinically, the disease may progress through several stages.
The earliest stage is asymptomatic.−Simple and Complicated types

Caplan syndrome is the term when pneumoconiosis (of any type) accompanies rheumatoid arthritis.

Asbestosis:−Associated with asbestos inhalation from exposure to shipbuilding, insulation around plumbing, roofing
material, floor tile, and ceiling tile. −Deposit inside the alveoli when inhaled. Alveolar macrophages phagocytose the
asbestos fibers and coat them with iron, creating golden brown, fusiform, dumbbell-shaped rods within the
macrophages called asbestos bodies or ferruginous bodies. −Asbestosis predominantly affects the lower lobes. Causes
benign calcified pleural plaques, which are not malignant precursors. → Asbestosis is associated with an increased risk
of mesothelioma, a malignancy with a poor prognosis that arises from the pleural serosa and eventually encases and
traps the lung. −Most common malignancy associated with asbestosis is bronchogenic carcinoma. −Asbestosis can
cause interstitial fibrosis and may also present with Caplan syndrome. There is no increased risk of tuberculosis with
asbestosis.

Silicosis - exposure to silicon dioxide (silica). −Sandblasters, metal grinders, miners.−Pulmonary pathology shows
dense nodular fibrosis of the upper lobes−Birefringent silica particles can be seen with polarized light.−Insidious onset of
dyspnea that is slowly progressive−X-ray shows fibrotic nodules in the upper zones of the lungs. −There is an increased
risk of TB.
Berylliosis - allergic granulomatous reaction −Exposure to beryllium in the nuclear, electronics, and aerospace
industries. −Genetic susceptibility appears to play a role, as does a type IV hypersensitivity reaction, resulting in
granuloma formation. −Clinically, acute exposure causes acute pneumonitis, while chronic exposure causes pulmonary
noncaseating granulomas and fibrosis, hilar lymph node granulomas, and systemic granulomas.

Berylliosis – beryllium in nuclear industry → non-caseating granulomas (see Sarcoidosis previously)

VASCULAR DISORDERS
Pulmonary edema - fluid accumulation within the lungs−due to imbalance of Starling forces or endothelial
injury.−Increased hydrostatic pressure → left-sided heart failure, mitral valve stenosis, high altitude pulmonary edema,
and fluid overload.−Decreased oncotic pressure → nephrotic syndrome and liver disease.

Increased capillary permeability - infections, drugs (bleomycin, heroin), shock, and radiation.
Macroscopy: Wet, heavy lungs (usually worse in lower lobes),

Microscopic examination shows intra-alveolar fluid, engorged capillaries, and hemosiderin-laden macrophages (heart-
failure cells)

Pulmonary hypertension – whenever hypoxia – decrease O2 delivery → the blood vessel – dilate, and lungs
hypoxia, you have only in lung → vaso constriction

Pulmonary hypertension- increased pulmonary artery pressure −Etiology - chronic obstructive pulmonary disease,
interstitial disease (hypoxic vasoconstriction); multiple ongoing pulmonary emboli; mitral stenosis and left heart failure.
−congenital heart disease with left to right shunts (atrial septal defect, ventricular septal defect, patent ductus
arteriosus); −primary (idiopathic) pulmonary hypertension, typically in young women.

Pulmonary artery atherosclerosis, small artery medial hypertrophy and intimal fibrosis, and plexogenic
pulmonary arteriopathy. Pulmonary hypertension may also damage the heart, leading to right ventricular hypertrophy
and then failure (cor pulmonale).

PULMONARY NEOPLASIA – Lung Cancer


Leading cause of cancer death in both men and women; - It has been increasing in women (increased smoking) in the
past few decades. - Most commonly age 50–80. Major risk factors include cigarette smoking, occupational exposure
(asbestosis, uranium mining, radiation, etc.), passive smoking, and air pollution.

Clinical features include cough, sputum production, weight loss, anorexia, fatigue, dyspnea, hemoptysis, and chest pain.
Obstruction may produce focal emphysema, atelectasis, bronchiectasis, or pneumonia. -Common genetic mutations in
lung cancer involve the oncogenes MYCL (small cell carcinomas) and KRAS (adenocarcinomas); tumor suppressor genes:
TP53 and RB1

Tomorrow – complete respiratory – prepare mcq on respiratory this week – done

September 22, 2022


Dressed in a Surgeon’s gown A Physician Might Make Some Significant Progress
How to approach a question – organized response how to answer the question about all conditions-

Definition, incidence, sex geography, aetiology, pathogenesis, Macroscopic


pathology, microscopic pathology, symptoms, signs, prognosis

Lung Cancer – FA 684


When you come to lung cancer - I want you to divide into central and peripheral

Lung Cancer is leading cause of cancer death.


Presentation: cough, hemoptysis, bronchial obstruction, wheezing, “coin” lesion on x-ray
Metastasis – liver, adrenals, bone, brain
Complications – Superior vena cava syndrome, Pancoast tumor, Horner syndrome, Paraneoplastic endocrine, pleural
effusions
Squamous and small cell are Central – smoking

Mesothelioma – cancer associated with asbestos and not smoking – exudative pleura thickening FA 678

Adenocarcinoma - Peripheral
More commonly seen in women and nonsmokers. – most common primary cancer. Causes a peripheral gray-white
mass, and the tumor may develop in areas of parenchymal scarring (scar carcinoma). Microscopically - Aciinar, papillary,
mucinous, and solid.
• Atypical adenomatous hyperplasia—progresses to adenocarcinoma in situ (noninvasive well-
differentiated tumor <3 cm) and to minimally invasive tumor before progressing to invasive
adenocarcinoma.
• Xray – hazy infiltrates – Better prognosis

Squamous cell carcinoma (SCC) - Central


• strongly related to smoking and affects males more than females.
• Squamous cell carcinoma arises from bronchial epithelium
• metaplasia → dysplasia → carcinoma in situ → invasive carcinoma
• Usually centrally located.
• Microscopically, nests of squamous cells with intercellular bridges (desmosomes) and keratin pearls
(“squamous pearls”) & intercellular bridges

Stop here for Jan 25, 2023 – Jan 26, 2023

Small cell carcinoma - Central


• Strong association with smoking, and affects males more than females.
• Undifferentiated - Very aggressive, with rapid growth and early dissemination.
• Associated with paraneoplastic syndromes – Cushing’s, SIADH (inappropriate ADH), Lambert Eaton myasthenic,
encephalitis
• Gross examination demonstrates central, gray-white masses.
• Microscopic examination shows small round or polygonal cells in clusters - Electron microscopy shows
cytoplasmic dense-core neurosecretory granules., Kulchitsky cells, Chromogrannin A
• Treatment - chemotherapy and radiation. NOT by surgery
• Prognosis is poor

Squamous cell carcinoma vs small cell carcinoma. The squamous cell - well differentiated, somewhat polygonal, with
abundant, eosinophilic cytoplasm (intracytoplasmic keratinization), intercellular bridging, and scattered keratin pearls.
Small cell carcinoma - smallish cells, no apparent cytoplasm and a “salt-and-pepper” chromatin pattern.

Large cell carcinoma - Periperal


• Large anaplastic cells without evidence of differentiation, Intrathoracic spread of lung cancer
• Lymph nodes - hilar, bronchial, tracheal, and mediastinal;
• Pleura (adenocarcinoma)
• Lung apex causing Horner syndrome (Pancoast tumor). Same side as cancer px have Horner -
• Horner Syndrome causes ipsilateral: Ptosis, Miosis, Anhidrosis, Enophthalmos
• Obstruction of the superior vena cava - superior vena cava syndrome, characterized by distended head and
neck veins, plethora, and facial and upper arm edema.
Esophageal obstruction → dysphagia.
• Phrenic Nerve Impingement – Recurrent laryngeal nerve involvement - hoarseness, while phrenic nerve
damage causes diaphragmatic paralysis.
• Extrathoracic sites of metastasis include adrenal (>50%), liver, brain, and bone.
• Histology Pleomorphic giant cells
• Poor prognosis

Case history – persistent hiccups – diaphragmatic – just diaphragm reacting – phrenic nerve stimulation – should also
consider cancer

Bronchial carcinoids – Central or Peripheal


Rare, slow growing, neuroendocrine pulmonary neoplasm, metastasis rare
• Younger age group (mean age 40 years)
• Symptoms – flushing, diarrhea, wheezing
• Polypoid intrabronchial mass or plaque
• Characterized on light microscopy by small, round, uniform cells growing in nests (organoid pattern).
• Electron microscopy - cytoplasmic dense-core neurosecretory granules.
• Carcinoid tumors → serotonin
• Atypical carcinoid is more aggressive than typical carcinoid – Excellent prognosis

Metastatic carcinoma - Most common malignant neoplasm in the lung. Multiple, bilateral, scattered nodules
Common primary sites include breast, stomach, pancreas, and colon.

Hamartomas Occur more commonly in middle-aged adults but also occur in children. Appear as coin lesions on
chest x-ray. Microscopically, they are comprised of nonencapsulated fibromyxoid tissue. Carney triad is the finding of a
hamartoma with a predominantly cartilaginous component (pulmonary chondroma), an extra-adrenal paraganglioma
and a gastric gastrointestinal stromal tumor

In question stem with coin lesion → think Hermatoma.

Break –

Paraneoplastic syndromes - a group of rare disorders that are triggered by an abnormal immune system
response to a cancerous tumor known as a "neoplasm." Paraneoplastic syndromes are thought to happen when cancer-
fighting antibodies or white blood cells (known as T cells) mistakenly attack normal cells in the nervous system.

Endocrine/metabolic syndromes – various paraneoplastic syndromes arise in setting of lung cancer

Cushing syndrome secondary to ACTH production SIADH secondary to ADH production

(Syndrome of Inappropriate Antidiuretic Hormone Secretion)


body to retain too much water and commonly leads to hyponatremia,
which is low levels of sodium in your blood.
Hypercalcemia secondary to PTH (squamous cell carcinoma). Eaton-Lambert syndrome – anti bodies Ca on
pre synaptic terminals. In Myathevia gravis you have antip boides against the post → weakness in px.

Acanthosis nigricans - hyperpigmentation of the


skin, a paraneoplastic syndrome, defined as a condition
that arises in association with a malignancy elsewhere
in the body but without malignant nature per se 1. It's a
rare dermatopathy that occurs in men and women over
age 40, without racial predilection or known familial
association.

Hypertrophic pulmonary osteoarthropathy is characterized by periosteal new bone formation with clubbing and
arthritis.

Laryngeal Cancer
Laryngeal (voice box) squamous cell carcinoma causes hoarseness, difficulty swallowing, pain, hemoptysis, and
eventual can lead to respiratory compromise.
Accounts for .8% of all cancer – slightly more in males. Poor prognosis
Risk factors include smoking, alcohol, and frequent cord irritation (professional singing or lecturing).
Complications include direct extension, metastases, and infection.
Disease of the Pleural Cavity
Pleural effusion - accumulation of fluid in the pleural cavity. Blood or pus – if pus→
Empyema - pus in pleural space.
Chylothorax - chylous fluid in the pleural space

Pneumothorax - air in the pleural cavity. → Due to traumatic penetrating chest wall injuries or spontaneous rupture of
apical blebs in typically tall young adults (spontaneous pneumothorax).

Tension pneumothorax - life-threatening shift of thoracic organs across midline occurs.

Hemothorax - blood in the pleural cavity. Due to → Trauma , Hypotension and shift of the trachea to the unaffected
side.

Chylothorax is lymphatic fluid in the pleural cavity. Malignancy is a common cause.

Before I move on – want to share CT angiograph scan – thoracic cavity → want short present on Thoracic Outlet
syndrome – Peace
Thoracic outlet syndrome (TOS) is a group of disorders that occur when blood vessels or nerves in the space
between your collarbone and your first rib (thoracic outlet) are compressed. This can cause shoulder and neck
pain and numbness in your fingers.

Only 1 arm has good contrast – read up on this

Respiratory Pharmacology
Inhaled β2-Adrenergic Selective Agonists
• Short acting, such as albuterol or levalbuterol, or long acting, such as salmeterol or formoterol.
• Act on β2 receptors of intracellular adenyl cyclase, the enzyme that catalyzes the conversion of
adenosine triphosphate (ATP) to cyclic adenosine monophosphate (AMP).Results in increased cyclic AMP
levels in bronchial smooth muscle, causing bronchial smooth muscle relaxation.
• Also thought to inhibit mast cell release of immediate hypersensitivity mediators.
Clinical Uses: Alleviate acute bronchospasm in acute asthma exacerbations (most common); as prophylaxis
against exercise-induced asthma; to alleviate bronchospasm in COPD;
• during general anesthesia; in NRDS or RSV infection; and as rapid treatment of critical hyperkalemia
(drive K+ into cells).
• Side Effects: tachycardia and palpitations (caused by some β1 effects, although
• levalbuterol is more β2 selective than albuterol and causes less tachycardia), tremors, anxiety,
and hypokalemia

Muscarinic Antagonists
• Ipratropium (short acting) and tiotropium (long acting).
• Antagonize acetylcholine at muscarinic receptors, prevent bronchoconstriction by preventing increase in
cyclic guanosine monophosphate (cGMP)
• Reduce secretions from mucous glands.
• Clinical Uses: COPD maintenance therapy, asthma exacerbations.
• Side Effects: Nasal mucosa dryness, xerostomia, bronchitis, and sinusitis.

Phosphodiesterase-4 Inhibitors
• Roflumilast.
• Selectively inhibit PDE4 leading to accumulation of cyclic AMP (cAMP) within inflammatory and structural cells.
• Clinical Uses: They are used in the treatment of COPD.
• Side Effects: Headache, diarrhea, and weight loss.

Methylxanthines - theophylline.
• Cause bronchodilation through inhibition of phosphodiesterase, the enzyme that breaks down cyclic AMP, thus
increasing cyclic AMP levels, Also thought to increase contraction force of diaphragm by increasing Ca2+ uptake
into the muscle.
• Clinical Uses: Asthma controller medication and treatment for COPD.
• Side Effects: Tremor, irritability, nausea, vomiting, and tachyarrhythmia.
• Have a narrow therapeutic index
• Cromolyn
• Mast cell stabilizer (blocks histamine release) and anti-inflammatory agent.
• Prevents bronchoconstriction in response to inhaled antigens.
• Clinical Uses: Asthma controller medication and to treat allergic rhinitis.
• Side Effects: Cromolyn has minimal toxicity.
• Omalizumab
• Anti-IgE monoclonal antibody.
• Clinical Uses: Uses include treatment of asthma that is poorly controlled with steroids or beta agonists.
• Side Effects: Omalizumab has minimal toxicity.

Leukotriene Inhibitors
Leukotrienes - mediators synthesized by leukocytes that promote bronchoconstriction and neutrophil chemotaxis.
Asthma
• Inhibition - an important target for asthma therapy.
• Montelukast, Zafirlukast
• Selectively antagonize leukotriene receptors.
• Clinical Uses: Asthma controller medication and are especially useful in triad asthma (asthma induced
by aspirin, NSAID use).
• Acute prevention of exercise-induced bronchoconstriction and for allergic rhinitis.
• Side Effects: Leukotriene inhibitors have minimal toxicity. Headache can be a side effect.
Zileuton Selectively inhibits 5-lipoxygenase, the enzyme that catalyzes the formation of leukotrienes from arachidonic
acid. Used as an asthma controller medication. Side Effects: This agent has minimal toxicity

Corticosteroids
Inhaled: Beclomethasone, budesonide, fluticasone, mometasone
Oral: Prednisone and dexamethasone
• Corticosteroids exert antiinflammatory effects by inhibiting inflammatory cells and production of
inflammatory mediators.
• Clinical Uses: Inhaled corticosteroids are first-line asthma controller medications; oral corticosteroids
are used to treat asthma exacerbations unresponsive to other measures, as well as other inflammatory
conditions.
Side Effects Inhaled: Minimal - systemic absorption and therefore minimal toxicity.
• Oral: Asthma –also →Osteoporosis, immunosuppression, hypertension, Cushing syndrome,
hyperglycemia, cataracts, glaucoma, fluid retention, avascular necrosis of femoral head, depression, mania,
adrenal insufficiency (on cessation). NEVER should be stopped abruptly -
Also beta – blockers, depressants, SSRI – long period -

Expectorants
Guaifenesin - Reduces the viscosity of tracheal and bronchial secretions.
• Clinical Uses: It is used as an expectorant in over-the-counter medications (e.g., Robitussin Mucinex) and
in intensive care units to help patients on mechanical ventilation clear secretions.
• Side Effects: Guaifenesin has minimal toxicity.

N-Acetylcysteine - cleaves disulfide bonds in mucous glycoproteins, thus loosening thick sputum. It functions as a
glutathione donor in acetaminophen overdose.
• Clinical Uses: It is a mucolytic used for cystic fibrosis, tracheostomy care, or any respiratory condition
with abnormal or inspissated mucous secretions and in acetaminophen overdose.
• Side Effects: N-Acetylcysteine has minimal toxicity.

Jan 26 – 9:30 am – GI next

End of respiratory system – begin GI tomorrow –break – begin with questions mcqs next

September 23, 2022

Gastrointestinal Tract
Anatomy -- Abdominal cavity – is protected by abdominal wall. Layer are very important in GI anatomy. Ex. Surgery –
need to know layers and what incising across.

Anterior-lateral Abdominal Wall Layers lateral or midline – they are different


Skin → Superficial fascia
– Camper (fatty)
– Scarpa (fibrous)
External oblique
Internal oblique
Transversus abdominis
Transversalis fascia
Extraperitoneal connective tissue
Parietal peritoneum
Midline Incision Skin → Camper fascia → Scarpa fascia → linea alba (aponeuroses of rectus sheaths) → transversalis
fascia → extraperitoneal fascia → parietal peritoneum.

Internal oblique, while ext – hands in pocket – crosses externally and laterally – d

Nerve Supply: Ventral primary rami of the lower 6 thoracic spinal nerves (includes the subcostal nerve),
Iliohypogastric Ilioinguinal branches
Arterial Supply:
Superior epigastric branch of the internal thoracic artery,
Inferior epigastric artery
Deep circumflex iliac branches of the external iliac artery.
Venous drainage
Superficial epigastric,
Lateral thoracic veins superiorly
Great saphenous vein inferiorly.
Lymph drainage
Axillary nodes superiorly
Superficial inguinal nodes inferiorly

form anastomosis

Clinically – inf Hasselback triangle(or inguinal triangle) – where you have abdominal; structure – hernia – forms
boundary of this
INGUINAL REGION AND CANAL
Oblique passageway (approximately 4 cm long) in the lower aspect of the anterior abdominal wall running parallel and
superior to the medial half of the inguinal ligament.
Site of Inguinal Hernias
Entrance - Deep inguinal ring; lateral to inferior epigastric vessels; superior to the midpoint of the inguinal
ligament.(1cm)
Superficial inguinal ring is the medial opening of the canal supero-lateral to the pubic tubercle.

Contents - Female Inguinal Canal - Round ligament, Ilioinguinal nerve (L1)


Male Inguinal Canal - Ilioinguinal nerve (L1), The spermatic cord

2 rings – deep – lateral to epigast and sup and can palpate – when you palpate the ant and feel pubic - midpoint of ant
inginal ligalemtn 1 cm superior to that midpoint → dep iguinal ring

Round ligament – hold uterus in place and connect to vagna wall –

Somatic cord – testical – dctus deferens, aut nerves, lymphatic – penis and scrotum - > lymph – in testes and y cancer
metatst – aortic lymph nodes

The Spermatic Cord – Contents - Testicular artery, Pampiniform venous plexus, Vas deferens (ductus deferens) and its
artery, Autonomic nerves, Lymphatics - Cancers of the penis and scrotum metastasize to the superficial inguinal lymph
nodes, and testicular cancer metastasize to the aortic (lumbar) nodes

Layers of the Spermatic Cord - External spermatic fascia is formed by the aponeuroses of the external abdominal
oblique muscle at the superficial ring. Middle or cremasteric muscle and fascia are formed by fibers of the internal
abdominal oblique. The cremasteric muscle elevates the testis and helps regulate the thermal environment of the testis.
Internal spermatic fascia is formed by the transversalis fascia at the deep ring.
Cremastic muscle – control temp

Inguinal canal – roof – cram – look at anatomy of GI tact – roof – transvers and muscle originates from iliac crest, while
form arch over somatic cord and that become s roof. Floor ext oblique – ext oblique muscles travel “in your pocket” by
time get to groin, they form the lower most fibers and cure inward and so that makes the ext

Boundaries of the Inguinal Canal - FA 369


Roof - fibers of the internal abdominal oblique and the transversus abdominis muscles arching over the spermatic
cord.
Anterior wall is formed by aponeurosis of the external abdominal oblique; and the internal abdominal oblique muscle
laterally
Floor - inguinal ligament & lacunar ligament at the medial end
Posterior wall- divided into lateral and medial areas:
Lateral area is formed by the transversalis fascia and represents the weak area of the posterior wall.
Medial area by fused aponeurotic fibers of the internal abdominal oblique and transversus abdominis muscles
(conjoint tendon).
Inferior epigastric artery and vein ascend the posterior wall just lateral to the weak area and just medial to the deep
ring
Posterior wall- hernia in lateral area

If direct or indirect hernia

Testes - Develops from the mesoderm of the urogenital ridge. Descends during last trimester
• Guided by the fibrous gubernaculum.
• Failure to descend leads to crypto-orchidism
• Arterial Supply?
• Lymphatic drainage?
Evagination of the parietal peritoneum and the peritoneal cavity extends into the inguinal canal called
the processus vaginalis
• Tunica vaginalis – remnant of processus vaginalis that surrounds the testis
• Persistent process vaginalis often results in a congenital indirect inguinal hernia.
• Collection of serous fluid in the tunica vaginalis forms a hydrocele

What is main anatomy of testes – mesoderm – last trimester – should pediatrics – check if descended – then observe –
by 3 months they descend. If fails, → must intervene

Peritoneal – carries part to obliterate, if not → persistent processes vaginalis - → hydrocele

Inguinal Hernia
Most common of the abdominal hernias - M>F Superior to the inguinal ligament.

Types FA 370 – indirect and direct - stop here on January 26, 2023 – will continue tomorrow
January 27, 2023 – lost 950 mls of blood → stable – Hx – 3-4 days unable to sleep – mild sedation – CV system –
pericardial 150 ml can be fatal

Wants to share images – CXR


Indirect−Contents protrude through the deep inguinal ring lateral to the inferior epigastric vessels.
Direct−Contents protrude medial to the inferior epigastric vessels (in the inguinal [Hesselbach’s] triangle).−rupture
through the posterior wall of the inguinal canal

Hesselbach’s triangle−Lateral border: inferior epigastric vessels−Medial border: rectus abdominis muscle −Inferior
border: inguinal ligament

EMBRYOLOGY - The primitive gut tube - the foregut, midgut, and hindgut → each supplied by a specific artery and
autonomic nerves.

Foregut−Artery: celiac−Referred pain to the epigastrium−Parasympathetic innervation: vagus nerves−Sympathetic


innervation:- Preganglionics: thoracic splanchnic nerves, T5−T9 -Postganglionic cell bodies: celiac ganglion

Shared with the dermato with epigastrum – that is why pain is where it is.

What are foregut derivatives → Foregut derivatives: Esophagus, Stomach, Duodenum (first and second parts),
Liver, Pancreas, Biliary apparatus, Gallbladder

Midgut−Artery: superior mesenteric−Parasympathetic innervation: vagus nerves−Sympathetic innervation: -


Preganglionics: thoracic splanchnic nerves, T9−T12 -Postganglionic cell bodies: superior mesenteric ganglion

→Derivatives: Duodenum (second, third, and fourth parts), Jejunum, Ileum, Cecum, Appendix, Ascending colon,
Transverse colon (proximal two-thirds and other 1/3 from hind gut – from endoderm from certain point – boundary is
rectum line endo to ecto line))

Why look at these – when px with abdominal pain – over right side -lower mid area, right flank – when such – what
structures are in that region → mid gut structures – that is why – clinically can separate

Hind gut – Artery: inferior mesenteric−Parasympathetic innervation: pelvic splanchnic nerves−Sympathetic innervation:
- Preganglionics: lumbar splanchnic nerves, L1−L2 -Postganglionic cell bodies: inferior mesenteric ganglion

→Derivatives: Transverse colon (distal third—splenic flexure), Descending colon, Sigmoid colon, Rectum, Anal canal
(above pectinate line)

Rotation and development – abdominal foregut rotates 90 deg clockwise, while midgut 270 deg counter clock
around axis SMA (superior mesenteric artery)

Peritontoneum – helps ward off infection → Divided into 2 layers: parietal and visceral -Intraperitoneal versus
Retroperitoneal Organs - Major Intraperitoneal Organs: Stomach, Liver and gallbladder, Spleen, Duodenum, 1st part,
Tail of pancreas, Jejunum, Ileum, Appendix, Transverse colon, Sigmoid colon Secondary Intraperitoneal Organs (Lost a
mesentry during development): Duodenum, 2nd and 3rd parts, Head, neck, and body of pancreas, Ascending colon,
Descending colon, Upper rectum

Retroperitoneal Organs: (not covered) Kidneys, Adrenal glands, Ureters, Aorta, IVC, Lower rectum, Anal canal
January 30, 2023 am

GASTROINTESTINAL HISTOLOGY - GI tract walls are composed of 4 layers →mucosa, submucosa,


muscularis externa, and serosa. Mucosa – Innermost layer and further divided into → Epithelium (which comes in
direct contact with food), then Lamina Propria - Blind-ended lymphatic vessels (lacteals); GALT (gut-associated
lymphoid tissue), responsible for IgA production → protection of GI tract Muscularis mucosa - innermost edge of the
mucosa−motility to esophagus to the mucosa and facilitates discharge of secretions from glands.

Esophagus - Nonkeratinized stratified squamous epithelium. Upper 1/3, striated muscle; middle and lower 2/3
smooth muscle, with some overlap at the transition

Stomach - Gastric glands, Parietal cells are eosinophilic, chief cells are basophilic.

Duodenum - Villi and microvilli absorptive surface. Brunner glands (bicarbonate-secreting cells of submucosa) and
crypts of Lieberkühn (contain stem cells that replace enterocytes/goblet cells and Paneth cells that secrete defensins,
lysozyme, and TNF).

Jejunum - Villi, crypts of Lieberkühn, and plicae circulares (also present in distal duodenum).

Ileum -Villi, Peyer patches (arrow in D; lymphoid aggregates in lamina propria, submucosa), plicae circulares
(proximal ileum), and crypts of Lieberkühn. Largest number of goblet cells in the small intestine.

Colon - Crypts of Lieberkühn with abundant goblet cells, but no villi. Most absorption occurs here

Submucosa−loose areolar connective tissue that attaches the mucosa to the muscularis externa and houses the
larger blood vessels and mucous-secreting glands. – like the skeletal system for the gut

Muscularis Externa−Comprised of 2 layers of muscle: an inner circular and an outer longitudinal. −Controls the
lumen size and is responsible for peristalsis.

Serosa−Composed of a mesothelium (a thin epithelium lining the thoracic and abdominal cavities) and loose
connective tissue. −Serosa surrounds each intestinal loop and then doubles to form the mesentery within which run
blood and lymphatic vessels. – when malignancy – spreads easily -ex esophageal cancer are rapidly aggressive, and no
serosa on the esophagus and px not aware and no serosa to help constrict. But by the time px is aware, the tumor is at
stage 4

INNERVATION Intrinsic: Meissner and Auerbach’s Plexus Extrinsic: Sympathetic & Parasympathetic
Innervation−Sympathetic (NE) (fight or flight) →- ↓ motility, ↓ secretions, ↑ constriction of sphincters, to prevent
any digestion activity−Parasympathetic (Ach, GRP, VIP) – see increased activity ↑ motility, ↑ secretions, ↓
constriction of sphincters, G cell ↑ gastrin GI Smooth Muscle is connected by gap junctions.
Eat meal and go to restroom – food → para is activated and so → empty of gut tube
GI – role of peristalsis – difficult to have muscle contract and propel food down and other sections – gap junctions– if
you have continuous connection between muscle fibers – it would keep occurring, Pacemaker of GI

GI Physiology
Endocrine control : Secretin −S cells lining duodenum−Stimulated by Acid entering duodenum−Inhibits
stomach motility and secretion−Stimulates fluid secretion (HCO3–)

CCK −Secreted by I Cells lining the duodenum−Stimulated by Fat and amino acids entering duodenum−Inhibits
emptying of stomach −Stimulates enzyme secretion in the pancreas−Stimulates contraction of the
gallbladder−Relaxation of sphincter of (Oddi)

Gastrin G cells of stomach, Antrum & Duodenum−Release stimulated by Stomach distension−Antrum –


Stimulated by Parasym (GRP) Peptides−Duodenum - Stomach acid inhibits gastrin release. – when too much acid,
inhibits gastrin release

GIP/GLP−Stimulated by K cells of the Duodenum −Stimulated by Fat, CHO and amino acids−Inhibits Stomach
motility−Increases insulin & Decreases glucagon

Vasoactive intestinal peptide (VIP)−Secreted by GI mucosal neurons−Stimulated by Vagal


parasympathetics−Action: ↑ Intestinal and pancreatic secretion of fluids and electrolytes−VIPomas cause profuse
watery diarrhea, facial flushing−Associated with MEN Type I

Somatostatin −Secreted by Pancreatic delta cells−Stimulated by Meals (fat, protein, and carbohydrates)Actions:
↓GI secretion, ↓GI hormones, ↓GI motility, ↓GH, TSH, Splanchnic vasoconstrictor (constricts blood vessels in GI tract
– and consider as a Brakes of the GI and endocrine systems; Octreotide—

GIP gastrin like peptide – by K cells – 3 p – pituitary , pancreatic –

NB -Somatostatin – decrease all hormones – very potent vaso constrictor – one drugs in px with bleeding

MOTILITY - Resting membrane potential −40 to −65 mV. Close to depolarization. Oscillation of membrane
potential is generated by interstitial cells (interstitial cells of Cajal) that act as pacemakers. Referred to as slow waves or
basic electrical rhythm Action potentials are generated by the opening of slow channels that allow the entry of both
sodium and calcium. The duodenum contracts the most often.

Motility – generally – slow waves Duodenum – most contracting part of the gut

Motor activity → Stretch (from food bolus and stretches the gut) produces a contractile response. Gap junctions
create an electrical syncytium within the smooth muscle. Slow waves create low level contractions, and action
potentials strengthen the contractions. Pacemaker activity from the interstitial cells creates the intrinsic motor activity.
Tonic contraction at sphincters act as valves. The gut is always contracting (just like the heart) however – when at rest
and not eating, the low level – basic slow waves, as soon as ingest something, they become stronger (Cajal cells) and
sphincters act as valves (like gate. Always contracting, and only open when need digestion)
Swallowing→ A reflex controlled from the brain stem. Efferent input is via the vagus nerve. Events during
swallowing:−Relaxation of upper esophageal skeletal muscle sphincter (UES)−Primary peristaltic wave−Relaxation of
lower esophageal smooth muscle sphincter (LES) via VIP.−Relaxation of proximal stomach (receptive relaxation)

-Chew – UES relaxes and once first wave – propels food down LES – relax via VIP – key to note – the para sympathetic
will activate bolus across gut tube

Gastric Motility Stimulation−Acetylcholine released in response to activation of parasympathetics−Local


distension Inhibition−Low pH of stomach contents inhibits the release of gastrin will decrease motility−Feedback
from duodenal release of hormones (CCK, secretin, and GIP) and also slow down gastrin. If take liquid vs carb like rice,
and another has steak has beef, and another cheese → Liquid fast, carb and next protein and fat

Now beyond GI motility – we also have to look at what we eat – which faster→ Liquids > CHO - carbs > protein > fat (> =
faster than)– take longer time. → Food in stomach – there is a valve system in place to regulated emptying → just a
reservoir acts to grind food →if see an endoscope – wall of stomach – rough rugae – tough muscle to grind food→
chyme →

Notes: The pyloris of the stomach acts as a sphincter to control the rate of stomach emptying CCK, GIP, and
secretin ↑degree of pyloric constriction and ↓slow stomach emptying → Pyloris – prevents food just passing, and
backflow – ask why is it that once we eat – food just flow. When we eat, the body parasympathetic , pancreas,
gallbladder, need time to be able to secrete the hormones that are required to digest food.

Notes: Condition in px that have gastrectomy – lost pylorus – once they eat – rapid inflow of food into the duodenum →
large conc of complex carbs – ex px had surgery due to gastric cancer ,and stomach removed. Eats bowl of rice → the
movement esophagus → duodenum → no grinding of food – duodenum is not prepared to received and necessary
enzymes and polysaccharides and disaccharides to break down excess carbs ingested.

Diabetic px → gastro peristalsis – slowed down – also depend on the health status of px

Small Intestinal Motility Rhythmic contractions in adjacent sections create segmentation contractions,
which are mixing movements.

Colon Motility−Segmentation contractions create bulges (haustrations) along the colon. Mass movements,
which are propulsive, are more prolonged than the peristaltic movements of the small intestine. Longer – push more
solid

Migrating Motor Complex (MMC) is a propulsive movement initiated during fasting. It begins in the stomach
and moves undigested material from the stomach and small intestine into the colon. MMC repeats every 90–120
minutes.

→ Motolin – if fast – whatever food not digested will be propelled down, and MMC is responsible
Salivary Secretions
Parotids, sub languid -> serous, mixed – parasympathetic – mainly isotonic, duct cells

Salivary Secretions−Parotid gland secretions are entirely serous (lack mucin). Submandibular and
Sublingual gland secretions are mixed mucus and serous. Control of the parasympathetic system, which promotes
secretion.−Electrolyte composition is isotonic and similar to interstitial fluid. Duct cells modify the initial acinar secretion

Composition Low in Na+, Cl– because of reabsorption High in K+, HCO3 because of secretion (pH = 8) Low
tonicity: Salivary fluid is hypotonic because of reabsorption of NaCl and impermeability of ducts to water. α-amylase
(ptyalin): secreted in the active form and begins the digestion of carbohydrates. Mucus, glycoprotein
Immunoglobulins and lysozymes

FA – 373 – chief cells, proteins can still be ingested

Gastric Secretions Highly viscous alkaline fluid (mucin plus bicarbonate) secreted by epithelial cells that
Parietal cells−secrete HCl and Intrinsic factor Chief
protect the stomach lining from the caustic action of Hcl.
Cells−Pepsinogen−Begins the digestion of protein but is not essential for life Mucous neck cells secrete the
protective mucus, HCO3 combination. → ECL, Parietal and G cell – note in left diagram→ H out K comes in

Pancreatic Secretions Exocrine tissue is organized into acini and ducts. Most of the control is via secretin and
CCK.Secretin: Action on the pancreas → is the release of fluid high in bicarbonate HCO3 and
Cholecystokinin (CCK): Action on the pancreas is the release of enzymes (amylases, lipases, proteases).

Digestion & Absorption 1) Carbohydrates: Only monosaccharides are capable of being absorbed by the
small intestine. (Glucose, galactose & fructose) Carbohydrate digestion begins with α-amylase in the saliva

2) Dietary lipids: Consist of cholesterol, triglycerides, and phospholipids. Digestion begins with lingual lipase and
gastric lipase hydrolyzing triglycerides → into free fatty acids and glycerol

3) Protein digestion begins in the stomach with chief cell secretion of pepsinogen, which is converted to pepsin in
the acidic environment.

Digestion and absorption – FA 373 - Lipids – cholesterol -absorbed in bowel,


September 26, 2022 – Jan 30, 2023 continued

Gastrin – what stimulates – alkaline foods, amino acids, pH <1.5, release when been on long term proton pump
inhibitors and H pylori – (it destroys acid secretion, there will be a -ve feedback in forming the G cells, decrease in acid
levels, so stimulates gastrin) ––gastrinomas tumor (Zollinger – Ellison)→ gastrinomas produce large amounts of the
hormone gastrin. Gastrin causes the stomach to produce too much acid, which leads to peptic ulcers.

Somatostatin – D cells, all hormones in GI tract – see action list – regulation – somatostatin – Octreotide – (a
medication used in the management and treatment of acromegaly and thyrotrophinomas. It is in the somatostatin
analog class of drugs ) suppresses hormones – bleeding, portal hypertension, gall bladder

Secretin – release by S cells – bicarbonate- bicarbonate increase from pancreas will neutralize gastic acid in
duodenum, without it – difficult for CCK release

Motilin – migrating moto complex – in prolonged fast – every 90 minutes will have intestines contract to propel
contents out. Motilin increase in fasting. Erythromycin used to stimulate. In diabetes – impairs nerves that supply GI to
peristalsis – so px will have constipation. T help TX – stimulate with something motilin

Vasoactive intestinal polypeptide – sphincter control – released by distention (vagal) – VIPoma tumor –
Nitric Oxide – achalasia – LES tone increase due to loss of nitrogen oxide secretion
Ghrelin – stimulate when hungry – eg fasting state, and decreased by food. – 5 minute Break
GI Pathology
ESOPHAGUS -Nonkeratinized stratified squamous epithelium−Skeletal muscle (upper 1/3)−Smooth muscle (lower
1/3) Langerhans cells - antigen-presenting cells—present in the epithelial lining – long tube – 40 cm long

Sialolithiasis - a benign condition where stones form in the salivary ducts. These stones are called calculi and are
mostly composed of calcium. They can develop in any of the salivary glands, (there are several)including the parotid,
submandibular, or sublingual - Dehydration, trauma, mumps cause this stone - infection

Salivary gland tumors - Most are benign → parotid gland (80-85%)– presents painless mass, facial paralysis can
occur as close to facial nerve

Pleomorphhic adenoma – most common


Mucoepidemoid carcinoma
Warthin tumor – benign cyst, smoking

Congenital and Mechanical Disorders


Tracheoesophageal fistula - Congenital connection between the esophagus and trachea −Often associated with
esophageal atresia.−Adults - occur secondary to malignancy, trauma, or iatrogenic causes.

Esophageal webs - web-like protrusions of the esophageal mucosa into the lumen−Typically present with
dysphagia. −Plummer-Vinson syndrome is a disease of middle-aged women characterized by esophageal webs, iron
deficiency anemia, and increased risk of carcinoma. −Schatzki rings are web-like narrowing at the gastroesophageal
junction
Achalasia - failure of the lower esophageal sphincter (LES) to relax with swallowing. Etiology is unknown in most
cases. In South America caused by infection- Chagas disease. Presentation is with progressive dysphagia. Dilated
proximal to the lower esophageal sphincter; barium swallow shows a “bird-beak” sign. Microscopically - a
loss of ganglion cells in the myenteric plexus. Treatment is LES balloon dilation or myotomy. Achalasia carries an
increased risk for esophageal carcinoma (squamous cell).

Chagas disease, a tropical parasitic disease common in South America−Trypanosoma cruzi. −Transmitted by reduviid or
“kissing” bugs. Chagas disease can cause:−Romaña’s sign (unilateral swelling of the eyelid) −Cardiomyopathy
−Megaesophagus and megacolon

Hematemesis and Esophageal Bleeding


Mallory-Weiss syndrome−Esophageal bleeding due to linear lacerations at the gastroesophageal junction from
severe prolonged vomiting;−Most common cause is acute alcohol ingestion and/or chronic alcoholism.−typically occur
acutely as a result of retching/vomiting −Esophageal rupture (Boerhaave syndrome- spontaneous rupture of the
esophagus that occurs during intense straining. It most typically occurs during an episode of forceful or repeated
vomiting) may result.

Esophageal varices are dilated submucosal veins in the lower third of the esophagus,−Secondary to portal
hypertension. The most common cause is cirrhosis. Presentation is asymptomatic−Massive hematemesis when the
varices are ruptured. −Complications include potentially fatal hemorrhage. −Treatment is generally band ligation (tie of
veins) , sclerotherapy (Sclerosant injected directly into the vein causes blood clots to form and stops the bleeding), or
balloon tamponade (temporarily stop bleeding is by inflating a balloon to put pressure on the varices for up to 24 hours).

Esophagitis
Gastroesophageal reflux disease (reflux esophagitis) (GERD)−Esophageal irritation and inflammation
due to reflux of gastric secretions into the esophagus. −Reflux typically presents with heartburn and regurgitation.
−Complications - bleeding, stricture, bronchospasm and asthma, and Barrett esophagus.

Barrett esophagus - metaplasia of the squamous esophageal mucosa columnar type (intestinal metaplasia).
−Chronic exposure to gastric secretions (→ change in epithelium) −DX Endoscopy - irregular gastroesophageal junction
with tongues of red granular mucosa extending up into the esophagus. −Increased risk for dysplasia and esophageal
adenocarcinoma.

Break – new set of slides…..January 30 8:55 am – now cancers….

Esophageal Carcinoma
Squamous cell carcinoma (SCC) -Most common type of esophageal cancer in the world.

Men > women - African Americans > Caucasians Typical age is age >50. Heavy smoking and alcohol use

Achalasia (hard to swallow) • Plummer-Vinson syndrome (classic triad of dysphagia, iron-deficiency anemia, and
esophageal webbing) • Tylosis ( an extremely rare, hereditary, autosomal dominant disorder characterized by
hyperkeratosis of the palms and soles, with thickening and fissuring of the skin & oral leukoplakia. This disorder is
associated with an extraordinarily high risk of developing esophageal cancer– SCC of esophagus is seen in 95% of those
affected) Esophageal Carcinoma also associated with • Lye ingestion - lye-based cleaner burn has been found to
complicate esophageal strictures and thus increase the risk of esophageal squamous cell carcinoma - Most of the
patients had accidentally swallowed liquid lye at 2-3 years of age. All 15 tumors were squamous cell carcinoma - Often
asymptomatic until late in the course. Progressive dysphagia, weight loss and anorexia, bleeding, hoarseness, and
cough. Diagnosis is by endoscopy with biopsy. Treatment is surgery -→ prognosis is poor.

Esophagus cancer

Adenocarcinoma of the esophagus - Caucasians > African Americans. - Arises in the distal esophagus.
Barrett metaplasia > dysplasia> invasive carcinoma - United States - adenocarcinoma and squamous cell carcinoma of
the esophagus occur with equal frequency. Prognosis is poor

Barrett's esophagus occurs when the cells in the lining of your esophagus are damaged from acid reflux and can
increase your risk (3-10%) of cancer of the esophagus.
Sept 26, 2023 - Assignment – project – submit Saturday after block one exam – Cancers that we learned in Cardiovascular, in Respiratory and GI. 3 cancers for each of
them – plot out a table one year survival – per cent – 2 years – for each of the cancers in cardiovascular , respiratory and GI – ex in CV – we learned the hemangiomas
and sarcomas – plot year one, up to year 5, Then lung – squamous, small cell of lung, adenoma of lung – then carcinoid tumor, → GI esophagus, squamous of
esophagus, stomach – gastric etc.

Stomach - Congenital Disorders


Pyloric stenosis: Congenital stenosis of the pylorus
Marked muscular hypertrophy of the pyloric sphincter -Male infants>females.

Associated with Turner and Edwards syndromes.

Presentation is the onset of regurgitation and vomiting in week 2 of life; waves of


peristalsis are visible on the abdomen and there is a palpable olive shape oval
abdominal mass.

Treatment: surgery

Congenital diaphragmatic hernia


Congenital defect in the diaphragm - herniation of the abdominal

organs into the thoracic cavity.

The stomach is the most commonly herniated organ due to

left-sided congenital diaphragmatic hernia.

Associated with intestinal malrotation - May be complicated by


significant lung hypoplasia.
Ménétrier disease - Hypertrophic Gastropathy - Rare disease of middle-aged men - Profound hyperplasia
of surface mucous cells, accompanied by glandular atrophy. Characterized by enlarged rugal folds in the body and
fundus. Patients experience decreased acid production, protein-losing enteropathy - increased risk of gastric cancer. Tx
– peptic ulcer drugs, high protein diet, Cetuximab, surgery

Acute Inflammation and Stress Ulcers


Acute hemorrhagic gastritis -Acute inflammation, erosion, and hemorrhage of the gastric mucosa, secondary to a
breakdown of the mucosal barrier and acid induced injury Causes - Chronic aspirin or NSAID use, alcohol use, smoking,
recent surgery, burns, ischemia, stress, uremia, and chemotherapy. Patients present with epigastric abdominal pain, or
with gastric hemorrhage, hematemesis, and melena

Gastric stress ulcers - multiple, small, round, superficial ulcers of the stomach and duodenum.
Causes - NSAID use - inhibit prostaglandins which form protective barrier over mucosa
of GI. Acid is always there, but absence of covering→ ulcer ex→ Severe stress, Sepsis,
Shock, Severe burn or trauma, Elevated intracranial pressure (Cushing ulcers- heard
injuires) - ICU patients have a high incidence of gastric stress ulcer. These ulcers
may be complicated by bleeding.

MOA – increase in vagus - increase acid release in the stomach – same with intercranial
pressure. ICU px – Cushing ulcer - Cushing's ulcer is a gastro-duodenal ulcer produced
by elevated intracranial pressure caused by an intracranial tumor, head injury or
other space-occupying lesion

Chronic Gastritis
Chronic gastritis - chronic inflammation of the gastric mucosa eventually leading to atrophy (chronic atrophic
gastritis).

Fundic type chronic gastritis - an autoimmune atrophic gastritis that involves the body and the fundus. Caused
by autoantibodies directed against parietal cells and/or intrinsic factor. - Loss of parietal cells, decreased acid secretion,
increased serum gastrin (G-cell hyperplasia), and pernicious anemia (megaloblastic anemia due to lack of intrinsic factor
and B12 malabsorption). Women > men. Grossly, - Loss of rugal folds in the body and fundus. - Microscopically,
mucosal atrophy is seen with loss of glands and parietal cells, chronic lymphoplasmacytic inflammation, and intestinal
metaplasia. Increased risk for gastric carcinoma.

Antral type chronic gastritis (also called Helicobacter pylori gastritis) is the most common form of
chronic gastritis in the United States. H. pylori organisms are curved, gram-negative rods which produce urease. -
Increases with age. Infection is also associated with duodenal/gastric peptic ulcer, and gastric carcinoma with intestinal
type histology.

Microscopically- organisms are visible in the mucous layer of the surface epithelium. - Other microscopic features
include foci of acute inflammation - chronic inflammation with lymphoid follicles, and intestinal metaplasia
Chronic peptic ulcer – 2 types
Peptic ulcers are ulcers of the distal stomach and proximal duodenum.
Predisposing factors include the following:

• Chronic NSAID and aspirin use • Steroid use • Smoking • H. pylori infection

Patients present with burning epigastric pain.

Diagnosis is by endoscopy with or without biopsy. Treatment is acid suppression (H2 blocker, proton pump inhibitor,
etc.) and eradication of H. pylori. Complications of peptic ulcer include hemorrhage, iron deficiency anemia,
penetration into adjacent organs, perforation (x-ray shows free air under the diaphragm), and pyloric obstruction.

Duodenal peptic ulcers are more common than gastric peptic ulcers.
Associations include the following: • H. pylori (~100%) • Increased gastric acid secretion • Increased rate of
gastric emptying • Blood group O • Multiple endocrine neoplasia (MEN) type I • Zollinger-Ellison syndrome •
Cirrhosis • Chronic obstructive pulmonary disease

Most duodenal peptic ulcers are located in the anterior wall of the proximal duodenum.

Like a wound – it can bleed – iron def, bleed or perforate → on x ray – free air – under
diaphragm – what does it look like - gas or air trapped within the peritoneal cavity, but
outside the lumen of the bowel. Pneumoperitoneum can be due to bowel perforation

Can be on either side – left or right – see area on right lung

Gastric peptic ulcers are associated with H. pylori (75%).


Located in the lesser curvature of the antrum. Grossly, they are small (<3 cm), sharply
demarcated (‘punched out’), solitary with round/oval shape, smooth borders, and radiating mucosal folds.

Gastric Carcinoma (Malignant Ulcer)


More common in Japan than in the United States - decreasing incidence in the United States (due to screening).

Risk factors:−Smoked fish and meats−Pickled vegetables−Nitrosamines−Benzpyrene−Reduced intake of fruits and


vegetables, H. pylori infection, chronic atrophic gastritis, smoking, blood type A, bacterial overgrowth in the stomach,
prior subtotal gastrectomy, and Ménétrier disease. Gastric carcinoma is often (90%) asymptomatic Symptoms: weight
loss and anorexia, Epigastric abdominal pain mimicking a peptic ulcer, early satiety, and occult bleeding with iron
deficiency anemia - Located in the lesser curvature of the antrum. Large (>3 cm) ulcers with heaped-up margins and a
necrotic ulcer base. Occur as a flat or polypoid mass.

Histological types• The intestinal type shows gland-forming adenocarcinoma • The diffuse type shows diffuse
infiltration of stomach by poorly differentiated tumor cells, numerous signet-ring cells (whose nuclei are displaced to the
periphery by intracellular mucin), and linitis plastica (thickened “leather bottle”–like stomach) gross appearance.
Metastasize to the left supraclavicular lymph node (Virchow sentinel node) and to the ovary (Krukenberg tumor).
Diagnosis is by endoscopy with biopsy; treatment is gastrectomy. Prognosis – poor Overall 5-year survival ~30%
GASTRIC LYMPHOMA - Marginal zone B-cell lymphoma and diffuse large B-cell lymphoma occur in the stomach.
February 1, 2023

SMALL AND LARGE INTESTINES


Volvulus - twisting of a segment of bowel on its vascular mesentery. Often
associated with congenital abnormalities such as intestinal malrotation. Common
locations include the sigmoid colon and small bowel. Complications include infarction
and peritonitis – infection setting in due to obstructed blood flow

Intussusception - telescoping of a proximal segment of the bowel into


the distal segment. Most common in infants and children.

Symptoms: Vomiting, abdominal pain, passage of blood per rectum, and


lethargy; a sausage-shaped mass is often palpable in the right
hypochondrium.

In adults, intussusception may be caused by a mass or tumor.

The intussuscepted segment can become infarcted.

Incarcerated hernia - segment of bowel that is imprisoned within a hernia; the condition can become complicated
by intestinal obstruction and infarction.

Hirschsprung disease (or congenital aganglionic megacolon) is caused by


congenital absence of ganglion cells in the rectum and sigmoid colon, resulting
in intestinal obstruction. The condition affects males more than females, and
can be associated with Down syndrome.

Hirschsprung may present with delayed passage of meconium, or with


constipation, abdominal distention, and vomiting.

Grossly, the affected segment is narrowed, and there is dilation proximal to the
narrow segment (megacolon). Microscopically, there is an absence of ganglion
cells in Auerbach and Meissner plexuses, and the diagnosis is established when
rectal biopsy demonstrates the absence of ganglion cells. Treatment - resection of the affected segment

Malabsorption Syndromes
Celiac sprue (or gluten-sensitive enteropathy and nontropical sprue) – Celiac Disease
Cause: hypersensitivity to gluten (and gliadin), resulting in loss of small bowel villi and malabsorption. HLA-DQ2 and/or -
DQ8 are carried by most patients. Microscopic exam demonstrates a loss of villi, with increased intraepithelial
lymphocytes and increased plasma cells in the lamina propria. Presents in childhood with malabsorption. Symptoms
include abdominal distention, bloating, and flatulence, along with diarrhea, steatorrhea, and weight loss.

CELIAC→Dermatitis herpetiformis may occur age >20. In adults, celiac presents between decades 4-7. - (DH) is
a chronic, intensely itchy, blistering skin manifestation of gluten-sensitive enteropathy, commonly known as celiac
disease. DH is a rash that affects about 10 percent of people with celiac disease. DH is caused by the body's immune
system reacting to a protein called gluten in foods containing wheat, barley and rye

Treatment is dietary restriction of gluten. Increased risk of gastrointestinal cancer.

September 27, 2022 – Feb 1, 2023

Environmental enteropathy (previously known as tropical sprue)−Malabsorption disease of unknown


etiology (infection and/or nutritional deficiency). −Affects residents of low-income countries with poor sanitation.
−Biopsy shows blunting of villi and a lymphocytic infiltrate → poor absorption of food & nutrients.

Note – with Celiac you have ABSENCE and with Environmental Enteropathy you have BLUNTING of villi

Whipple disease - infectious disease involving many organs−small intestines, joints, lung, heart, liver, spleen, and
central nervous system. Typically affects Caucasian males age 30-50.−Infecting organism is Tropheryma whipplei.

Microscopically, the small bowel lamina propria is filled with macrophages stuffed with the PAS-positive, gram positive,
rod-shaped bacilli. −Present with malabsorption, weight loss, and diarrhea. Treatment is antibiotics- metronidazole

Inflammatory bowel disease (IBD) - spectrum of diseases

Crohn’s disease (CD) (or regional enteritis) & Ulcerative colitis (UC)
Colitis of indeterminate type. Etiology - Caucasians develop IBD more frequently than non-Caucasians.
Incidence of IBD is increasing. Age distribution varies with the disease: • CD has a bimodal distribution with peaks at
age 10–30 and 50–70 • UC peaks at age 20–30

IBD can present with episodes of bloody diarrhea or stools with mucus, crampy lower abdominal pain, or fever. Px has
had no recent travel – no diet changes – symptoms for past 5 years → could be either of CD or UC → now look out for
other features:

CD may present with malabsorption or extraintestinal manifestations→ fatty stool (steatorrhea) and fat-soluble
vitamins deficiency (ADEK). It may mimic appendicitis- CD can occur anywhere from mouth to anus, UC is only the
colon – secum to sigmoid colon. CD can affect appendix may cause perianal fistulas. UC does not present with
extraneous in other organ systems.

Diagnosis of IBD requires endoscopic biopsy and clinicopathologic correlation.

New studies indicate that risk of colorectal carcinoma (CRC) in CD and UC are equivalent for similar extent and duration
of disease; the risk of CRC is not as high as previous studies suggested.

Crohn’s Disease
cobblestone pattern
NOD2/CARD15 gene produces a bacterial lipopolysaccharide receptor in mucosal - Paneth cells, and mutations in this
gene affect activation of nuclear factor kappa B that is part of an innate immune response. CD patients generally have a
pANCA negative / ASCA positive serologic pattern

Ulcerative colitis Crohn’s Disease


Location colon & rectum all of GI tract

Ileum except for backwash-ileitis yes

Backflow from cecum to ileum with defective sphincter - ulcerative colitis (UC) may develop inflammation in the distal
ileum thought to be due to "backwash" of cecal contents

Colon left>right right>left

Rectum yes -

Distribution Diffuse - continuous Segmental -discontinuous

Ulcers superficial aphtoid ulcer, confluent deep linear

Pseudopolyps common -

Skip lesions - yes

Cobblestone pattern - yes

Deep fissures - yes

Fistulae - yes

Mucosal atrophy marked minimal

Wall thickness normal increased

Fat wrapping - yes

Ischemic bowel disease - decreased blood flow and ischemia of the bowel. Secondary to atherosclerosis with
thrombosis. Most common in older individuals. Typical presentation abdominal pain and bloody diarrhea. Disease
distribution tends to involve watershed areas (e.g., splenic flexure), and affected areas typically show hemorrhagic
infarction. Treatment surgical resection prognosis is poor, with >50% mortality
Hemorrhoids - tortuous, dilated anal submucosal veins caused by increased venous pressure. Risk factors -
constipation and prolonged straining during bowel movements, pregnancy, and cirrhosis. Complications painful
thrombosis and streaks of bright red blood on hard stool.

Angiodysplasia arteriovenous malformations of the intestines. −Occurs in the cecum and right colon. −Individuals
age >55 are most commonly affected, presenting with multiple episodes of rectal bleeding. −Associated with
OslerWeber-Rendu and CREST syndromes. −Treatment is surgical resection. Also note __>Heyde syndrome is a
multisystem disorder characterized by the triad of aortic stenosis (AS), gastrointestinal bleeding, and acquired von
Willebrand syndrome. Heyde syndrome is an uncommon condition that can lead to severe morbidity and mortality

Melanosis coli - common with laxative abuse; −causes black pigmentation of the colon due to the ingestion of the
laxative pigment by macrophages in the mucosal and submucosa. It can mimic colitis or malignancy.

Osler-Weber-Rendu Syndrome a.k.a. hereditary hemorrhagic telangiectasia−Autosomal


dominant −Telangiectasias (spider veins)of skin and mucous membranes−Common on lips, tongue,
and fingertips−May develop iron deficiency anemia

Pseudomembranous colitis (antibiotic-associated colitis) - acute colitis characterized by the formation


of inflammatory pseudomembranes in the intestines. −Usually caused by Clostridiodes difficile infection (often brought
on by a course of broad-spectrum antibiotics, especially clindamycin and ampicillin), −Caused by ischemic bowel
disease.−Gross examination shows yellow-tan mucosal membranes. Microscopic exam shows the pseudomembranes
are composed of an adherent layer of acute inflammatory cells, mucus and necrotic debris overlying sites of colonic
mucosal injury. → smell of stool, unique - gassy

Presentation is with diarrhea, fever, and abdominal cramps. Diagnosis detection


of C. difficile toxin in the stool. Treatment of clostridial pseudomembranous colitis is vancomycin or metronidazole

Appendicitis - commonly caused by obstruction of the appendix by a fecalith. Initial presentation: Periumbilical pain
that subsequently localizes to the right lower quadrant. Symptoms Nausea, vomiting, and fever may also be present.
Lab studies: Elevated white blood cell count. Complication: Appendiceal rupture leading to peritonitis. Grossly, a
fibrinopurulent exudate may be seen on the appendiceal serosa; Microscopically, neutrophils are present within the
mucosa and muscular wall (muscularis propria) of the appendix.

Diverticula - Meckel diverticulum - a congenital small bowel diverticulum caused by persistence of a remnant
of the vitelline (omphalomesenteric) duct. - “rule of 2s” applies:−2% of the normal population−2 feet from the
ileocecal valve−Length 2 cm−Age ≤2 years at time of diagnosis

Most Meckel diverticula are asymptomatic but they may contain rests of ectopic gastric & Pancreatic mucosa and
present with intestinal bleeding.
Diverticulosis - Colonic diverticulosis - acquired outpouching of the bowel wall, characterized by herniation of
the mucosa and submucosa through the muscularis propria (pseudodiverticulum). It is extremely common in the United
States.−Incidence increases with age−Major risk factor is a low-fiber diet, which leads to increased intraluminal
pressure−Most common location is sigmoid colon. Asymptomatic – many. Symptoms: Constipation alternating with
diarrhea - Left lower quadrant abdominal cramping and discomfort, occult bleeding and an iron deficiency anemia, or
lower gastrointestinal tract hemorrhage – one of the most common causes. Complications: infection→Diverticulitis,
fistulas, and perforation with accompanying peritonitis.

Polyps - Hamartomatous polyps include nonfamilial juvenile polyps and polyps associated with a familial
(Peutz-Jeghers) syndrome.

Nonsyndromic polyps do not have malignant potential.


Hyperplastic polyps are the most common histologic type; Most often in the left colon and are usually <5 mm.
Although previously considered not to have malignant potential, newer studies suggest they are part of a group of
polyps with serrated histology and risk of progression to cancer.

Serrated polyps occur more often in the right colon.


Tubular and villous adenoma as have long been known to have malignant potential.
Microscopically - Cellular dysplasia and either pure tubular, pure villous or tubulovillous histology

Familial Syndromes
Familial adenomatous polyposis (FAP), also called adenomatous polyposis coli (APC), - autosomal dominant
mutation of the APC gene on chromosome 5q21. Develop thousands of colonic adenomatous polyps;
Diagnosis - >100 adenomatous polyps on endoscopy.
Complications: by age 10, have hundreds and by age 40 many polyps, virtually 100% will develop an invasive
adenocarcinoma and increased risks for developing duodenal adenocarcinoma and adenocarcinoma of the papilla of
Vater.

Gardner syndrome - autosomal dominant variant of familial adenomatous polyposis characterized by numerous
colonic adenomatous polyps, multiple osteomas, fibromatosis, and epidermal inclusion cysts.

Turcot syndrome is a rare variant of familial adenomatous polyposis characterized by numerous colonic
adenomatous polyps and central nervous system tumors (gliomas).

Hereditary nonpolyposis colorectal cancer (HNPCC), or Lynch syndrome - an autosomal dominant


mutation of a DNA nucleotide mismatch repair gene
Predisposes for colon cancer. Associated with an increased risk of cancer - endometrium and the ovary.
Peutz-Jeghers syndrome is an autosomal dominant condition characterized by multiple hamartomatous polyps
(primarily in the small intestine); melanin pigmentation of the oral mucosa; and increased risk of cancer at numerous
sites including the lung, pancreas, breast, and uterus

Neoplasia
Colonic adenocarcinoma - the third most common tumor in the United States,\
Risk factors include: • Dietary factors (low fiber, low fruits/vegetables and high in red meat and animal fat) • Colon
polyps (isolated adenomatous polyps, hereditary polyposis syndromes) • Other colon disease (Lynch syndrome,
ulcerative colitis, and Crohn’s disease) Diagnosis is established via endoscopy with biopsy

Important that you know:

Cancer genetics: Mutations of the APC gene cause activation of the Wnt pathway – what is this? Overexpression
of growth. leading β-catenin to translocate to the nucleus where it causes the overexpression of growth-promoting
genes. DNA mismatch repair causes microsatellite instability, which is another genetic carcinogenesis pathway.
Treatment - surgical resection and chemotherapy (for metastatic disease); CEA levels can be used to monitor for
disease recurrence. Screening is recommended for the general population beginning age 50. Current
Guidelines suggest: • Colonoscopy every 10 years or annual fecal occult blood test (FOBT), or• Combination of FOBT
(every 3 years) and sigmoidoscopy (every 5 years)
→If px has family hx – 10 years before age of relative you begin screen

Carcinoid tumors are neuroendocrine tumors that often produce serotonin.


Locations include the appendix (most common) and the terminal ileum. Metastasis to the liver may result in carcinoid
heart disease.

Carcinoid syndrome is characterized by diarrhea, cutaneous flushing, bronchospasm and wheezing, and fibrosis.
The diagnosis is substantiated by demonstrating elevated urinary 5-HIAA (5-hydroxyindoleacetic acid).

Gastrointestinal stromal tumor (GIST) is the most common sarcoma of the GI tract. Most cases have a KIT
mutation. The peak incidence is in decade 7. Treatment is resection and a tyrosine-kinase inhibitor

PANCREAS
CONGENITAL ANOMALIES OF THE PANCREAS - Pancreatic agenesis is incompatible with life.
Pancreatic divisum is a variant of pancreatic duct anatomy. Annular pancreas encircles the duodenum and
presents as obstruction. Ectopic pancreatic tissue can hemorrhage, become inflamed, or give rise to a
neuroendocrine tumor. These most often arise in the stomach, duodenum, or jejunum

INFLAMMATION OF THE PANCREAS


Acute pancreatitis−Gallstones−Alcohol (#1)−Hypercalcemia−Drugs−Shock−Infections−Trauma−Scorpion sting
→ Tityus trinitatis (Trinidad)
Cell injury (from above list) results in activation of pancreatic enzymes and enzymatic destruction of the pancreatic
parenchyma. Symptoms - stabbing epigastric abdominal pain radiating to the back. Severe acute pancreatitis can cause
shock. Lab studies - elevated serum amylase and lipase. Complications→ acute respiratory distress syndrome (ARDS-
ground glass appearance, damage to wall) – to treat the ARDS you need to treat the pancreatitis, disseminated
intravascular coagulation (DIC – clotting factor is released and then bleeding), pancreatic pseudocyst; pancreatic
calcifications, and hypocalcemia. 30% mortality rate. Macroscopy - focal hemorrhage and liquefication in the
pancreas, accompanied by chalky, white-yellow fat necrosis of adjacent adipose tissue. Microscopy - liquefactive
necrosis of the pancreatic parenchyma with acute inflammation and enzymatic fat necrosis. Necrosis of blood vessels
causes hemorrhage in the px. Grey Turner sign

Chronic pancreatitis - Irreversible changes in pancreatic function. Px with gallstones who does not have them
out can lead to:→ Chronic inflammation, atrophy, and fibrosis of the pancreas secondary to repeated bouts of
pancreatitis. Manifestations include abdominal pain, pancreatic insufficiency and malabsorption, pancreatic
calcifications, pseudocyst, and secondary diabetes mellitus (late complication). Common in middle-aged male
alcoholics. Pathology shows grossly firm, white, and fibrotic pancreas. Microscopically - extensive fibrosis with
parenchymal atrophy and chronic inflammation. Autoimmune pancreatitis can occur in association with IgG4-
associated fibrosing disorders; this variant responds to steroid therapy

PANCREATIC TUMORS
Insulinoma (β-cell tumor) (most common type of islet cell tumor)−Produces insulin−Can cause hypoglycemia,
sweating, hunger, confusion, and insulin coma−Surgical excision is curative

Gastrinoma (G-cell tumor)−Produces gastrin−Excess gastrin manifests as Zollinger-Ellison syndrome, which is


characterized by thick gastric folds, elevated serum gastrin, gastric hyperacidity, and intractable peptic
ulcers−Gastrinomas may arise outside the pancreas−May be associated with MEN I – pituitary, parathyroid, pancreas – 3
P’s

Glucagonoma (α-cell tumor)−Produces glucagon−Excess glucagon causes hyperglycemia (diabetes), anemia, and
skin rash

Somatostatinoma (δ-cell tumor)−Produces somatostatin−it is the BRAKES Excess somatostatin inhibits


insulin secretion, leading to diabetes−Can also inhibit gastrin secretion (leading to hypochlorhydria) and cholecystokinin
secretion (leading to gallstones and steatorrhea)−Prognosis is poor

VIPoma−Produces vasoactive intestinal peptide (VIP) −Excess VIP causes WDHA syndrome: watery diarrhea,
hypokalemia, and achlorhydria

Pancreatic carcinoma is the third most common cause of cancer death in the United States. −Most common ages
60-80 – Risk −Smoking is a risk factor. −Presents with only vague signs and symptoms until late in course.
retroperitoneal Signs and symptoms - abdominal pain, migratory thrombophlebitis, and obstructive jaundice. Tumor
may occur in the head (60%), body (15%), and tail (5%). Microscopically, the adenocarcinoma arises from the duct
epithelium. Tumor desmoplasia and perineural invasion are common. Tumor markers - CEA and CA19-9, but they are
not useful screening assays. Treatment surgical excision (Whipple procedure). Prognosis is very poor - with 5-year
survival only ~5%
Pancreatic cystic neoplasms: Serous neoplasms account for 25% of pancreatic cystic neoplasms; −most are
benign (cystadenomas) and the tumors carry a mutation of VHL.

Mucinous neoplasms: Mucinous cystic neoplasms are common in women and can harbor dysplasia or carcinoma;
distal pancreatectomy is curative in most cases. −Intraductal papillary mucinous neoplasms are common in men and
tend to arise in the head of the pancreas; - GNAS mutations are common and carcinoma may arise in the neoplasm.

Gallbladder and Biliary Tract


Gallbladder is a small, pear-shaped organ in your upper right abdomen. Your gallbladder stores and releases bile to help
your digestive system break down fats. The most common issue you may develop with your gallbladder is gallstones

Gallstones −Frequently asymptomatic but can cause biliary colic (right upper quadrant pain due to impacted stones).
−Diagnosis is by U/S. Complications include cholecystitis, choledocholithiasis (calculi within the biliary tract), biliary tract
obstruction, pancreatitis, and cholangitis.

Cholesterol stones - composed mostly of cholesterol monohydrate. Cholesterol, bile salts & phospholipids in
balance - Incidence increases with age. Risk factors - female gender, obesity, pregnancy, oral contraceptives, and
hormone replacement therapy. Native American Pima and Navajo Indians have an increased incidence of cholesterol
gallstones.

Pigmented bilirubinate stones are composed of calcium salts and unconjugated bilirubin. Risk factors are
chronic hemolytic anemias (sickle cell anemia – heme broken down), cirrhosis (liver fibrosis in ducts), bacterial infection,
and parasites (Ascaris or Clonorchis [Opisthorchis] sinensis)

INFLAMMATORY CONDITIONS OF GALL BLADDER


Acute cholecystitis−An acute inflammation of the gallbladder, usually caused by cystic duct obstruction by
gallstones. −Present with biliary colic, right upper quadrant tenderness on palpation, nausea and vomiting, low-grade
fever, and leukocytosis. −Complications include gangrene of the gallbladder, perforation and peritonitis, fistula
formation and gallstone ileus (small bowel obstruction by a large gallstone). Acute acalculous cholecystitis (not
stones) is associated with surgery, trauma, and sepsis.

Chronic cholecystitis - chronic inflammation of the gallbladder, −show stromal and mural lymphocytic and
plasmacytic infiltrates. −Macrophages and granulomas may also be present. The wall is thickened.

Ascending cholangitis - bacterial infection of the bile ducts ascending up to the liver. Associated with obstruction
of bile flow. Obstruction by Bile duct stones. Presents with biliary colic, jaundice, high fever, and chills. Organisms are
usually gram-negative enteric bacteria.

Cholesterolosis – cholesterol in wall of gall bladder - an accumulation of cholesterol-laden macrophages within


the mucosa of the gallbladder wall. Gross examination shows yellow speckling of the red-tan mucosa (“strawberry
gallbladder”). Microscopic examination shows lipid-laden macrophages within the lamina propria.
Hydrops of the gallbladder (mucocele) - chronic obstruction of the cystic duct leads to the resorption of the
normal gallbladder contents and enlargement of the gallbladder by the production of large amounts of clear fluid
(hydrops) or mucous secretions (mucocele)

BILIARY TRACT CANCER


Gallbladder cancer - asymptomatic until late in the course. Symptoms: cholecystitis, enlarged palpable
gallbladder, or biliary tract obstruction (uncommon). CXR - X-ray may show a calcified “porcelain gallbladder.”
Microscopically, the tissues show adenocarcinoma. Prognosis poor; 5-year survival rate is ~12%.

Bile duct cancer - Bile duct carcinoma is carcinoma of the extrahepatic bile ducts, while cholangiocarcinoma is
carcinoma of the intrahepatic bile ducts. Klatskin tumor - carcinoma of the bifurcation of the right and left hepatic
bile ducts. Risk factors:−Clonorchis (Opisthorchis) sinensis (liver fluke) in Asia and primary sclerosing cholangitis. Bile
duct cancer typically presents with biliary tract obstruction. −Microscopic examination shows adenocarcinoma arising
from the bile duct epithelium. Prognosis poor.

Adenocarcinoma of the ampulla of Vater may exhibit duodenal, biliary, or pancreatic epithelium. Patients
present with painless jaundice. The 5-year survival rate is <50% in spite of resection.

Liver
JAUNDICE – increase in levels of bilirubin. Clinical jaundice occurs with bilirubin levels >2–3 mg/dL. Presentation is
yellow skin (jaundice) and sclera (icterus). Causes - overproduction of bilirubin, defective hepatic bilirubin uptake,
defective conjugation, and defective excretion. Also due to: Increased red blood cell (RBC) turnover. RBCs are the major
source of bilirubin. Jaundice related to overproduction of bilirubin can be seen in hemolytic anemia and ineffective
erythropoiesis (thalassemia, megaloblastic anemia, etc.). Laboratory studies show increased unconjugated bilirubin.

Chronic hemolytic anemia patients often develop pigmented bilirubinate gallstones. – like sickle cell px

Most common cause of marked jaundice in the newborn is blood group incompatibility (most commonly ABO)
between mother and child, causing hemolytic disease of the newborn

Physiologic jaundice of the newborn - transient unconjugated hyperbilirubinemia due to the


immaturity of the liver. Risk factors include prematurity and hemolytic disease of the newborn (erythroblastosis
fetalis). Complication – kernicterus - bilirubin-induced neurological damage. Treatment - phototherapy. - Jaundice
also occurs in newborns who have infections.
Hereditary hyperbilirubinemias
Gilbert syndrome - common benign inherited disorder that causes unconjugated hyperbilirubinemia due to
bilirubin UDP-glucuronosyltransferase (UGT) deficiency. Kernicterus rarely occurs and the treatment is phenobarbital.

Crigler-Najjar syndrome causes unconjugated hyperbilirubinemia due to bilirubin glucuronosyltransferase (UGT)


absence or deficiency. Treatment for type 1 is gene replacement therapy and liver transplantation. For a milder type
2, phenobarbital is used.

Dubin-Johnson syndrome - benign autosomal recessive disorder characterized by decreased bilirubin excretion
due to a defect in the canalicular cationic transport protein.−Produces conjugated hyperbilirubinemia and a distinctive
black pigmentation of the liver, but has no clinical consequences.

Rotor syndrome - autosomal recessive conjugated hyperbilirubinemia that is similar to Dubin-Johnson syndrome,
but without liver pigmentation. There are no clinical consequences.

NB -Enzyme deficiencies are always Autosomal recessive → but structural protein deficiencies are Autosomal dominant

Primary biliary cirrhosis (PBC)


Chronic liver disease that is characterized by inflammation and granulomatous destruction of
intrahepatic bile ducts. Females have 10 times the incidence of primary biliary cirrhosis compared to males; the
Peak incidence is age 40–50. Presentation includes obstructive jaundice and pruritus; xanthomas, xanthelasmas, and
elevated serum cholesterol; fatigue; and cirrhosis (late complication). Most patients also have autoimmune disease
(scleroderma, rheumatoid arthritis or systemic lupus erythematosus). Laboratory studies show elevated conjugated
bilirubin, elevated alkaline phosphatase, and elevated 5 ́-nucleotidase. Treatment with oral ursodeoxycholic acid slows
disease progression. Antimitochondrial autoantibodies (AMA) are present in >90% of cases. Microscopically,
lymphocytic and granulomatous inflammation involves interlobular bile ducts

Primary sclerosing cholangitis (PSC)- chronic liver disease characterized by segmental inflammation and
fibrosing destruction of intrahepatic and extrahepatic bile ducts. Exact etiologic mechanism is not known ?immunologic
basis. Male to female ratio is 2:1; peak age 20–40. -Most cases of PSC are associated with ulcerative colitis. What
conditions? The presentation is similar to PBC. Complications include biliary cirrhosis and cholangiocarcinoma.
Microscopically - periductal chronic inflammation with concentric fibrosis around bile ducts and segmental stenosis of
bile ducts. - Cholangiogram shows “beaded appearance” of bile ducts.
CIRRHOSIS - end-stage liver disease characterized by 1 disruption of the liver
architecture by 2 bands of fibrosis which 3 divide the liver into nodules of regenerating
liver parenchyma. Causes - alcohol, viral hepatitis, biliary tract disease,
hemochromatosis, cryptogenic/idiopathic, Wilson disease, and α-1-antitrypsin
deficiency.
Gross Pathology: Micronodular cirrhosis has nodules <3 mm, while macronodular
cirrhosis has nodules >3 mm; Also a mixed micronodular and macronodular cirrhosis can
also occur. Etiology may not be distinguished based on the appearance.

Sequela - an abnormal condition resulting from a previous disease: →portal hypertension, ascites,
splenomegaly/hypersplenism, esophageal varices, hemorrhoids, caput medusa, decreased detoxification, hepatic
encephalopathy, spider angiomata, palmar erythema, gynecomastia, decreased synthetic function, hepatorenal
syndrome and coagulopathy.

Px – history chronic alcohol – but vomit blood and ascites, had no hemorrhoids, no sweat breath, no edema,

September 28, 2022 – Hurricane Ian

Viral Hepatitis

7 in number A to G – vary degree - Have to know type- associated virus, and incubation period and mode of transmission

Eg Hep A – RNA, family Picomnaviridae, liver, symptoms, 2-6 week, fecal oral route – ingest food, Dx IgM

Hep B – DNA virus, severe liver, chronic disease, 3-26 weeks (6 months) parenteral (IV drug abuse) or sex, IgM

Hep C Flaviviridae, see B more chronic, 2-33 weeks (2 weeks up to 8 months) IV, Dx PCR test of RNA

Hep D, Deltaviridae- RNA virus, on own does not cause infection of liver→ needs HBV to cause symptoms/damage to
liver (co-infection), high mortality, 6-26 weeks, IV, IgM – no D unless also B

Hep E, Callciviridae, pregnant females, 2-6 weeks, Ingestion fecal, Test IgM

Route B & C – rectal cancer, and which one acute liver – stop here today Feb 1 and tomorrow Feb 2 finish GI and do Beh
Science – Today February 2, 2023
Example – given HepB – results – how interpret → viral

HB core antibodies – antigens and antibodies

How can we tell this? If you have individual with no sex and drug abuse – 10-year-old – mother no history→ and no
vaccine. Blood and lab→ negative to everything → no antibodies either. If received Vac B, → antibodies Hep B surface
antigen.

Now infect with acute B – second one → see surface antigen +ve, DNA present, and for B if early at the IgM Ab are the
first to respond. But no immune response, how come they have antibodies to core present because first site core body
cell attacks, the IgM antibody attack, an If individual fights infection, able to control, kill the virus, all the signs of virus
eliminated , no HbB

Past infection – Hep B core antibody for someone with past infection.

Chronic Hep B – contain – how tell – virus is still present, surface antigen there, also, the HepB viral DNA and +/- e
antigen

How infectious is person? → in question stem – one of the only ways to tell → levels of levels antibody e antigens. If
high, they are highly contagious, present with clinical follow up and markers reveal – or have immigrant, in US have
vaccination. Presents with yellow of yes markers reveal – with jaundice already → +ve HBsAB (surface antiboides)

AMEBIC LIVER ABSCESS - tropical areas with poor sanitation. Entamoeba histolytica. Presentation - RUQ pain,
fever, and hepatic tenderness. Detection of a space-occupying liver lesion with positive serology is diagnostic.
Treatment is antibiotics. Drainage is rarely necessary

ALCOHOLIC LIVER DISEASE - progressive


Fatty change (steatosis) - reversible with abstinence.
Macroscopy: Enlarged, yellow, greasy liver. Microscopically the liver initially shows centrilobular macrovesicular
steatosis (reversible) that can eventually progress to fibrosis around the central vein (irreversible). Can progress to →

Alcoholic hepatitis - heavy drinking binge. - Asymptomatic to RUQ pain, hepatomegaly, jaundice, malaise,
anorexia, or even fulminant liver failure. Microscopically the liver shows hepatocyte swelling (ballooning) and necrosis,

Mallory bodies (cytokeratin intermediate filaments), neutrophils, fatty change, and eventual fibrosis around the central
vein. −The prognosis can be poor, 20% risk of death, and repeated episodes increase the risk of developing cirrhosis.

Alcoholic cirrhosis develops in 15% of alcoholics, and is typically a micronodular or Laennec cirrhosis.

METABOLIC LIVER DISEASE


Wilson disease (hepatolenticular degeneration) - genetic disorder of
copper metabolism resulting in the accumulation of toxic levels of copper in
various organs. Affects the liver (fatty change, chronic hepatitis, and
micronodular cirrhosis), cornea (Kayser-Fleischer rings [copper deposition in
Descemet’s membrane]), and brain (neurological and psychiatric
manifestations, movement disorder – Wilson affects the brain stem and tends
to result in Parkinson like symptoms). Diagnosis - decreased serum
ceruloplasmin levels, increased tissue copper levels (liver biopsy), and
increased urinary copper excretion. Treatment includes copper chelators (D-
penicillamine); liver transplantation is curative. Autosomal recessive, and the
WD gene (ATP7B on chromosome 13) codes for a hepatocyte canalicular
copper-transporting ATPase. Damage - leads to a decreased biliary excretion of
copper. Wilson disease Presents in children or adolescents with liver disease. –
Tx – can chelate→ liver transplant often needed

GI tract – diabetes – Pharmacology – Tx for diabetes –

Hemochromatosis - Increased levels of iron, leading to tissue injury. Hereditary (primary) hemochromatosis -
recessive disorder of the HFE gene on chromosome 6p. Most common mutation of the HFE gene is the C282Y mutation,
which increases small intestine duodenum, absorption of iron. Males > Females - Hemochromatosis - micronodular
cirrhosis and hepatocellular carcinoma (200 times the normal risk ratio); secondary diabetes mellitus; hyperpigmented
skin (“bronzing”); congestive heart failure and cardiac arrhythmias; and hypogonadism.

Diagnosis – elevated serum iron and ferritin or increased tissue iron levels (Prussian blue stain) on liver biopsy.
Treatment is phlebotomy.

bronzing diabetesdue to Fe – in liver and pancreas


α-1-antitrypsin deficiency - autosomal recessive disorder characterized
by production of defective α-1-antitrypsin (α1-AT), which accumulates in
hepatocytes and causes liver damage and low serum levels of α1-AT.

α1-AT is produced by the SERPINA1 gene (chromosome 14); >75 gene variants
are described. -Normal individuals are homozygous PiMM. Heterozygotes have
intermediate levels of the enzyme. Homozygous PiZZ have severe reductions
(10% of normal) in enzyme levels. - α-1-antitrypsin deficiency affects the liver
(micronodular cirrhosis and an increased risk of hepatocellular carcinoma) and
lungs (panacinar emphysema). Microscopically, PAS positive, eosinophilic
cytoplasmic globules are found in hepatocytes. Treatment includes smoking
abstinence/cessation to prevent emphysema; liver transplantation is curative.

Reye syndrome - young children with viral illness (varicella or influenza) treated
with aspirin. (except for Kawasaki – what else do you give to Kawasaki
px???immunoglobin (IVIG)) Mechanism is unknown; mitochondrial injury and
dysfunction play an important role. Reye - hepatic fatty change (microvesicular
steatosis) and cerebral edema/ encephalopathy. Complete recovery in 75% of
patients, but those that do not recover may experience permanent neurologic deficits.
Coma and death may result. Treatment is supportive.

Nonalcoholic fatty liver disease - lipids accumulating in hepatocytes that is not associated with heavy alcohol
use. Occurs equally in men and women and is strongly associated with obesity, hyperinsulinemia, insulin resistance, and
type 2 diabetes mellitus. Pathogenesis involves lipid accumulation in hepatocytes that can progress to steatohepatitis
(NASH—nonalcoholic steatohepatitis) and finally cirrhosis. Nonalcoholic fatty liver disease is a diagnosis of exclusion
(exclude other causes first).

HEMODYNAMIC LIVER DISEASES


Budd-Chiari syndrome (hepatic vein thrombosis) - occlusion of the
hepatic vein by a thrombus, often resulting in death. → Idiopathic, polycythemia
vera, pregnancy, oral contraceptives, paroxysmal nocturnal hemoglobinuria, or
hepatocellular carcinoma. - Presents with abdominal pain, hepatomegaly, ascites,
jaundice, splenomegaly, and in some cases, death. - Initial diagnostic test is
ultrasonography. Microscopically, the liver shows centrilobular congestion and
necrosis. In the chronic form, fibrosis develops. Treatment includes supportive care
and treatment of the underlying condition. -Some patients require lifelong
anticoagulation.

Chronic passive congestion of the liver refers to a “backup of blood” into the liver, usually due to right-sided
heart failure. Liver - nutmeg pattern of alternating dark (congested central areas) and light (portal tract areas) liver
parenchyma. Microscopically, the liver shows centrilobular congestion. Complications include centrilobular necrosis,
which is an ischemic necrosis of centrilobular hepatocytes. Long-standing congestion can lead to centrilobular fibrosis,
which can eventually become cardiac cirrhosis (sclerosis).
LIVER TUMORS
Hemangioma - most common primary tumor of the liver, Mostly affecting women. Benign vascular tumor that
typically forms a subcapsular, red, spongy mass. Asymptomatic and detected incidentally on CT or MRI. Resection is
rarely indicated, and liver biopsy carries the risk of bleeding.

Hepatocellular adenoma (HCA) - young women and is related to oral contraceptive use. Half of cases are
asymptomatic. Symptoms – abdominal pain or spontaneous intraperitoneal hemorrhage (25% of cases). Due to the
risk of transformation to HCC, resection is often recommended.

3 subtypes of HCA:−H-HCA is a solitary or multiple tan steatotic nodule with rare transformation into HCC - Mutation
of hepatocyte nuclear factor 1

HCA can resemble HCC histologically and transforms to HCC in some cases
Was named for the associated beta-catenin mutations I-HCA shows inflammatory infiltrates, sinusoidal dilatation, and
thick walled arteries. Acute inflammatory markers are elevated; malignant transformation occurs less frequently

Focal nodular hyperplasia - subcapsular lesion often discovered incidentally by the radiologist. Laboratory values
are generally normal. It is a nodular proliferation in response to a vascular anomaly. There is a central, stellate scar.

Hepatocellular carcinoma (HCC) - most common primary malignant tumor of the liver in adults. −The incidence
is higher in Asia and Japan than in the United States. −Risk factors include cirrhosis, hepatitis B and C viruses, alcohol,
aflatoxin B1. HCC has a tendency for hematogenous spread and invasion of portal and hepatic veins→ thrombosis. The
tumor marker is α-fetoprotein (AFP). The fibrolamellar variant affects younger age, has fibrous bands, and has a better
prognosis

NOTE - Carcinoma → always go to lymph, while sarcoma → thru blood → how to recall – C – and lymphatic ends in C,
but there are exceptions→ carcinoma – renal cell

Angiosarcoma is a rare, fatal tumor associated with exposure to vinyl chloride.


Hepatoblastoma is the most common hepatic malignancy in infants and children.−Lobectomy is the standard of
care. Histology shows immature precursor cells.

Metastatic tumors are the most common tumors found within the liver. Common primary sites include the colon,
breast, and lung. −Occur as multiple well-circumscribed masses

GIT Pharmacology
Antiemetics – Ondansetron -Ondansetron - serotonin 5-HT3 antagonist. −QT prolongation is a potential side
effect. Metoclopramide - dopamine antagonist,−Its muscarinic activity increases upper GI motility. −Also used as a
prokinetic agent in conditions such as diabetic gastroparesis.−Metoclopramide has an overlapping mechanism of action
with antipsychotics, so extrapyramidal side effects (e.g., dystonia, parkinsonism, irreversible tardive dyskinesia) may
occur. −Avoid in Small bowel Obstruction & Parkinsonism

Antacids - Proton Pump Inhibitors - directly inhibit H+-K+-ATPase on gastric parietal cells, thereby ↓ gastric H+
secretion and raising pH Long-term use has been associated with pneumonia and enteric (including C. difficile)
infections. Vit B12 deficiency

Histamine-2 (H2) blockers antagonize H2 receptors on parietal cells, causing decreased cAMP activation of H+,K+-
ATPase.

Cimetidine inhibits multiple cytochrome P-450 enzymes, leading to elevated levels of interacting medications.
−Warfarin, Phenytoin – Other noninteracting H2 blockers are generally preferred.

Weak bases neutralize gastric hydrochloric acid. Examples include sodium bicarbonate, calcium carbonate, and
aluminum hydroxide. They provide symptomatic relief of mild GERD.

Cytoprotective Agents
Bismuth Sucralfate−Bismuth sucralfate binds the ulcer base, protecting it against the acidic environment and digestive
enzymes.

Misoprostol is a prostaglandin E1 analog that promotes gastric secretion of mucus, bicarbonate, and gastrin. It is used
to prevent NSAID-induced ulcers. Prostaglandin E1 analog - used as a cervical ripening agent for induction of labor. Also
be used to maintain patent ductus arteriosus in congenital heart anomalies by the same mechanism of action.

Inflammatory Bowel Disease IBD Management


Sulfasalazine -Precursor to the NSAID 5-aminosalicylic acid (5-ASA). This compound is used in the treatment of IBD
because it is not converted to its active form until reaching the ileum. Poorly absorbed in the small bowel, so it can reach
sufficiently high concentrations to be locally anti-inflammatory without causing systemic side effects.

Glucocorticoids may be needed to manage IBD. If still unresponsive, immunomodulators such as 6-mercaptopurine,
azathioprine, and methotrexate should be considered. Anti–tumor necrosis factor (TNF) biologic therapy such as
adalimumab and infliximab is generally reserved for refractory cases

Statins−HMG-CoA reductase inhibitors, which interfere with hepatic cholesterol synthesis by inhibiting the
transformation of HMG-CoA into mevalonic acid (covered in Cardio vascular system), the rate-limiting step in
cholesterol synthesis. Increased LDL receptors lead to a decrease in serum LDL levels. Side effects are rare but include
hepatotoxicity and myositis, so ALT, AST, and creatine kinase levels are monitored during treatment.

Cholestyramine is a bile acid–binding resin that renders bile salts unabsorbable. Currently cholestyramine - used
for those with disease of the terminal ileum (e.g., Crohn disease). Bile salts that are not absorbed in the terminal ileum
act as an irritant in the colon. Side effects include GI upset and cholesterol gallstones. Cholestyramine interferes with
lipid absorption, it may interfere with the absorption of fat-soluble vitamins and medications.
Loperamide−decreases GI motility via opioid receptor–induced cholinergic inhibition of the enteric nervous system.
−Loperamide is an agonist at the opioid receptor. −Does not cross the blood–brain barrier, however, so it does not have
analgesic properties.

Octreotide is a synthetic somatostatin used in the treatment of esophageal varices because of its effects as a
splanchnic vasoconstrictor. Additionally, because of its inhibitory effects on the endocrine system, it is used in the
treatment of VIPoma, gastrinoma, glucagonoma, carcinoid syndrome, and acromegaly.

Orlistat - is a pancreatic lipase inhibitor that prevents the digestion of triglycerides into free fatty acids. It is used in
the treatment of obesity. Predictable side effect of steatorrhea

Feb 2 10:48 am – go to mcqs Dolapo

Break – end of GI tract – next

Behavioural Science
Stages of Development
Newborns – large bright objects, contrast, moving, curves, complex, facial → reflexes →
Moro – extend head 0-6 months (nuclei)

Grasp -0- 6 months – finger in palm (nuclei)


Rooting – turn mouth to cheek stimulus – 0- 6 months (trigeminal)

Trunk incurvation – withdraw when stroke ventral – 0 – 9 months (spinal cord)


Placing – step up when foot dorsum stimulated – 0-6months (central cortex)

Tonic neck – fencing posture – 0-6 months (vestibular nuclei)


Parachute – extend arms when falling – 6months – adult (vestibular)
Stranger anxiety – peak at 8 months – 1 year
Separation anxiety – 8 months to 2 years
Piaget’s Cognitive Development Theories
0-2 yrs – Sensimotor – senses – object permanence, stranger anxiety - 2 – 7 years – preoperational – intuitive, rather
than logic – pretend play, egocentrism - 7-11 – concrete operational – logic – conservation, math transformations -
12 – adult – formal operational – abstract reasoning and logic →moral reasoning

Erik Erikson Stages of development:


Freud’s Psychosexual stages – oral, anal, phallic, latency and genital stages:

Sexuality – Gender Identity is established by age 3


Tanner Stage of Development – Stage I to Stage V – from birth to mature
Stage Female Both Male

Breast Pubic Hair Genitalia

I pre-adolescent none childhood size

II breast bud sparse, straight enlargement of scrotum, testes

III areolar diam ↑darker, curling length and enlarge

IV contours course adult type length, breadth, scrotum darkens, teste enlarge

V mature extend to thighs adult

AGING - The leading causes of death for patients age >65 include:−Heart disease−Malignancy−Cerebrovascular
disease−Chronic lower respiratory disease

Medical care of the geriatric population includes preventive care, vaccinations, and screening

Physiologic – sexuality, sleep.

Kubler-Ross – stages of Grief – can occur in different order


Denial – refuse truth → Anger – place blame even on physician who can respond with anger → Bargaining –
attempt to postpone inevitable → Depression – feeling loss of control/hopelessness → diabetic more likely
to experience → amputation -→ Acceptance - resignation

Mourning and Loss in Adults – Grief – Depression – how to tell difference – in pleasure,
Grief Depression
Focus on Identifiable loss focus on self – specific loss not necessary

Fluctuating ability for pleasure can’t feel pleasure

Comfort from others isolation

Wide range emotions emotions get “stuck”

Some guilt and esteem ok lots of gilt, worthless

Death – want to be reunited death/suicide due to worthless feeling and inability to cope c pain

SUICIDE -10th leading cause of death in the United States. Men > women; - Women attempt suicide more often
(pills/poison) but men are more successful and Elderly are most successful and attempt less frequently. Adolescents
attempt more frequently.
Ethnic group with the highest suicide rate is Native Americans; within this group adolescents > elderly.
Firearms account for >50% of all suicides. -50% have seen a physician in the past month
Decreased levels of 5-HIAA (serotonin metabolite) are associated with aggression and suicide.
High risk factors for suicide include: Previous suicide attempt, Age, Gender, High socioeconomic status (SES),
Unemployed, Medical/psychiatric comorbidities, HopelessnessIsolation

Theories of Learning - Two main types of learning: classical and operant.


Stop here today – Hurricane Ian in progress – roof blows off…….

Lecture for Thursday September 29 and Friday September 30 were not recorded. This is from
Lecture notes
Classical Conditioning
Conditioned response after pairing UCS unconditional stimulus with CS conditioned stimulus

Example – px receives chemo (UCS) → nausea (UCR). The hospital alone -CS) now elicits nausea which is now CR
conditioned response

Operant Conditioning
Learning when behavior follows event – reinforcer increases response. +ve reinforcer – strengthens, -ve reinforcer
stimulus when removed strengthens prob of response

Continuous Reinforcement – every response followed by reinforcement → fast learning but fast extinction when stop

Intermittent – not every response – learning slower, but harder to extinguish

Punishment – stimuli decrease probability of response

Extinction – disappearance of response when no reinforcement

Behavioural Modification
Systemic Desensitization – forced exposure until fear extinguished

Aversive conditioning – aversive stimulus paired with stimulus that produces undesired behavior

Substance Use Disorders


Alcohol / tobacco – genetic link – dopamine increase wakefulness & desire for stimulus, serotonin reduces cravings

Fetal Alcohol Syndrome (FAS) – low nasal bridge, thin upper lip, micrognathia, minor ear abnormalities

Alcohol – TX – Thiamine, MVI, benzodiazepines cause limited decrease in REM and Stage 4

Opiates – Tx naloxone

Sleep Disorders
Non-rapid Eye Movement (NREM) – 3 stages

REM – awake brain in “paralyzed body”

Narcolepsy – decrease ability to control sleep-wake cycle

Sleep Apnea – loud snore – obstructive in upper or central from incorrect brain signals

Parasomnias – night terror, common in young boys, familial, no memory of event vs nightmares where have memory

Psychopharmacology
Schizophrenia / Hiccups, Tourette, bipolar

Lithium – bipolar – absorbed in GI tract, good kidney function A/E treor, thirst, diabetes, acne

DSM-5 Disorders
Most common genetic is Down and Fragile X, FAS Fetal Alcohol is most common intellectual disability
Autism – Boys>girls,

ADHD – boys>girls 10:1, symptoms >6 months, symptoms<12, dopamine imbalance Tx methyphenidate (Ritlalin)
Conduct disorder - <18, boys>girls, theft, lying, break rules, destroy property, genetic?
Oppositional Defiant Disorder – preteen, teens -boys>girls, 6 months negative, hostile defiant against authority
Schizophreniform – hallucinations, delusions, >1 month <6 months
Schizophrenia - >6 months - TX Antipsychotics

Schizoaffective disorder – above + plus mood disorders like depression and mania – 2 weeks of pyscotic
symptoms in absence of mood symptoms

Mood Disorders
SAD – light Persistent Depressive Disorder - symptoms > 2years Bipolar – mania/depression Cyclothymic - >2
years, mood swings, hypomain/depression, TX psychotherapy

Anxiety Disorders
Panic - >1 month, boyd BP, tremors etc. Generalized Anxiety Disorder - >6 months, constant worry,
interferes with life – SSRI buspirone Phobias Body Dysmorphic Disorder - serotonin
Trichotillomania – pull out hair
Anorexia – BMI<17, girls> boys,
Bulimia Nervosa – binge eat then purge, vomit, callous on hand

Factitious disorder is the conscious production of signs and symptoms of a mental or physical illness. There are 2
types: imposed on self and imposed on others.

Paraphilic Disorders
● Pedophilia: sexual urges toward children; most common paraphilia

● Exhibitionism: recurrent desire to expose genitals to stranger

● Voyeurism: sexual pleasure from watching others who are naked, grooming, or having sex; begins early in childhood

● Sadism: sexual pleasure derived from others’ pain

● Masochism: sexual pleasure derived from being abused or dominated

● Fetishism: sexual focus on objects, e.g., shoes, stockings; transvestite fetishism involves fantasies or actual dressing by
heterosexual men in female clothes for sexual arousal

● Frotteurism: male rubbing of genitals against fully clothed woman to achieve orgasm; subways and buses

● Zoophilia: animals preferred in sexual fantasies or practices

● Coprophilia: combining sex and defecation

● Urophilia: combining sex and urination

● Necrophilia: preferred sex with cadavers

● Hypoxyphilia: altered state of consciousness secondary to hypoxia while experiencing orgasm; achieved with
autoerotic asphyxiation, poppers, amyl nitrate, nitric oxide

You might also like